[obm-l] Re: [obm-l] Re: [obm-l] Re: [obm-l] Combinatória

2024-03-13 Por tôpico Claudio Buffara
Mas este caso tem 7 pessoas. E o enunciado fala em 3 A e 3 C.

On Wed, Mar 13, 2024 at 9:28 AM Pedro Júnior 
wrote:

> Eu pensei sim, mas e os casos do tipo ACCACAC. Esse caso não entra na
> conta 6! - 2* 3!* 3!.
>
> Em qua., 13 de mar. de 2024 às 09:09, Claudio Buffara <
> claudio.buff...@gmail.com> escreveu:
>
>> Pense no oposto: de quantas maneiras as crianças e adultos podem se
>> sentar separados uns dos outros.
>>
>> On Wed, Mar 13, 2024 at 8:39 AM Pedro Júnior 
>> wrote:
>>
>>> Olá pessoal, bom dia.
>>> Alguém poderia me ajudar nesse problema?
>>>
>>> Seis poltronas enfileiradas em um cinema e entram 3 adultos e 3
>>> crianças. De quantas maneiras podem sentar-se 2 crianças juntas e dois
>>> adultos juntos?
>>>
>>>
>>> Desde já fico grato!
>>>
>>> --
>>> Esta mensagem foi verificada pelo sistema de antivírus e
>>> acredita-se estar livre de perigo.
>>
>>
>> --
>> Esta mensagem foi verificada pelo sistema de antivírus e
>> acredita-se estar livre de perigo.
>
>
>
> --
>
> Pedro Jerônimo S. de O. Júnior
>
> Professor de Matemática
>
> Geo João Pessoa – PB
>
> --
> Esta mensagem foi verificada pelo sistema de antivírus e
> acredita-se estar livre de perigo.

-- 
Esta mensagem foi verificada pelo sistema de antiv�rus e
 acredita-se estar livre de perigo.



[obm-l] Re: [obm-l] Combinatória

2024-03-13 Por tôpico Claudio Buffara
Pense no oposto: de quantas maneiras as crianças e adultos podem se sentar
separados uns dos outros.

On Wed, Mar 13, 2024 at 8:39 AM Pedro Júnior 
wrote:

> Olá pessoal, bom dia.
> Alguém poderia me ajudar nesse problema?
>
> Seis poltronas enfileiradas em um cinema e entram 3 adultos e 3 crianças.
> De quantas maneiras podem sentar-se 2 crianças juntas e dois adultos juntos?
>
>
> Desde já fico grato!
>
> --
> Esta mensagem foi verificada pelo sistema de antivírus e
> acredita-se estar livre de perigo.

-- 
Esta mensagem foi verificada pelo sistema de antiv�rus e
 acredita-se estar livre de perigo.



Re: [obm-l] Divisibilidade, pedido de esclarecimento

2024-03-02 Por tôpico Claudio Buffara
Isso só perguntando pra quem elaborou a questão.
Mas a ideia pode ter surgido quando, ao manipular expressões desse tipo, a
pessoa notou que:
9r + 5r +4(2r +3s) = 17(r + s)
e isso a fez pensar no enunciado.


On Sat, Mar 2, 2024 at 12:37 PM Marcone Borges 
wrote:

> Sendo r e s inteiros, mostre que 9r +5s divide 17 se, e somente se, 2r +
> 3s divide 17.
> De 9r + 5s ==0(mod 17), assim como de 2r + 3s ==0(mod17), segue que
> r==7s (mod17). Daí sai a resposta.
> Ou podemos mostrar o que foi pedido usando 9r + 5r +4(2r +3s) = 17(r + s)
> Mas, do ponto de vista de quem elaborou a questão, por que vincular essas
> expressões ao fato de que quando uma for um múltiplo de 17 a outra também
> será?
>
> --
> Esta mensagem foi verificada pelo sistema de antivírus e
> acredita-se estar livre de perigo.
>

-- 
Esta mensagem foi verificada pelo sistema de antiv�rus e
 acredita-se estar livre de perigo.



[obm-l] Re: [obm-l] Desigualdade e frações

2024-02-26 Por tôpico Claudio Buffara
Deveria ser a e b inteiros positivos, não?
Pois se forem inteiros sem restrição, então como 2022/2023 < 2022,5/2023,5
< 2023/2024, bastaria tomar a sequência:
a(n) = -20225*n  e  b(n) = -20235*n.
Daí teríamos 2022/2023 < a(n)/b(n) < 2023/2024 e a sequência a(n)+b(n)
seria ilimitada inferiormente.

Assim, suponhamos que a e b sejam inteiros positivos.
2022/2023 < a/b < 2023/2024 implica que b > a+1, já que a sequência
(n/(n+1)) é crescente.
Além disso, usando razões e proporções, achamos que:
2022 < a/(b-a) < 2023 < b/(b-a) < 2024
==> para que a+b seja o menor possível, b-a deverá ser o menor possível.
E o menor valor possível de b-a é 2.
Usando frações equivalentes, dá pra escrever 4044/4046 < a/b < 4046/4048 e
daí teríamos uma única fração a/b com b - a = 2.
Seria a/b = 4045/4047 ==> a+b mínimo = 8092.

[]s,
Claudio.




On Mon, Feb 26, 2024 at 10:12 PM Pedro Júnior 
wrote:

> Quem puder me ajudar, fixo grato.
>
> Sejam a e b dois números inteiros. Sabendo que 2022/2023 < a/b <
> 2023/2024, determine o menos calor da soma a + b.
>
> --
> Esta mensagem foi verificada pelo sistema de antivírus e
> acredita-se estar livre de perigo.

-- 
Esta mensagem foi verificada pelo sistema de antiv�rus e
 acredita-se estar livre de perigo.



[obm-l] Re: Const. de triângulo

2024-01-16 Por tôpico Claudio Buffara
"Há vários problemas de CT com duas soluções."

Claro!...   Fora o óbvio , com infinitas soluções (todas semelhantes
entre si...) tem o  se, por exemplo, A for agudo e a < b < a/sen(A).

O Geogebra certamente é uma tremenda ferramenta.
Mas quantos professores sabem usá-lo adequadamente?

[]s,
Claudio.


On Mon, Jan 15, 2024 at 7:53 PM Luís Lopes  wrote:

> Oi Claudio,
>
> Eu acho que para os problemas no contexto que estamos falando a álgebra
> pode decidir. Como o 17-gon. É construtível mas talvez a construção em si
> poderia não ser conhecida. Os problemas dados 3 pontos da lista do Wernick
> também precisaram de pesquisas para se decidir. Mas não sei muito sobre o
> assunto.
>
> Há vários problemas de CT com duas soluções.
>
> O problema do quadrilátero é muito legal e também muito difícil acho que
> para qualquer um. Há soluções (não sei se são fundamentalmente diferentes)
> no livro do Virgílio, Court e do FG-M.
>
> Mas, pra mim, a principal função destes problemas de construção e’
> pedagógica.
>
> É isso aí. Muita criatividade. E o Geogebra pode ajudar muito.
>
> Abs,
> Luís
>
>
>
> On Jan 14, 2024, at 11:21 AM, Claudio Buffara 
> wrote:
>
> Não tenho dúvidas de que o nível de dificuldade destes problemas varia de
> “trivial” até “extremamente difícil”. Talvez até existam problemas em
> aberto - ninguém acha uma solução e nem consegue provar que não existe
> solução.
>
> O problem dos dados e’ interessante: existem triplas de dados que resultam
> em dois ou mais triângulos não congruentes? Os casos clássicos de
> congruência sugerem que a resposta é não. Mas talvez alguns tipos de dado
> sejam mais “fracos” e não determinem totalmente o triângulo.
>
> Saindo dos triângulos, um legal e não muito fácil (pra mim…) é construir
> um quadrilátero inscritível dados os comprimentos dos lados.
>
> Mas, pra mim, a principal função destes problemas de construção e’
> pedagógica. Inseridos num curso de geometria, eles são uma variante
> interessante de problemas métricos (a enorme maioria dos problemas vistos
> na escola) nos quais os estudantes precisam usar a criatividade pra aplicar
> propriedades básicas de figuras geométricas simples mas de um jeito
> diferente, com muito mais necessidade de visualização.
>
> []s,
> Claudio
>
> Em dom., 14 de jan. de 2024 às 11:41, Luís Lopes 
> escreveu:
>
>> Oi Claudio,
>>
>> Mando pra vc com CC pra lista pra fazer mais um teste e ver se a lista
>> recebe. Reply não funciona.
>>
>> Outra maneira seria usando o triângulo AMaMb. Esse problema é simples.
>> Mais interessantes são (d_a; e_a bissetrizes interna e externa)  e
>>  e os primos esquecidos <,e_a>.
>>
>> Problemas com e_a não são muito vistos. Como aquele que apareceu no
>> WhatsApp do Madeira: construir o triângulo retângulo dados D_b, D_c e X,
>> ponto do incírculo na reta BC. Não considerei com E_b , E_c, a gente acaba
>> esquecendo. Nem sei como seria. Ou até com X_a, ponto do
>> A-exincírculo. A lista é enorme.
>>
>> Considere agora . Tirei o < _a>. Bem fácil. E como dados dois qq
>> entre  o terceiro fica determinado (sem falar em
>> B-C), então  e  também são fáceis. E  cai na
>> categoria  e .
>>
>> O que pode ser um desafio é a discussão sobre os dados nos problemas
>> . Todos eles têm somente uma solução (considerando triângulos não
>> congruentes, a segunda solução no , m>=h> não conta). No  os
>> dados têm que satisfazer d sin(A/2) < h <= d. Para  não sei como
>> determinar.
>>
>> Abs,
>> Luís
>>
>>
>> On Jan 14, 2024, at 7:48 AM, Claudio Buffara 
>> wrote:
>>
>> 
>>
>> Trace AM com comprimento m_a.
>> Trace a circunferência com diâmetro AM.
>> Trace AP com comprimento h_a e P na circunferência.
>>
>> * M será o ponto médio de BC e P o pé da altura relativa a A.
>>
>> Prolonga AM até MA', com AM = MA'.
>>
>> * AA' será a diagonal do paralelogramo ABA'C, cujas diagonais se
>> bissectam em M.
>>
>> Traça arco capaz de 180-A sobre AA'.
>>
>> * Já que, num paralelogramo, ângulos consecutivos são suplementares.
>>
>> Chame de B o ponto de intersecção deste arco capaz com a reta PM.
>> Marque C na reta PM tal que B-M-C e MC = MB.
>> E acabou.
>>
>> Há outra solução marcando P na outra semicircunferência de diâmetro AM (a
>> menos que h_a = m_a).
>>
>> []s,
>> Claudio.
>>
>>
>> On Sun, Jan 14, 2024 at 12:58 AM Luís Lopes 
>> wrote:
>>
>>> Saudações, oi Anderson,
>>>
>>> Soluções usando fórmulas serv

[obm-l] Re: [obm-l] Re: [obm-l] Const. de triângulo

2024-01-14 Por tôpico Claudio Buffara
Não tenho dúvidas de que o nível de dificuldade destes problemas varia de
“trivial” até “extremamente difícil”. Talvez até existam problemas em
aberto - ninguém acha uma solução e nem consegue provar que não existe
solução.

O problem dos dados e’ interessante: existem triplas de dados que resultam
em dois ou mais triângulos não congruentes? Os casos clássicos de
congruência sugerem que a resposta é não. Mas talvez alguns tipos de dado
sejam mais “fracos” e não determinem totalmente o triângulo.

Saindo dos triângulos, um legal e não muito fácil (pra mim…) é construir um
quadrilátero inscritível dados os comprimentos dos lados.

Mas, pra mim, a principal função destes problemas de construção e’
pedagógica. Inseridos num curso de geometria, eles são uma variante
interessante de problemas métricos (a enorme maioria dos problemas vistos
na escola) nos quais os estudantes precisam usar a criatividade pra aplicar
propriedades básicas de figuras geométricas simples mas de um jeito
diferente, com muito mais necessidade de visualização.

[]s,
Claudio

Em dom., 14 de jan. de 2024 às 11:41, Luís Lopes 
escreveu:

> Oi Claudio,
>
> Mando pra vc com CC pra lista pra fazer mais um teste e ver se a lista
> recebe. Reply não funciona.
>
> Outra maneira seria usando o triângulo AMaMb. Esse problema é simples.
> Mais interessantes são (d_a; e_a bissetrizes interna e externa)  e
>  e os primos esquecidos <,e_a>.
>
> Problemas com e_a não são muito vistos. Como aquele que apareceu no
> WhatsApp do Madeira: construir o triângulo retângulo dados D_b, D_c e X,
> ponto do incírculo na reta BC. Não considerei com E_b , E_c, a gente acaba
> esquecendo. Nem sei como seria. Ou até com X_a, ponto do
> A-exincírculo. A lista é enorme.
>
> Considere agora . Tirei o < _a>. Bem fácil. E como dados dois qq
> entre  o terceiro fica determinado (sem falar em
> B-C), então  e  também são fáceis. E  cai na
> categoria  e .
>
> O que pode ser um desafio é a discussão sobre os dados nos problemas
> . Todos eles têm somente uma solução (considerando triângulos não
> congruentes, a segunda solução no , m>=h> não conta). No  os
> dados têm que satisfazer d sin(A/2) < h <= d. Para  não sei como
> determinar.
>
> Abs,
> Luís
>
>
> On Jan 14, 2024, at 7:48 AM, Claudio Buffara 
> wrote:
>
> 
>
> Trace AM com comprimento m_a.
> Trace a circunferência com diâmetro AM.
> Trace AP com comprimento h_a e P na circunferência.
>
> * M será o ponto médio de BC e P o pé da altura relativa a A.
>
> Prolonga AM até MA', com AM = MA'.
>
> * AA' será a diagonal do paralelogramo ABA'C, cujas diagonais se bissectam
> em M.
>
> Traça arco capaz de 180-A sobre AA'.
>
> * Já que, num paralelogramo, ângulos consecutivos são suplementares.
>
> Chame de B o ponto de intersecção deste arco capaz com a reta PM.
> Marque C na reta PM tal que B-M-C e MC = MB.
> E acabou.
>
> Há outra solução marcando P na outra semicircunferência de diâmetro AM (a
> menos que h_a = m_a).
>
> []s,
> Claudio.
>
>
> On Sun, Jan 14, 2024 at 12:58 AM Luís Lopes  wrote:
>
>> Saudações, oi Anderson,
>>
>> Soluções usando fórmulas servem para mostrar que o triângulo é
>> construtível e qual é sua forma e tamanho. Já ajuda naquela parte - suponha
>> o problema resolvido. Mas a construção procurada deverá ser feita usando as
>> propriedades da figura.
>>
>> Posso mandar no privado para quem se interessar as construções com as
>> figuras que um correspondente me enviou. Esse que tem h_c/b como dado é bem
>> interessante.
>>
>> Agora o problema  pode ser resolvido de 3 ou mais maneiras.
>> Com medianas é sempre bom pensar em simetrias e paralelogramos.
>>
>> Luís
>>
>>
>> --
>> Esta mensagem foi verificada pelo sistema de antivírus e
>>  acredita-se estar livre de perigo.
>>
>>
>> =
>> Instru�ões para entrar na lista, sair da lista e usar a lista em
>> http://www.mat.puc-rio.br/~obmlistas/obm-l.html
>> =
>>
>
> --
>
> Esta mensagem foi verificada pelo sistema de antiv�rus e
>
> acredita-se estar livre de perigo.
>
>

-- 
Esta mensagem foi verificada pelo sistema de antiv�rus e
 acredita-se estar livre de perigo.



[obm-l] Re: [obm-l] Const. de triângulo

2024-01-14 Por tôpico Claudio Buffara


Trace AM com comprimento m_a.
Trace a circunferência com diâmetro AM.
Trace AP com comprimento h_a e P na circunferência.

* M será o ponto médio de BC e P o pé da altura relativa a A.

Prolonga AM até MA', com AM = MA'.

* AA' será a diagonal do paralelogramo ABA'C, cujas diagonais se bissectam
em M.

Traça arco capaz de 180-A sobre AA'.

* Já que, num paralelogramo, ângulos consecutivos são suplementares.

Chame de B o ponto de intersecção deste arco capaz com a reta PM.
Marque C na reta PM tal que B-M-C e MC = MB.
E acabou.

Há outra solução marcando P na outra semicircunferência de diâmetro AM (a
menos que h_a = m_a).

[]s,
Claudio.


On Sun, Jan 14, 2024 at 12:58 AM Luís Lopes  wrote:

> Saudações, oi Anderson,
>
> Soluções usando fórmulas servem para mostrar que o triângulo é
> construtível e qual é sua forma e tamanho. Já ajuda naquela parte - suponha
> o problema resolvido. Mas a construção procurada deverá ser feita usando as
> propriedades da figura.
>
> Posso mandar no privado para quem se interessar as construções com as
> figuras que um correspondente me enviou. Esse que tem h_c/b como dado é bem
> interessante.
>
> Agora o problema  pode ser resolvido de 3 ou mais maneiras. Com
> medianas é sempre bom pensar em simetrias e paralelogramos.
>
> Luís
>
>
> --
> Esta mensagem foi verificada pelo sistema de antivírus e
>  acredita-se estar livre de perigo.
>
>
> =
> Instru�ões para entrar na lista, sair da lista e usar a lista em
> http://www.mat.puc-rio.br/~obmlistas/obm-l.html
> =
>

-- 
Esta mensagem foi verificada pelo sistema de antiv�rus e
 acredita-se estar livre de perigo.



Re: [obm-l] OBM 88 Problema 6.

2023-12-29 Por tôpico Claudio Buffara
Dá um Google em "IMO 88".
Vai ter até vídeo com a solução deste problema.

On Thu, Dec 28, 2023 at 4:35 PM Pedro José  wrote:

> Boa tarde!
> Com referência a esse problema criei uma conjectura, não consegui provar
> com a pretensão de abranger todas as soluções da equação:
>
> (a^2+b^2)/(ab+1)= k, com a,b,k Naturais e a>1, b>1 e k>1 Fiz essa
> restrição para retirar as soluções triviais.
> E SPG considerei a>b, já que a=b só ocorre para a=b=1, que está fora pela
> restrição acima e por ser uma equação simétrica em relação à a e b.
> O problema era provar que k era um quadrado perfeito.
> Gostaria de saber se alguém teria conhecimento da resolução em si do
> problema, i.e., quais ternos (a*,b*,k*) são solução da equação.
> Caso ninguém tenha resolvido a equação, ainda, gostaria como faço para dar
> divulgação da minha conjectura, onde tenho a pretenção de ter encontrado
> todas as soluções possíveis para a equação em epígrafe, no Universo dos
> Naturais, com a restrição a>1, b>1 e K>1.
>
> Agradeço quem puder me orientar.
>
> Cordialmente,
> PJMS
>
> --
> Esta mensagem foi verificada pelo sistema de antivírus e
> acredita-se estar livre de perigo.

-- 
Esta mensagem foi verificada pelo sistema de antiv�rus e
 acredita-se estar livre de perigo.



[obm-l] Re: [obm-l] Re: [obm-l] Re: [obm-l] Pesagens ( Balança Eletrônica)

2023-11-19 Por tôpico Claudio Buffara
Por que você não começa com um caso menor, tal como 4, 6 ou 9 moedas no
total?

Como você não consegue distinguir, numa dada pesagem, um grupo só com
moedas verdadeiras e um grupo com 2 moedas falsas, um algoritmo
pra resolver este problema com o menor número possível de pesagens não me
parece óbvio.  Daí, a análise de um caso menor pode dar alguma luz.

[]s,
Claudio.


On Sun, Nov 19, 2023 at 3:50 PM Jeferson Almir 
wrote:

> Eu tinha errado umas contas, mas sua cota está correta Ralph, preciso
> montar um exemplo com 21 pesagens
>
> Em dom., 19 de nov. de 2023 às 15:00, Jeferson Almir <
> jefersonram...@gmail.com> escreveu:
>
>> Pelo visto, está sim Ralph!! Agora temos que montar uma estratégia que
>> com 21 pesagens.
>>
>> Em dom., 19 de nov. de 2023 às 13:55, Ralph Costa Teixeira <
>> ralp...@gmail.com> escreveu:
>>
>>> Existem 2022*2021/2 possibilidades para as 2 falsas. Qualquer estratégia
>>> que seja criada com k pesagens que dão apenas 2 respostas cada distingue no
>>> máximo dentre 2^k possibilidades. Então devemos ter 2^k >= 2022*2021/2...
>>> hmm, isso daria k como pelo menos 21? Errei algo?
>>>
>>> On Sun, Nov 19, 2023, 12:16 Jeferson Almir 
>>> wrote:
>>>
 Amigos, encontrei como K mínimo o valor 11 mas desconfio que seja
 menos. Se alguém souber uma ideia que acabe o problema serei grato.


 Em Villa Par todas as moedas autênticas pesam uma quantidade par de
 gramas e todas as moedas falsas pesam uma quantidade impar de gramas.

 Se você tiver 2022 moedas entre as quais sabe que exatamente 2 são
 falsas.

 Se tiver uma balança eletrônica que informe apenas se o peso total dos
 objetos colocados nela é par ou impar.

 Determine o valor mínimo de k para qualquer estratégia que permita
 identificar as moedas falsas usando a balança  no máximo k vezes.

 --
 Esta mensagem foi verificada pelo sistema de antivírus e
 acredita-se estar livre de perigo.
>>>
>>>
>>> --
>>> Esta mensagem foi verificada pelo sistema de antivírus e
>>> acredita-se estar livre de perigo.
>>
>>
> --
> Esta mensagem foi verificada pelo sistema de antivírus e
> acredita-se estar livre de perigo.

-- 
Esta mensagem foi verificada pelo sistema de antiv�rus e
 acredita-se estar livre de perigo.



Re: [obm-l] x^x^x^x....=2 e x^x^x...=4

2023-11-01 Por tôpico Claudio Buffara
Se entendi direito, você pegou L = 15 e fez x = 15^(1/15) = 1,19786.  Foi
isso?
Mas este x está no intervalo [e^(-e), e^(1/e)].
Daí, pra este x, a sequência converge (pra 1,254088...).

Pra x > 1, quando você aumenta a "quantidade de x" o valor da torre de
expoentes aumenta.
Ou seja, x > 1 ==> x < x^x < x^x^x < ...
Mas o que acontece é que, para x > e^(1/e), a sequência (x, x^x, x^x^x, ...
)  cresce para além de qualquer limite (ou seja, diverge para +infinito).
E para 1 < x <= e^(1/e), ela converge para um limite <= e.
Não tem "meio-termo", ou seja, não existe x tal que x^x^x^... = 4 ou
qualquer outro número > e.

[]s,
Claudio.






On Wed, Nov 1, 2023 at 6:38 PM Pacini Bores  wrote:

> Oi Claudio, mas sabe,  o que mais me incomoda é o fato de que em  lnx =
> lnL/L, se tomarmos a função g(L) = lnL/L , teremos  0< g(L) <= 1/e. Para
> um único valor de "x" temos dois valores para L e, daí reforçando ( não sei
> se estou bobeando em algo) a ideia  de que na hipótese de existir lim
> a(n+1) = lim a(n) = L ,e se tomarmos  L=15 por exemplo , teremos um único
> "x" no intervalo em que colocastes anteriormente. No Wolfram ou geogebra
> fui fazendo f(x)= x^x^x... com o aumento na quantidade  de"x" , o gráfico
> me pareceu crescente a partir de um certo momento e tendo sempre uma reta
> paralela ao eixo horizontal intersectando sempre o gráfico de "f(x)" . Ou
> seja, aquele fato de que x^x^x...=4 e dizer que é impossível me causou
> estranheza. Desculpem se estou cometendo erros conceituais, mas de qualquer
> forma agradeço a  atenção de todos.
>
> Pacini
>
> Em qua., 1 de nov. de 2023 às 16:17, Claudio Buffara <
> claudio.buff...@gmail.com> escreveu:
>
>> Dando um Google em x^x^x, eu achei sites que NADA tinham a ver com este
>> problema...
>> Mas procurando um pouco mais, achei a afirmação (sem demonstração) de que
>> a sequência converge para e^(-e) <= x <= e^(1/e).
>> Explorando numericamente, me convenci de que isso está (provavelmente)
>> correto.
>> Ou seja, dado x naquele intervalo, existe L tal que x^L = L
>> Em particular, L = 1/e ==> (e^(-e))^(1/e) = 1/e,  e  L = e ==>
>> (e^(1/e))^e = e.
>> Ou seja, minha conjectura é: a função f é crescente, tem domínio
>> [e^(-e),e^(1/e)] e imagem [1/e,e].
>>
>> []s,
>> Claudio.
>>
>> On Wed, Nov 1, 2023 at 1:21 PM Claudio Buffara 
>> wrote:
>>
>>> A ideia me parece ser definir a sequência (a(n)) por:
>>> a(0) = x   e   a(n+1) = x^a(n)
>>> e daí ver para que valores de x ela converge e, se convergir, para qual
>>> limite.
>>>
>>> Se a(n) convergir para L, então  x^L = L.
>>>
>>> Com L = 2 e L = 4, x^L = L implica que x = raiz(2).
>>>
>>> Explorando numericamente com uma planilha, eu noto que para x = raiz(2),
>>> a sequência parece convergir para 2.
>>>
>>> O problema pode ser reformulado como sendo o de obter o maior intervalo
>>> I de R para o qual é possível definir uma função f:I -> R tal que f(x) =
>>> limite da sequência (a(n)) acima com valor inicial a(0) = x.
>>> Daí, a análise informal acima sugere que raiz(2) pertence a I,
>>> f(raiz(2)) = 2, e 4 não pertence a f(I).
>>>
>>> O que você está dizendo é que e^(1/e) = sup(I).  Vamos ver...
>>>
>>> Se f(x) = L, então x^L = L ==> x = L^(1/L).
>>> Agora, a função g(L) = L^(1/L) atinge seu valor máximo, igual a e^(1/e),
>>> para L = e.
>>> ( g(L) = e^log(L^(1/L)) = e^(log(L)/L) ==> g'(L) = g(L)*(1 - log(L))/L^2
>>> = 0 para L = e )
>>> Assim, se f(x) está definida, deve ser x <= e^(1/e).
>>> Além disso, numericamente parece plausível que f(e^(1/e)) = e.
>>> Se este for o caso, então, dado que e < 4, realmente 4 não pertence à
>>> imagem de f.
>>>
>>> []s,
>>> Claudio.
>>>
>>>
>>>
>>> On Wed, Nov 1, 2023 at 8:47 AM Pacini Bores 
>>> wrote:
>>>
>>>> Olá pessoal, gostaria da opinão de vocês com relação a essas duas
>>>> equações, em que ambas , é claro garantindo a convergência, temos a mesma
>>>> resposta para "x". O que muitos falam que a segunda igualdade não é
>>>> possível. O que me intriga é que é possível mostrar( se não estiver
>>>> errado), é que o "x"  é que varia entre "0" e  " e^(1/e)" para que a
>>>> igualdade x^x^x..=k(k>0) e não o "k". Ou seja, há dois valores possíveis
>>>> para "k", enquanto há apenas um valor para "x".
>>>>
>>>> A minha pergunta : Estou errando em algo ?
>>>>
>>>> Pacini
>>>>
>>>> --
>>>> Esta mensagem foi verificada pelo sistema de antivírus e
>>>> acredita-se estar livre de perigo.
>>>
>>>
>> --
>> Esta mensagem foi verificada pelo sistema de antivírus e
>> acredita-se estar livre de perigo.
>
>
> --
> Esta mensagem foi verificada pelo sistema de antivírus e
> acredita-se estar livre de perigo.

-- 
Esta mensagem foi verificada pelo sistema de antiv�rus e
 acredita-se estar livre de perigo.



Re: [obm-l] x^x^x^x....=2 e x^x^x...=4

2023-11-01 Por tôpico Claudio Buffara
Dando um Google em x^x^x, eu achei sites que NADA tinham a ver com este
problema...
Mas procurando um pouco mais, achei a afirmação (sem demonstração) de que a
sequência converge para e^(-e) <= x <= e^(1/e).
Explorando numericamente, me convenci de que isso está (provavelmente)
correto.
Ou seja, dado x naquele intervalo, existe L tal que x^L = L
Em particular, L = 1/e ==> (e^(-e))^(1/e) = 1/e,  e  L = e ==> (e^(1/e))^e
= e.
Ou seja, minha conjectura é: a função f é crescente, tem domínio
[e^(-e),e^(1/e)] e imagem [1/e,e].

[]s,
Claudio.

On Wed, Nov 1, 2023 at 1:21 PM Claudio Buffara 
wrote:

> A ideia me parece ser definir a sequência (a(n)) por:
> a(0) = x   e   a(n+1) = x^a(n)
> e daí ver para que valores de x ela converge e, se convergir, para qual
> limite.
>
> Se a(n) convergir para L, então  x^L = L.
>
> Com L = 2 e L = 4, x^L = L implica que x = raiz(2).
>
> Explorando numericamente com uma planilha, eu noto que para x = raiz(2), a
> sequência parece convergir para 2.
>
> O problema pode ser reformulado como sendo o de obter o maior intervalo I
> de R para o qual é possível definir uma função f:I -> R tal que f(x) =
> limite da sequência (a(n)) acima com valor inicial a(0) = x.
> Daí, a análise informal acima sugere que raiz(2) pertence a I, f(raiz(2))
> = 2, e 4 não pertence a f(I).
>
> O que você está dizendo é que e^(1/e) = sup(I).  Vamos ver...
>
> Se f(x) = L, então x^L = L ==> x = L^(1/L).
> Agora, a função g(L) = L^(1/L) atinge seu valor máximo, igual a e^(1/e),
> para L = e.
> ( g(L) = e^log(L^(1/L)) = e^(log(L)/L) ==> g'(L) = g(L)*(1 - log(L))/L^2 =
> 0 para L = e )
> Assim, se f(x) está definida, deve ser x <= e^(1/e).
> Além disso, numericamente parece plausível que f(e^(1/e)) = e.
> Se este for o caso, então, dado que e < 4, realmente 4 não pertence à
> imagem de f.
>
> []s,
> Claudio.
>
>
>
> On Wed, Nov 1, 2023 at 8:47 AM Pacini Bores  wrote:
>
>> Olá pessoal, gostaria da opinão de vocês com relação a essas duas
>> equações, em que ambas , é claro garantindo a convergência, temos a mesma
>> resposta para "x". O que muitos falam que a segunda igualdade não é
>> possível. O que me intriga é que é possível mostrar( se não estiver
>> errado), é que o "x"  é que varia entre "0" e  " e^(1/e)" para que a
>> igualdade x^x^x..=k(k>0) e não o "k". Ou seja, há dois valores possíveis
>> para "k", enquanto há apenas um valor para "x".
>>
>> A minha pergunta : Estou errando em algo ?
>>
>> Pacini
>>
>> --
>> Esta mensagem foi verificada pelo sistema de antivírus e
>> acredita-se estar livre de perigo.
>
>

-- 
Esta mensagem foi verificada pelo sistema de antiv�rus e
 acredita-se estar livre de perigo.



Re: [obm-l] x^x^x^x....=2 e x^x^x...=4

2023-11-01 Por tôpico Claudio Buffara
A ideia me parece ser definir a sequência (a(n)) por:
a(0) = x   e   a(n+1) = x^a(n)
e daí ver para que valores de x ela converge e, se convergir, para qual
limite.

Se a(n) convergir para L, então  x^L = L.

Com L = 2 e L = 4, x^L = L implica que x = raiz(2).

Explorando numericamente com uma planilha, eu noto que para x = raiz(2), a
sequência parece convergir para 2.

O problema pode ser reformulado como sendo o de obter o maior intervalo I
de R para o qual é possível definir uma função f:I -> R tal que f(x) =
limite da sequência (a(n)) acima com valor inicial a(0) = x.
Daí, a análise informal acima sugere que raiz(2) pertence a I, f(raiz(2)) =
2, e 4 não pertence a f(I).

O que você está dizendo é que e^(1/e) = sup(I).  Vamos ver...

Se f(x) = L, então x^L = L ==> x = L^(1/L).
Agora, a função g(L) = L^(1/L) atinge seu valor máximo, igual a e^(1/e),
para L = e.
( g(L) = e^log(L^(1/L)) = e^(log(L)/L) ==> g'(L) = g(L)*(1 - log(L))/L^2 =
0 para L = e )
Assim, se f(x) está definida, deve ser x <= e^(1/e).
Além disso, numericamente parece plausível que f(e^(1/e)) = e.
Se este for o caso, então, dado que e < 4, realmente 4 não pertence à
imagem de f.

[]s,
Claudio.



On Wed, Nov 1, 2023 at 8:47 AM Pacini Bores  wrote:

> Olá pessoal, gostaria da opinão de vocês com relação a essas duas
> equações, em que ambas , é claro garantindo a convergência, temos a mesma
> resposta para "x". O que muitos falam que a segunda igualdade não é
> possível. O que me intriga é que é possível mostrar( se não estiver
> errado), é que o "x"  é que varia entre "0" e  " e^(1/e)" para que a
> igualdade x^x^x..=k(k>0) e não o "k". Ou seja, há dois valores possíveis
> para "k", enquanto há apenas um valor para "x".
>
> A minha pergunta : Estou errando em algo ?
>
> Pacini
>
> --
> Esta mensagem foi verificada pelo sistema de antivírus e
> acredita-se estar livre de perigo.

-- 
Esta mensagem foi verificada pelo sistema de antiv�rus e
 acredita-se estar livre de perigo.



[obm-l] Re: [obm-l] Números primos

2023-10-04 Por tôpico Claudio Buffara
Fatoração, com certeza.  Por exemplo, diga pra garotada analisar os números
de 2 a 100 e determinar quais podem ser expressos como produto de números
naturais menores.  Como dica, pra facilitar o trabalho, diga pra eles
consultarem a tabuada (e também pra observarem que, na tabuada, nem todos
os números aparecem como resultado de alguma multiplicação).  Acho que
essa é uma boa motivação pra definição de número primo.
As dificuldades encontradas por eles nesta tarefa podem motivar a busca de
uma forma sistemática (um algoritmo) pra determinar os números primos na
sequência de números naturais. Esse seria o crivo de Eratóstenes, cuja
descoberta poderia ser guiada por perguntas e dicas pertinentes.

Outra forma de motivar a definição de primo é representar o natural N (N =
1, 2, 3, ...) por N bolinhas, que devem ser dispostas num arranjo
retangular com 2 ou mais linhas (ou colunas).  Para alguns valores de N,
isso será impossível.  Estes são os números primos.
Numa digressão, faça a garotada determinar pra quais N as bolinhas podem
ser particionadas em pares (conjuntos com 2 elementos)... daí o nome.
Há vários probleminhas interessantes que podem ser resolvidos com esta
representação dos números - o do jovem Gauss, por exemplo, ou o da soma dos
ímpares consecutivos, ou determinar pra quais N o arranjo pode ter o mesmo
número de linhas e de colunas.

[]s,
Claudio.


On Wed, Oct 4, 2023 at 3:49 PM carlos h Souza  wrote:

> Boa tarde,
>
> Para fins didáticos é mais fácil encontrar os números primos em forma de
> fatoração numérica ou usar o Crivo de Eratóstenes ?
>
> Obrigados a todos.
>
> --
> Esta mensagem foi verificada pelo sistema de antivírus e
> acredita-se estar livre de perigo.

-- 
Esta mensagem foi verificada pelo sistema de antiv�rus e
 acredita-se estar livre de perigo.



Re: [obm-l] Contagem

2023-08-08 Por tôpico Claudio Buffara
O que vc quer é o número de PAs de 3 termos distintos contidas em
{1,2,3,...,2023}.
Daí dá pra enumerar na mão e achar o padrão:
(1,2,3), (2,3,4), ..., (2021,2022,2023) ==> 2021 PAs de razão 1
(1,3,5), (2,4,6), ..., (2019,2021,2023) ==> 2019 PAs de razão 2
(1,4,7), (2,6,8), ..., (2017,2020,2023) ==> 2017 PAs de razão 3
...
(1,1011,2021), (2,1012,2022), (3,2023,2023) ==> 3 PAs de razão 1010
(1,1012,2023) ==> 1 PA de razão 1011.
Total = 1+3+5+...+2017+2019+2021 = 1011^2 = 1022121 PAs.


On Tue, Aug 8, 2023 at 1:02 PM Jamil Silva  wrote:

> Quantos conjuntos de três números inteiros positivos menores ou iguais a
> 2023 contêm a média aritmética de seus elementos ?
>
> --
> Esta mensagem foi verificada pelo sistema de antivírus e
> acredita-se estar livre de perigo.

-- 
Esta mensagem foi verificada pelo sistema de antiv�rus e
 acredita-se estar livre de perigo.



Re: [obm-l] Propriedade do no 7

2023-05-12 Por tôpico Claudio Buffara
Iniciando uma discussão sobre pedagogia...

Quem acha que esse tipo de problema deveria fazer parte do currículo de
álgebra na escola, digamos a partir do 8o ou 9o ano?
Há vários outros na mesma linha...
1) Ache todas as triplas de primos trigêmeos (triplas de naturais da forma
(n,n+2,n+4) em que todos são primos);
2) O quadrado de um inteiro ímpar excede em 1 algum múltiplo de 8;
3) Ache todos os primos da forma n^4 + 4^m
4) Ache todos os naturais que podem ser expressos como uma diferença de
quadrados de naturais.  Pra quais naturais essa expressão é única?
etc...

Problemas do tipo "ache todos" ou "prove que não existe"  são interessantes
porque mostram pra garotada que nem  todo problema tem uma solução única e
que provar que um dado problema não tem solução também é, de certa forma,
uma solução.

[]s,
Claudio.


On Thu, May 11, 2023 at 10:57 AM Caio Costa  wrote:

> N = n³-1 = (n-1)*(n² + n + 1).
> n-1 divide n³ - 1, logo se n³ -1 é primo, então n-1 = 1, daí n = 2 e N = 7.
>
> Em qui., 11 de mai. de 2023 às 11:23, Luiz Alberto Salomao <
> ladsalo...@gmail.com> escreveu:
>
>> Olá, Artur
>> Cultura sempre é útil. Muito bacana!
>> Você conhece alguma prova desse resultado?
>> Luiz Alberto.
>>
>> Em qui., 11 de mai. de 2023 às 08:20, Artur Costa Steiner <
>> artur.costa.stei...@gmail.com> escreveu:
>>
>>> 7 é o único primo seguido por um cubo. Alguns talvez achem isso uma
>>> curiosidade interessante. Outros talvez achem cultura inútil.rsss
>>>
>>> Artur
>>>
>>> --
>>> Esta mensagem foi verificada pelo sistema de antivírus e
>>> acredita-se estar livre de perigo.
>>
>>
>> --
>> Esta mensagem foi verificada pelo sistema de antivírus e
>> acredita-se estar livre de perigo.
>
>
> --
> Esta mensagem foi verificada pelo sistema de antivírus e
> acredita-se estar livre de perigo.

-- 
Esta mensagem foi verificada pelo sistema de antiv�rus e
 acredita-se estar livre de perigo.



[obm-l] Re: [obm-l] Re: [obm-l] cadeira de 3 pés

2023-01-23 Por tôpico Claudio Buffara
" No mundo 2D um banco de 2 pernas não balança, mas um de 3 pernas pode
balançar."

Aqui estamos falando de uma curva no plano e de um número real positivo d,
suficientemente pequeno de modo que, para todo ponto A na curva, existe um
ponto B na curva com dist(A,B) = d.
A e B são as pontas das pernas do banco 2D.
Isso será verdade se todo ponto da curva for o centro de uma circunferência
de raio d que intersecta a curva.

Um terceiro ponto C, a distâncias predeterminadas de A e B, pode não
pertencer à curva (ou seja, o triângulo ABC não estará inscrito na curva).
Mas é possível deslizar a corda AB, de comprimento constante, ao longo de
toda a curva.

No caso 3D, temos uma superfície no espaço e 3 números reais positivos a,
b, c, com cada um menor do que a soma dos outros dois, de modo que, para
cada A na superfície, é possível achar B e C na superfície com dist(A,B) =
c, dist(A,C) = b e dist(B,C) = a.
E é possível deslizar o triângulo ABC, com lados de comprimento constante,
ao longo de toda a superfície.
Mas, fixado A na superfície, não me parece tão simples construir B e C.


On Mon, Jan 23, 2023 at 11:29 AM Eduardo Wagner  wrote:

> O banco de 3 pernas não balança porque nosso mundo é tridimensional.
> Não tem nada a ver com plano ou triângulo. Um banco de 3 pernas não
> balança se for colocado
> no teto de um carro.
> No mundo 2D um banco de 2 pernas não balança, mas um de 3 pernas pode
> balançar.
> Em um mundo 4D uma cadeira de 4 pernas não balança.
> Wbs
> Wagner
>
>
> Em dom., 22 de jan. de 2023 às 23:24, Claudio Buffara <
> claudio.buff...@gmail.com> escreveu:
>
>> Achei na internet duas explicações distintas para a estabilidade de uma
>> cadeira (ou mesa ou banco) de 3 pés.
>> Aqui estão:
>> https://www.somatematica.com.br/curiosidades/c98.php
>>
>> http://colegiofarroupilha.com.br/site/qual-cadeira-e-mais-firme-a-que-tem-tres-ou-quatro-pes/
>>
>> Qual das duas é a explicação correta?
>> Ou nenhuma das duas? E, nesse caso, qual a explicação?
>>
>> []s,
>> Claudio.
>>
>> --
>> Esta mensagem foi verificada pelo sistema de antivírus e
>> acredita-se estar livre de perigo.
>
>
> --
> Esta mensagem foi verificada pelo sistema de antivírus e
> acredita-se estar livre de perigo.

-- 
Esta mensagem foi verificada pelo sistema de antiv�rus e
 acredita-se estar livre de perigo.



[obm-l] Re: [obm-l] Re: [obm-l] cadeira de 3 pés

2023-01-23 Por tôpico Claudio Buffara
Obrigado, Wagner e Ponce:

Eu tinha pensado em algo na linha do que o Ponce escreveu, ainda que em
certos casos patológicos (pelo menos de piso...) o terceiro pé pode não
encontrar apoio: imagine um piso com um pico fino em algum lugar (p.ex. a
superfície gerada pela revolução de z = 1/(1+(x^2+y^2)^20) em torno do eixo
z). Neste caso, precisaríamos de uma cadeira bem pequena, ou pelo menos com
as pontas dos pés bem próximas umas das outras.

Mas, pelo que o Wagner escreveu, acho que ainda tem um teorema mais
profundo aí.

[]s,
Claudio.


On Mon, Jan 23, 2023 at 11:54 AM Rogerio Ponce  wrote:

> Ola' Claudio!
> Eu diria que as duas explicações estão erradas, pois não se depende de
> ter apenas um plano definido pelas pontas dos pés, visto que uma
> cadeira de 4 pés pode, perfeitamente, ter as pontas dos 4 pés em
> apenas um plano, e, ainda assim, ela não é necessariamente estável.
>
> Um explicação menos ruim é que, numa cadeira de 3 pés, sempre podemos
> apoiar quaisquer 2 pés num piso (mesmo irregular), e, em torno do eixo
> definido pelos 2 pés já apoiados, podemos girar a cadeira até que o
> terceiro pé encontre o piso, de modo que a cadeira fique totalmente
> apoiada.
> Já numa cadeira de 4 pés, é comum que um dos pés fique sem contato com
> o chão, permitindo que a cadeira oscile em torno do eixo definido
> pelos 2 pés vizinhos ao pé sem contato.
>
> []'s
> Rogerio Ponce
>
> On Sun, Jan 22, 2023 at 11:23 PM Claudio Buffara
>  wrote:
> >
> > Achei na internet duas explicações distintas para a estabilidade de uma
> cadeira (ou mesa ou banco) de 3 pés.
> > Aqui estão:
> > https://www.somatematica.com.br/curiosidades/c98.php
> >
> http://colegiofarroupilha.com.br/site/qual-cadeira-e-mais-firme-a-que-tem-tres-ou-quatro-pes/
> >
> > Qual das duas é a explicação correta?
> > Ou nenhuma das duas? E, nesse caso, qual a explicação?
> >
> > []s,
> > Claudio.
> >
> > --
> > Esta mensagem foi verificada pelo sistema de antivírus e
> > acredita-se estar livre de perigo.
>
> --
> Esta mensagem foi verificada pelo sistema de antivírus e
>  acredita-se estar livre de perigo.
>
>
> =
> Instru�ões para entrar na lista, sair da lista e usar a lista em
> http://www.mat.puc-rio.br/~obmlistas/obm-l.html
> =
>

-- 
Esta mensagem foi verificada pelo sistema de antiv�rus e
 acredita-se estar livre de perigo.



[obm-l] Re: [obm-l] Re: [obm-l] cadeira de 3 pés

2023-01-23 Por tôpico Claudio Buffara
Será que o argumento usando apenas o plano é suficiente?  Pois um banco de
3 pés também fica estável num piso irregular.
O resultado mais geral em que pensei foi o seguinte: dada qualquer
superfície bi-dimensional contínua (por exemplo, que seja o gráfico de uma
função contínua de RxR em R - uma suposição razoável se estamos tentando
modelar um piso), você sempre consegue encostar nela as pontas dos 3 pés do
banco, de modo que o banco fique "firme" ou sem folgas.
Ou, mais formalmente, dado um triângulo ABC no espaço, existe uma isometria
(do espaço) tal que as imagens de A, B e C por esta isometria estão em S.

Enfim, esse talvez seja um problema mais de topologia do que de geometria.
Pois, no fim das contas, "3 pontos não colineares determinam um único
plano", assim como "2 pontos determinam uma única reta" são afirmações que
têm um certo ar topológico, pelo menos pra mim.

[]s,
Claudio.

On Mon, Jan 23, 2023 at 7:02 AM Anderson Torres <
torres.anderson...@gmail.com> wrote:

>
>
> Em dom, 22 de jan de 2023 23:23, Claudio Buffara <
> claudio.buff...@gmail.com> escreveu:
>
>> Achei na internet duas explicações distintas para a estabilidade de uma
>> cadeira (ou mesa ou banco) de 3 pés.
>> Aqui estão:
>> https://www.somatematica.com.br/curiosidades/c98.php
>>
>> http://colegiofarroupilha.com.br/site/qual-cadeira-e-mais-firme-a-que-tem-tres-ou-quatro-pes/
>>
>
> Nesse caso específico, a primeira me parece mais correta. Ou melhor, a
> segunda tem falhas.
>
> A rigidez dos triângulos (TCC caso LLL de igualdade de triângulos) é
> irrelevante para a questão da cadeira bamba. Em qualquer cadeira física e
> palpável, as pernas são rígidas - portanto o polígono formado pelas
> extremidades dessas pernas é rígido também.
>
> Mas um polígono rígido não é necessariamente um polígono bidimensional - o
> que a cadeira bamba de 4 pernas exemplifica perfeitamente.
>
> Contate o webmaster da segunda página sugerindo correções!
>
>
>> Qual das duas é a explicação correta?
>> Ou nenhuma das duas? E, nesse caso, qual a explicação?
>>
>> []s,
>> Claudio.
>>
>> --
>> Esta mensagem foi verificada pelo sistema de antivírus e
>> acredita-se estar livre de perigo.
>
>
> --
> Esta mensagem foi verificada pelo sistema de antivírus e
> acredita-se estar livre de perigo.

-- 
Esta mensagem foi verificada pelo sistema de antiv�rus e
 acredita-se estar livre de perigo.



[obm-l] cadeira de 3 pés

2023-01-22 Por tôpico Claudio Buffara
Achei na internet duas explicações distintas para a estabilidade de uma
cadeira (ou mesa ou banco) de 3 pés.
Aqui estão:
https://www.somatematica.com.br/curiosidades/c98.php
http://colegiofarroupilha.com.br/site/qual-cadeira-e-mais-firme-a-que-tem-tres-ou-quatro-pes/

Qual das duas é a explicação correta?
Ou nenhuma das duas? E, nesse caso, qual a explicação?

[]s,
Claudio.

-- 
Esta mensagem foi verificada pelo sistema de antiv�rus e
 acredita-se estar livre de perigo.



Re: [obm-l] Irracionalidade de Pi

2023-01-22 Por tôpico Claudio Buffara
Valeu!
E os links extras contém uma boa discussão, às vezes meio acalorada, sobre
motivação pra certas demonstrações.
Eu particularmente me interesso bastante por este tema.
Pois acho que demonstrações "mágicas", baseadas em ideias "vindas do além",
são problemáticas do ponto de vista pedagógico, pois acho que podem
desmotivar estudantes de matemática, que passam a achar que o assunto é só
pra gênios.
Estas não devem ser confundidas com demonstrações/soluções brilhantes mas
que são "óbvias a posteriori", ou seja, que dependem de uma sacada que o
leitor poderia ter tido se tivesse prestado mais atenção ou feito um
desenho mais preciso ou pensado um pouquinho mais no problema.  Não me
parece ser o caso dessa demonstração do Niven da irracionalidade de Pi.

[]s,
Claudio.




On Sun, Jan 22, 2023 at 9:53 AM Anderson Torres <
torres.anderson...@gmail.com> wrote:

> Em sáb., 21 de jan. de 2023 às 13:27, Claudio Buffara
>  escreveu:
> >
> > A demonstração tradicional da irracionalidade de Pi começa estabelecendo
> algumas propriedades da função:
> > x |--> x^n * (1-x)^n / n!
> > no intervalo (0,1).
> >
> > Essa função me parece tirada da cartola, sem qualquer motivação prévia.
> > Alguém sabe o que levou o autor da demonstração a usar esta função?
>
> Bem, eu fiz uma rápida busca no Google por "motivated demonstration
> irrationality pi" e encontrei isso:
>
> "Discovering and Proving that π Is Irrational" por Timothy W. Jones
>
> The American Mathematical Monthly
> Vol. 117, No. 6 (June-July 2010), pp. 553-557 (5 pages)
> https://doi.org/10.4169/000298910x492853
>
> E também uns links extras:
>
>
> https://mattbaker.blog/2015/03/15/a-motivated-and-simple-proof-that-pi-is-irrational/
>
> https://math.stackexchange.com/questions/4051354/what-is-the-motivation-behind-the-steps-in-this-simple-proof-that-pi-is-irr
> https://page.math.tu-berlin.de/~mdmv/archive/19/mdmv-19-zhou.pdf
>
> Divirta-se :)
>
> >
> > []s,
> > Claudio.
> >
> > --
> > Esta mensagem foi verificada pelo sistema de antivírus e
> > acredita-se estar livre de perigo.
>
> --
> Esta mensagem foi verificada pelo sistema de antivírus e
>  acredita-se estar livre de perigo.
>
>
> =
> Instru�ões para entrar na lista, sair da lista e usar a lista em
> http://www.mat.puc-rio.br/~obmlistas/obm-l.html
> =
>

-- 
Esta mensagem foi verificada pelo sistema de antiv�rus e
 acredita-se estar livre de perigo.



[obm-l] Irracionalidade de Pi

2023-01-21 Por tôpico Claudio Buffara
A demonstração tradicional da irracionalidade de Pi começa estabelecendo
algumas propriedades da função:
x |--> x^n * (1-x)^n / n!
no intervalo (0,1).

Essa função me parece tirada da cartola, sem qualquer motivação prévia.
Alguém sabe o que levou o autor da demonstração a usar esta função?

[]s,
Claudio.

-- 
Esta mensagem foi verificada pelo sistema de antiv�rus e
 acredita-se estar livre de perigo.



[obm-l] Probabilidade

2022-12-15 Por tôpico Claudio Buffara
Alguém conhece alguma boa referência para métodos de resolução de problemas
do tipo:
1) Joga-se uma moeda 1000 vezes. Qual a probabilidade de se ter uma
sequência de exatamente 20 caras consecutivas?  De pelo menos 20 caras
consecutivas?
1a) Analogamente com um dado.
2) Dadas 100 amostras independentes de uma distribuição uniforme em [0,1],
qual a probabilidade de (exatamente ou pelo menos) 20 delas estarem no
intervalo [2/5,1/2]?  Qual a probabilidade de 20 delas estarem num
intervalo de comprimento 1/10?

Também tenho interesse em bibliografia sobre testes de aleatoriedade
aplicados a situações como as acima (p.ex. dados os resultados de 1000
lançamentos de uma moeda, qual a probabilidade da moeda ser honesta/dos
lançamentos terem sido independentes)

[]s,
Claudio.

-- 
Esta mensagem foi verificada pelo sistema de antiv�rus e
 acredita-se estar livre de perigo.



[obm-l] Re: [obm-l] OBM e Olímpiadas internacionais

2022-12-07 Por tôpico Claudio Buffara
Eu começaria olhando as provas de anos anteriores, por exemplo aqui:
https://www.obm.org.br/como-se-preparar/provas-e-gabaritos/

On Wed, Dec 7, 2022 at 3:39 AM Obindinachukwu Desire Yema <
obindinachukwu.y...@usp.br> wrote:

> Bom dia a todos,
> Nesse ano eu despertei um interesse em matemática pura, pensando um pouco
> decidi que iria tentar no próximo ano fazer a OBM nivel universitário.
> Pesquisando no site da OBM, eu não achei nada relacionado com o conteúdo
> que cai na prova.
> Eu queria perguntar para vocês como que me preparo para a prova, no
> sentido de: conteúdo que devo saber.
> Desde já agradeço a atenção.
>
> --
> Esta mensagem foi verificada pelo sistema de antivírus e
> acredita-se estar livre de perigo.

-- 
Esta mensagem foi verificada pelo sistema de antiv�rus e
 acredita-se estar livre de perigo.



[obm-l] Re: [obm-l] Re: [obm-l] Re: [obm-l] Caracterização de Inteiros

2022-11-17 Por tôpico Claudio Buffara
Só completando...

Apesar de números irracionais serem conhecidos desde a época de Pitágoras
(vide a famosa historinha do pitagórico Hipaso, que supostamente foi
afogado por ter "vazado" o segredo da existência dos irracionais), me
parece que eles só começaram a realmente fazer falta no século 19, quando
os matemáticos estavam preocupados em formalizar a análise. Foi naquele
ponto que o Dedekind se deu conta da importância da completeza (essa
palavra tá certa?...) dos reais e da necessidade dela ser garantida por um
axioma. Se não me engano, o teorema que ele queria provar é o de que uma
sequência monótona e limitada de números reais sempre converge. E descobriu
que não conseguiria sem fazer uma hipótese adicional sobre o corpo
dos reais, capaz de diferenciá-lo dos racionais (também um corpo ordenado),
mas dentro do qual nem toda sequência converge (por exemplo, x(n+1) =
(1/2)*(x(n) + 2/x(n)), com x(0) = 1, a qual converge pra raiz(2): um
irracional). A princípio, bastaria ele incluir o axioma de completeza dos
reais e provar que este axioma não contradiz os demais axiomas. Mas ele foi
mais longe e acabou inventando uma construção dos reais a partir dos
racionais (via os chamados "cortes de Dedekind"). Talvez (estou
conjecturando aqui) ele só tenha feito isso por uma necessidade psicológica
de provar que algo é possível (no caso a extensão dos racionais aos reais)
exibindo uma construção explícita. Isso talvez seja um reflexo da ojeriza
que matemáticos da época tinham (e alguns poucos têm até hoje) de
argumentos não construtivos pra provar a existência de fatos ou objetos
matemáticos. Por exemplo, demonstrações usando o princípio das casas de
pombos são desse tipo.

Sobre "entender sem compreender", lembrei de um ditado do John von
Neumann: "Em matemática, você nunca entende as coisas. Apenas se acostuma
com elas."

[]s,
Claudio.

On Wed, Nov 16, 2022 at 6:52 PM Claudio Buffara 
wrote:

> Não entendi como uma homotetia poderia reduzir um par ordenado a um único
> número... enfim...
>
> O que se faz, no caso da relação de equivalência que descrevi, é
> representar o par (a,b) pela notação a-b.
> Daí, (a,b) e (c,d) são equivalentes sss a-b = c-d.
> E a novidade são os números negativos: as classes de equivalência de pares
> (a,b) com a < b, representadas, por exemplo, pelo par (0,c), onde c = b-a.
> Ou, na notação usual, -c.
>
> Mas não acho que se deva perder muito tempo com a construção de sistemas
> numéricos via classes de equivalência, estendendo naturais para inteiros
> para racionais para reais e para complexos.  Até porque é extremamente
> sacal, a cada etapa, checar que as operações usuais (+ e *), quando
> aplicadas aos novos números, têm todas as propriedades que conhecemos da
> escola.
> Essas construções foram a maneira que os matemáticos acharam pra
> formalizar os sistemas numéricos, a partir de conceitos mais básicos (no
> caso, pares ordenados e relações de equivalência) - é o programa do Hilbert
> (ou de Russell e Whitehead), de reduzir toda a matemática à teoria dos
> conjuntos.
> Mas, no fundo, esta é uma construção artificial, ex post.  Pois
> matemáticos já usavam todos os números muito antes dessa formalização ser
> inventada.
> E não acho que ela renda muitos frutos, nem pedagógicos (a menos que seu
> objetivo seja "entender sem compreender") e nem pra ampliação da fronteira
> do conhecimento, exceto colocar os sistemas numéricos numa base axiomática
> sólida.
> Em particular, no que diz respeito aos números reais, a única coisa que
> interessa é que eles são um corpo ordenado completo. Tanto é que vários
> livros de análise partem deste axioma e não se preocupam em construir os
> reais a partir dos naturais.
>
> []s,
> Claudio.
>
>
>
> On Tue, Nov 15, 2022 at 5:07 PM Pedro José  wrote:
>
>> Obrigado a você e ao Cláudio. Mas não sou criativo para inventar. Mas já
>> vi que terei que fazer uma homotetia, para as classes de equivalência para
>> representar só como um número e não como um par, creio eu.
>>
>> Cordialmente,
>> PJMS
>>
>> Em ter., 15 de nov. de 2022 às 16:00, Anderson Torres <
>> torres.anderson...@gmail.com> escreveu:
>>
>>>
>>>
>>> Em ter, 15 de nov de 2022 14:33, Pedro José 
>>> escreveu:
>>>
>>>> Boa tarde!
>>>> Para os |Naturais, temos os postulados de Peano.
>>>>
>>>> Para os Inteiros há alguma formalização?
>>>>
>>>
>>> invente uma!
>>>
>>> Pode ser por exemplo o conjunto de pares (p,q) tais que p-q é constante.
>>>
>>> ou melhor (p1,q1)=(p2,q2) se e só se p1+q2=p2+q1.
>>>
>>>
>>>> Acho pobre dizer que é necessário ter outros números devido a

[obm-l] Re: [obm-l] Re: [obm-l] Re: [obm-l] Caracterização de Inteiros

2022-11-16 Por tôpico Claudio Buffara
Não entendi como uma homotetia poderia reduzir um par ordenado a um único
número... enfim...

O que se faz, no caso da relação de equivalência que descrevi, é
representar o par (a,b) pela notação a-b.
Daí, (a,b) e (c,d) são equivalentes sss a-b = c-d.
E a novidade são os números negativos: as classes de equivalência de pares
(a,b) com a < b, representadas, por exemplo, pelo par (0,c), onde c = b-a.
Ou, na notação usual, -c.

Mas não acho que se deva perder muito tempo com a construção de sistemas
numéricos via classes de equivalência, estendendo naturais para inteiros
para racionais para reais e para complexos.  Até porque é extremamente
sacal, a cada etapa, checar que as operações usuais (+ e *), quando
aplicadas aos novos números, têm todas as propriedades que conhecemos da
escola.
Essas construções foram a maneira que os matemáticos acharam pra formalizar
os sistemas numéricos, a partir de conceitos mais básicos (no caso, pares
ordenados e relações de equivalência) - é o programa do Hilbert (ou de
Russell e Whitehead), de reduzir toda a matemática à teoria dos conjuntos.
Mas, no fundo, esta é uma construção artificial, ex post.  Pois matemáticos
já usavam todos os números muito antes dessa formalização ser inventada.
E não acho que ela renda muitos frutos, nem pedagógicos (a menos que seu
objetivo seja "entender sem compreender") e nem pra ampliação da fronteira
do conhecimento, exceto colocar os sistemas numéricos numa base axiomática
sólida.
Em particular, no que diz respeito aos números reais, a única coisa que
interessa é que eles são um corpo ordenado completo. Tanto é que vários
livros de análise partem deste axioma e não se preocupam em construir os
reais a partir dos naturais.

[]s,
Claudio.



On Tue, Nov 15, 2022 at 5:07 PM Pedro José  wrote:

> Obrigado a você e ao Cláudio. Mas não sou criativo para inventar. Mas já
> vi que terei que fazer uma homotetia, para as classes de equivalência para
> representar só como um número e não como um par, creio eu.
>
> Cordialmente,
> PJMS
>
> Em ter., 15 de nov. de 2022 às 16:00, Anderson Torres <
> torres.anderson...@gmail.com> escreveu:
>
>>
>>
>> Em ter, 15 de nov de 2022 14:33, Pedro José 
>> escreveu:
>>
>>> Boa tarde!
>>> Para os |Naturais, temos os postulados de Peano.
>>>
>>> Para os Inteiros há alguma formalização?
>>>
>>
>> invente uma!
>>
>> Pode ser por exemplo o conjunto de pares (p,q) tais que p-q é constante.
>>
>> ou melhor (p1,q1)=(p2,q2) se e só se p1+q2=p2+q1.
>>
>>
>>> Acho pobre dizer que é necessário ter outros números devido ao problema
>>> de fechamento nos naturais para a subtração que é fato e daí introduzir os
>>> simétricos que são inteiros e ainda não foram caracterizados.
>>>
>>> No meu antigo ginásio aprendi que os Reais era a união dos conjuntos
>>> disjuntos irracionais e racionais. Os racionais haviam sido bem definidos.
>>> Aí questionei e o que são irracionais? resposta: são os Reais que não são
>>> racionais, os que não podem ser escritos na forma p/q p e q inteiros e
>>> q<>0. Mas me deram um tombo. Definiram os |Reais com base nos irracionais e
>>> os irracionais com base nos |Reais. 3 +2i também não pode ser inscrito na
>>> forma p/q. Só mais tarde no científico, é que meu professor definiu
>>> irracional como um número que não podia ser escrito na forma p/q e cuja
>>> representação decimal tinha uma infinidade de algarismos, sem haver uma
>>> periodicidade.
>>> Na época foi o maior nó que tive com a matemática. O mestre demonstrou
>>> que os racionais eram densos, mas entre eles ainda cabiam os irracionais.
>>> Não satisfeito mostrou que os racionais eram enumeráveis e por absurdo
>>> mostrou que os |Reais não. Não satisfeito mostrou que a cardinalidade do
>>> intervalo [0,1] era maior que a dos |Naturais. Não conseguia conceber que
>>> havia um infinito maior que outro. Outra coisa que demorei a aceitar,mesmo
>>> vendo a bijeção, era que os inteiros e naturais tinham a mesma
>>> cardinalidade. Na minha cabeça, os inteiros têm todos os naturais ainda
>>> sobram os negativos, como é igual?
>>> Hoje, depois de velho, arrumei uma enteada, que muito me pergunta e
>>> estou enrolado. Para dar um ar de superioridade, questionei se conhecia os
>>> inteiros de Gaus, que 5 não era primo nos inteiros de Gaus. Estrepei-me, a
>>> danada foi pesquisar e me questiona sobre o que não tenho um domínio pleno.
>>> Em suma, como apresentei a ela os postulados de Peano para a
>>> caracterização dos Naturais, ela me cobra por algo semelhante para os
>>> Inteiros, e não sei responder.
>>> HELP! SOCORRO! AU SECOURS! AYUDA! AIUTO! HILFE!
>>> Cordialmente,
>>> PJMS
>>>
>>> --
>>> Esta mensagem foi verificada pelo sistema de antivírus e
>>> acredita-se estar livre de perigo.
>>
>>
>> --
>> Esta mensagem foi verificada pelo sistema de antivírus e
>> acredita-se estar livre de perigo.
>
>
> --
> Esta mensagem foi verificada pelo sistema de antivírus e
> acredita-se estar livre de perigo.

-- 
Esta mensagem foi verificada pelo sistema de 

Re: [obm-l] Caracterização de Inteiros

2022-11-15 Por tôpico Claudio Buffara
A única que conheço e’ a que define uma relação de equivalência em pares 
ordenados de naturais (união {0}) dada por (a,b) ~ (c,d)  <==> a+d = b+c. Os 
inteiros são as classes de equivalência desta relação.

Enviado do meu iPhone

> Em 15 de nov. de 2022, à(s) 14:33, Pedro José  escreveu:
> 
> 
> Boa tarde!
> Para os |Naturais, temos os postulados de Peano.
> 
> Para os Inteiros há alguma formalização?
> 
> Acho pobre dizer que é necessário ter outros números devido ao problema de 
> fechamento nos naturais para a subtração que é fato e daí introduzir os 
> simétricos que são inteiros e ainda não foram caracterizados. 
> 
> No meu antigo ginásio aprendi que os Reais era a união dos conjuntos 
> disjuntos irracionais e racionais. Os racionais haviam sido bem definidos. 
> Aí questionei e o que são irracionais? resposta: são os Reais que não 
> são racionais, os que não podem ser escritos na forma p/q p e q inteiros e 
> q<>0. Mas me deram um tombo. Definiram os |Reais com base nos irracionais e 
> os irracionais com base nos |Reais. 3 +2i também não pode ser inscrito na 
> forma p/q. Só mais tarde no científico, é que meu professor definiu 
> irracional como um número que não podia ser escrito na forma p/q e cuja 
> representação decimal tinha uma infinidade de algarismos, sem haver uma 
> periodicidade.
> Na época foi o maior nó que tive com a matemática. O mestre demonstrou que 
> os racionais eram densos, mas entre eles ainda cabiam os irracionais. Não 
> satisfeito mostrou que os racionais eram enumeráveis e por absurdo mostrou 
> que os |Reais não. Não satisfeito mostrou que a cardinalidade do intervalo 
> [0,1] era maior que a dos |Naturais. Não conseguia conceber que havia um 
> infinito maior que outro. Outra coisa que demorei a aceitar,mesmo vendo a 
> bijeção, era que os inteiros e naturais tinham a mesma cardinalidade. Na 
> minha cabeça, os inteiros têm todos os naturais ainda sobram os negativos, 
> como é igual?
> Hoje, depois de velho, arrumei uma enteada, que muito me pergunta e estou 
> enrolado. Para dar um ar de superioridade, questionei se conhecia os inteiros 
> de Gaus, que 5 não era primo nos inteiros de Gaus. Estrepei-me, a danada foi 
> pesquisar e me questiona sobre o que não tenho um domínio pleno.
> Em suma, como apresentei a ela os postulados de Peano para a caracterização 
> dos Naturais, ela me cobra por algo semelhante para os Inteiros, e não sei 
> responder.
> HELP! SOCORRO! AU SECOURS! AYUDA! AIUTO! HILFE!
> Cordialmente,
> PJMS
> 
> -- 
> Esta mensagem foi verificada pelo sistema de antivírus e 
> acredita-se estar livre de perigo.

-- 
Esta mensagem foi verificada pelo sistema de antiv�rus e
 acredita-se estar livre de perigo.


=
Instru��es para entrar na lista, sair da lista e usar a lista em
http://www.mat.puc-rio.br/~obmlistas/obm-l.html
=


Re: [obm-l] problema de probabilidade

2022-11-09 Por tôpico Claudio Buffara
Essa também:
https://thedailyviz.com/2016/09/17/how-common-is-your-birthday-dailyviz/


On Wed, Nov 9, 2022 at 12:04 PM Claudio Buffara 
wrote:

> Achei isso aqui interessante: https://www.panix.com/~murphy/bday.html
>
> []s,
> Claudio.
>
> On Tue, Nov 8, 2022 at 9:56 PM Ralph Costa Teixeira 
> wrote:
>
>> Mis ou menos... O que faltou foi a hipótese exata da distribuição de
>> probabilidade dos aniversários.
>>
>> Se a gente supõe que cada mês tem os mesmos 1/12 de chance para cada
>> aluno, e que os meses são independentes entre si, sim,
>> p=12/12^2=1/12~8.3%.
>>
>> Agora, talvez um modelo um pouco mais preciso seria supor que cada DIA do
>> ano tem a mesma probabilidade (e que são independentes entre si). Isto
>> afeta um tiquinho a resposta, porque cada mes têm um número ligeiramente
>> diferente de dias! Ignorando anos bissextos (huh!?!), temos:
>> -- 7 meses com 31 dias;
>> -- 4 meses com 30 dias;
>> -- 1 mes com 28 dias;
>> Portanto, seria um pouco mais "realista" usar:
>> p=(7*31^2+4*30^2+28^2)/(365^2) ~ 8.34003%
>>
>> Eu ponho esse "realista" bem entre aspas; primeiro, porque eu ignorei
>> anos bissextos (fique à vontade para inclui-los e refazer a conta :D :D
>> :D); mas a hipótese de que todos os dias do ano tem a mesma probabilidade
>> não é tão realista quanto parece! Existe uma certa "concentração" de
>> aniversários em determinadas épocas do ano... mas, sem dados exatos sobre
>> como seja a tal concentração, o melhor que podemos fazer seria uma das
>> estimativas acima.
>>
>> Ainda tem um segundo problema sutil: *mesmo que todos os dias tivessem a
>> mesma probabilidade, talvez n*ã*o seja 100% correto supor que os
>> aniversários dos alunos da mesma turma do CMBel sejam independentes*!
>> Por exemplo, existe uma probabilidade maior que zero de ter gêmeos numa
>> mesma turma (comum uma família com gêmeos colocá-los na mesma escola), o
>> que afeta a independência dos dados, e muda um pouquinho aqueles 8.3% (para
>> cima)... sem uma estimativa desta probabilidade de ter gêmeos na mesma
>> turma, não conseguimos calcular a resposta "exata".
>>
>> Isto tudo dito... em quase qualquer problema de probabilidade a gente vai
>> ter que fazer ALGUMA hipótese simplificadora para poder sair do lugar.
>> Assim, eu diria que o problema não está 100% bem posto, mas não acho
>> ridículo fazer uma das hipóteses simplificadoras acima que levam a 8.3%
>> ou 8.34003% (e a diferença me parece tão pequena que eu aceitaria ambas as
>> respostas como corretas, desde que as hipóteses utilizadas em cada caso
>> fossem citadas).
>>
>> Abraço, Ralph.
>>
>> On Tue, Nov 8, 2022 at 3:07 PM Luis Paulo  wrote:
>>
>>> Prezados, o problema abaixo está bem posto?
>>>
>>> Uma turma do CMBel tem 25 alunos. Escolhendo-se aleatoriamente dois
>>> estudantes dessa turma, qual a probabilidade de eles façam aniversário no
>>> mesmo mês?
>>>
>>> A resposta da banca: 1/12.
>>>
>>>
>>>
>>> --
>>> Esta mensagem foi verificada pelo sistema de antivírus e
>>> acredita-se estar livre de perigo.
>>
>>
>> --
>> Esta mensagem foi verificada pelo sistema de antivírus e
>> acredita-se estar livre de perigo.
>
>

-- 
Esta mensagem foi verificada pelo sistema de antiv�rus e
 acredita-se estar livre de perigo.



Re: [obm-l] problema de probabilidade

2022-11-09 Por tôpico Claudio Buffara
Achei isso aqui interessante: https://www.panix.com/~murphy/bday.html

[]s,
Claudio.

On Tue, Nov 8, 2022 at 9:56 PM Ralph Costa Teixeira 
wrote:

> Mis ou menos... O que faltou foi a hipótese exata da distribuição de
> probabilidade dos aniversários.
>
> Se a gente supõe que cada mês tem os mesmos 1/12 de chance para cada
> aluno, e que os meses são independentes entre si, sim,
> p=12/12^2=1/12~8.3%.
>
> Agora, talvez um modelo um pouco mais preciso seria supor que cada DIA do
> ano tem a mesma probabilidade (e que são independentes entre si). Isto
> afeta um tiquinho a resposta, porque cada mes têm um número ligeiramente
> diferente de dias! Ignorando anos bissextos (huh!?!), temos:
> -- 7 meses com 31 dias;
> -- 4 meses com 30 dias;
> -- 1 mes com 28 dias;
> Portanto, seria um pouco mais "realista" usar:
> p=(7*31^2+4*30^2+28^2)/(365^2) ~ 8.34003%
>
> Eu ponho esse "realista" bem entre aspas; primeiro, porque eu ignorei
> anos bissextos (fique à vontade para inclui-los e refazer a conta :D :D
> :D); mas a hipótese de que todos os dias do ano tem a mesma probabilidade
> não é tão realista quanto parece! Existe uma certa "concentração" de
> aniversários em determinadas épocas do ano... mas, sem dados exatos sobre
> como seja a tal concentração, o melhor que podemos fazer seria uma das
> estimativas acima.
>
> Ainda tem um segundo problema sutil: *mesmo que todos os dias tivessem a
> mesma probabilidade, talvez n*ã*o seja 100% correto supor que os
> aniversários dos alunos da mesma turma do CMBel sejam independentes*! Por
> exemplo, existe uma probabilidade maior que zero de ter gêmeos numa mesma
> turma (comum uma família com gêmeos colocá-los na mesma escola), o que
> afeta a independência dos dados, e muda um pouquinho aqueles 8.3% (para
> cima)... sem uma estimativa desta probabilidade de ter gêmeos na mesma
> turma, não conseguimos calcular a resposta "exata".
>
> Isto tudo dito... em quase qualquer problema de probabilidade a gente vai
> ter que fazer ALGUMA hipótese simplificadora para poder sair do lugar.
> Assim, eu diria que o problema não está 100% bem posto, mas não acho
> ridículo fazer uma das hipóteses simplificadoras acima que levam a 8.3%
> ou 8.34003% (e a diferença me parece tão pequena que eu aceitaria ambas as
> respostas como corretas, desde que as hipóteses utilizadas em cada caso
> fossem citadas).
>
> Abraço, Ralph.
>
> On Tue, Nov 8, 2022 at 3:07 PM Luis Paulo  wrote:
>
>> Prezados, o problema abaixo está bem posto?
>>
>> Uma turma do CMBel tem 25 alunos. Escolhendo-se aleatoriamente dois
>> estudantes dessa turma, qual a probabilidade de eles façam aniversário no
>> mesmo mês?
>>
>> A resposta da banca: 1/12.
>>
>>
>>
>> --
>> Esta mensagem foi verificada pelo sistema de antivírus e
>> acredita-se estar livre de perigo.
>
>
> --
> Esta mensagem foi verificada pelo sistema de antivírus e
> acredita-se estar livre de perigo.

-- 
Esta mensagem foi verificada pelo sistema de antiv�rus e
 acredita-se estar livre de perigo.



Re: [obm-l] Provar que a inteira f é um polinômio de grau positivo

2022-07-14 Por tôpico Claudio Buffara
Use o fato de que toda função meromorfica  em C união {inf} é da forma 
f(z)/g(z), onde f, g são polinômios.
Daí, como a função do enunciado é inteira, g(z) é constante (e não nula).
E como f(z) rende a inf quando z tende a inf, f é um polinômio não constante.

Enviado do meu iPhone

> Em 14 de jul. de 2022, à(s) 16:41, Artur Costa Steiner 
>  escreveu:
> 
> Oi amigos!
> 
> Um teorema da Análise Complexa diz que, se f é inteira e lim z —> oo f(z) 
> = oo, então f é um polinômio (claramente não constante). Nos livros em 
> que estudei isso era dado como exercício, de modo que nunca vi a 
> demonstração deste teorema. Eu consegui dar duas demonstrações para ele, 
> sendo que uma delas sei que está certa A outra acho que também está certa, 
> mas a primeira me parece bem melhor. 
> 
> Alguém aqui pode dar uma prova, para comparar com a minha? Se houver 
> interesse (Análise Complexa não costuma aparecer aqui) eu dou as minhas. 
> 
> Obrigado
> 
> Artur
> 
> 
> 
> -- 
> Esta mensagem foi verificada pelo sistema de antivírus e 
> acredita-se estar livre de perigo.

-- 
Esta mensagem foi verificada pelo sistema de antiv�rus e
 acredita-se estar livre de perigo.


=
Instru��es para entrar na lista, sair da lista e usar a lista em
http://www.mat.puc-rio.br/~obmlistas/obm-l.html
=


Re: [obm-l] Ajuda em Repunits

2022-07-10 Por tôpico Claudio Buffara
Se quiser se divertir mais com isso, veja o seguinte:
1/7 = 0,142857142857142...
O período é 142 857 e 1+8 = 4+5 = 2+7 = 9.

1/11: o período é 09 e 0+9 = 9.

1/13: o período é 076 923 e 0+9 = 7+2 = 6+3 = 9.

Determine, com demonstração, para quais números N, o período de 1/N tem
esta propriedade.




On Sun, Jul 10, 2022 at 8:41 AM Rubens Vilhena Fonseca <
rubens.vilhen...@gmail.com> wrote:

> Muito obrigado ao Ralph Costa Teixeira e ao Claudio Buffara por todos os
> ótimos esclarecimentos.
> [[ ]]'s
>
> Em dom., 10 de jul. de 2022 às 01:39, Ralph Costa Teixeira <
> ralp...@gmail.com> escreveu:
>
>> Argh, corrigindo um detalhe ali perto do fim:
>> -- Sabemos que 10^q*B-B=r/10^w, portanto 9*(111...)**x**10^w = r*n.
>> Novamente, como n é primo com 2, 3 e 5 *e x*, conclui-se que n divide
>> 111 (com q 1's), e portanto q>=p=k.
>>
>> On Sun, Jul 10, 2022 at 1:24 AM Ralph Costa Teixeira 
>> wrote:
>>
>>> A chave: *os "restos parciais" que aparecem são exatamente os restos
>>> que x, 10x, 100x,  deixam na divisão por n.*
>>> ---///---
>>>
>>> MAIS SPOILERS ABAIXO
>>>
>>>
>>> ...
>>>
>>>
>>> 
>>>
>>>
>>> ...
>>>
>>>
>>> 
>>>
>>> Acho que facilita bastante pensar no "período" de 1/n de outro jeito:
>>> ---///---
>>> LEMA:
>>> (i) Dado n não divisível por 2 ou 5, existe algum número da forma
>>> 111...111 que é múltiplo de n.
>>> (ii) Se n não for divisível por 2, 3 ou 5, o *menor* número do tipo
>>> 111...111 que é múltiplo de n tem k dígitos, onde k é exatamente o tamanho
>>> do período (fundamental) da dízima em 1/n.
>>> PROVA:
>>>
>>> (i) Olhe os restos de 1, 11, 111, , ... na divisão por n. São n
>>> possibilidades, de 0 a n-1, então alguma hora algum resto tem que repetir.
>>> Isto significa que .. (com A dígitos) e 11...111 (com B dígitos,
>>> B>> ...1110 (A 1's e B 0's) = 111 * (10^B) é múltiplo de n. Mas
>>> n não tem fator comum com aquele 10^B (pois não é divisível por 2 nem por
>>> 5), portanto ...111 (com k=A-B dígitos) é divisível por n.
>>>
>>> (ii) Denote por P=111111 (com p dígitos) o menor daqueles caras com
>>> apenas "1s" que é múltiplo de n, e denote por k o "período fundamental" na
>>> dízima de 1/n.
>>> Por um lado, como 9P=999=10^p-1 é múltiplo de n, temos 10^p *
>>> (1/n) - 1/n inteiro. Mas isso significa que a parte decimal de 1/n "se
>>> repete" de p em p dígitos, ou seja, que a dízima de 1/n tem período p. Em
>>> particular, p>=k.
>>> Por outro lado, sendo k o período fundamental, temos 10^k * (1/n) -
>>> (1/n) com número finito de casas decimais, ou seja, (10^k-1)/n = m/10^z com
>>> m inteiro, e z=número de casas decimais que "sobraram". Mas daqui vem
>>> 9*(111...111)*10^z = m*n (com k dígitos 1s). Como n é primo com 2, 3 e 5,
>>> conclui-se que 111... (k 1's) tem que ser múltiplo de n, e portanto
>>> k>=p.
>>>
>>> Note um efeito colateral disso tudo: provamos que 10^k*(1/n)- 1/n =
>>> 10^p*(1/n)-1/n = inteiro. Assim aquele z vale 0, ou seja, não tem "casas
>>> decimais que sobram" -- a dízima periódica do 1/n se inicia logo no
>>> primeiro dígito!
>>>
>>> ---///---
>>> Agora fica tudo bem simples:
>>> a) Na notação acima, provamos que k=p, e n divide 111 com p
>>> dígitos.
>>> b) Seja q o período (fundamental) da dízima de B=x/n irredutível.
>>>
>>> Em primeiro lugar, provemos que q=k. Basicamente repetimos o que fizemos
>>> no lema:
>>> -- Sabemos que 10^q*B-B=r/10^w, portanto 9*(111...)*10^w = r*n.
>>> Novamente, como n é primo com 2, 3 e 5, conclui-se que n divide 111
>>> (com q 1's), e portanto q>=p=k.
>>> -- Por outro lado, como (10^k-1)/n é inteiro, (10^k-1)*x/n=10^k*B-B
>>> também é inteiro, ou seja, a dízima de B tem período k (e se inicia no
>>> primeiro dígito!). Portanto k>=q.
>>>
>>> *Enfim, note que os tais "restos parciais" que aparecem são exatamente
>>> os restos que x, 10x, 100x, , 10^q.x deixam na divisão por n. *A
>>> soma desses caras vale (...)*x, que é divisível por n pois temos
>>> ali q=k=p dígitos 1. Por isso, ao dividir esses restos parciais por n, a
>>> soma dos novos restos tem que ser múltiplo de n tambem.
>>>
&

Re: [obm-l] Ajuda em Repunits

2022-07-09 Por tôpico Claudio Buffara
Se n não é divisível por 2 e nem por 5, então 1/n = 0,a1a2...ak a1a2...ak
a1...  (dízima periódica simples de período k)
Daí (10^k)*n - n = a1a2...ak ==> (99...9)*n é inteiro (onde há k algarismos
9) ==> n é fator de 99...9 = 9*(11...1).
Mas n é primo com 3 ==> n | 11...1

Pra segunda parte, a ideia é tentar ver porque é verdade com exemplos
concretos.
Por exemplo, 1/7:
10*1 = 1*7 + 3
10*3 = 4*7 + 2
10*2 = 2*7 + 6
10*6 = 8*7 + 4
10*4 = 5*7 + 5
10*5 = 7*7 + 1
10*1 = 1*7 + 3  (e as equações se repetem a partir daqui)

1/13:
10*1 = 0*13 + 10
10*10 = 7*13 + 9
10*9 = 6*13 + 12
10*12 = 9*13 + 3
10*3 = 2*13 + 4
10*4 = 3*13 + 1
10*1 = 0*13 + 10 (idem)

Assim, no caso geral, pra calcular a representação de 1/n, as k primeiras
divisões sucessivas resultam em:
10*1 = a1*n + r1
10*r1 = a2*n + r2
10*r2 = a3*n + r3
...
10*r(k-1) = ak*n + rk

Como n é primo com 2 e 5, 1/n será uma dízima periódica simples, digamos de
período k.
Isso significa que rk, o resto da k-ésima divisão, será necessariamente
igual a 1, já que os dividendos (os algarismos aj que formam o período)
irão se repetir a partir da (k+1)-ésima equação.
Ou seja, a(k+1) = a1 e, portanto, r(k+1) = r1.

Somando as k equações, obtemos:
10*(1+r1+r2+ ...r(k-1)) = (a1+a2+a3...+ak)*n + (r1+r2+r3+...+rk).
Como rk = 1, isso fica:
10*(rk+r1+r2+ ...r(k-1)) = (a1+a2+a3...+ak)*n + (r1+r2+r3+...+rk) ==>
9*(rk+r2+...+r(k-1)) = (a1+a2+a3+...+ak)*n
Como n é primo com 3 (e, portanto, com 9), concluímos que n divide
r1+r2+...+rk.




On Sat, Jul 9, 2022 at 7:16 PM Rubens Vilhena Fonseca <
rubens.vilhen...@gmail.com> wrote:

> Gostaria de uma demonstração para o seguinte teorema.
> *Teorema*. Seja n um inteiro positivo não divisível por 2, 3 ou 5, e
> suponha que a expansão decimal de l/n tenha período k. Então n é um fator
> do inteiro 111 ... 11 (k 1 's). Além disso, a soma dos restos parciais na
> divisão obtida de cada fração irredutível x/n é um múltiplo de n.
> Comentário:
> Pelo que entendi, se 1/13  tem período k =6. Então 13  divide 11 ( k=6
> 1's).
> Essa parte consegui provar.
> Quanto à segunda parte  para 1/13 os resto da divisão sem repetição são
> {10, 9, 12, 3, 4, 1}. Então 10+9+12+3+4+1= 13q . (Não soube provar)
> Não consigo organizar uma sequência  de passos para a demonstração
> dos dois fatos.
> Agradeço qualquer ajuda.
> [[ ]]'s
>
> --
> Esta mensagem foi verificada pelo sistema de antivírus e
> acredita-se estar livre de perigo.

-- 
Esta mensagem foi verificada pelo sistema de antiv�rus e
 acredita-se estar livre de perigo.



[obm-l] Re: [obm-l] Dúvida e ajuda.

2022-04-08 Por tôpico Claudio Buffara
A volta é fácil também: ao calcular a representação decimal de a/b (a e b
naturais), nas divisões sucessivas por b só existem b-1 restos possíveis
(resto = 0 em alguma etapa implica numa decimal finita) e, portanto, após
não mais do que b-1 divisões, um resto vai se repetir, marcando o início de
um novo período na representação decimal.

Agora, suponha que  X =
0,123456789112233445566778899111222333444555666777888999... seja racional.
Então existirão n e p naturais tais que, a partir da n-ésima casa decimal
(1/10^n), os algarismos de X vão se repetir numa sequência com período p.

Mas, pela lei de formação de X, vai existir uma sequência de n+p+1
algarismos iguais a 1, e esta sequência vai começar após a n-ésima casa
decimal.
Ou seja, a sequência vai estar incluída na parte periódica da representação
decimal de X.
Mas como o período é p, isso implica que a parte periódica teria que
ser 111..11 (p algarismos 1) ==> contradição à lei de formação de X.

[]s,
Claudio.


On Fri, Apr 8, 2022 at 11:17 AM Pedro José  wrote:

> Bom dia!
> Posso concluir que um número representado por uma infinidade de algarismos
> decimais é racional se e somente se tem um período de repetições desses
> algarismos?
> A ida é fácil se tiver o período é racional.
> Já a volta não sei se é verdade e se for há como provar?
>
> Meu objetivo primário é saber se:
> 0,123456789112233445566778899111222333444555666777888999... é racional. As
> reticências se referem ao aumento de mais um algarismo repetido a cada
> sequência, ou seja a primeira aparição de 1 será 1, a 2a 11 a 3a 111 e
> assim sucessivamente, o mesmo vale para os demais algarismos.
>
> Alguém poderia me ajudar?
> Grato,
> PJMS
>
> --
> Esta mensagem foi verificada pelo sistema de antivírus e
> acredita-se estar livre de perigo.

-- 
Esta mensagem foi verificada pelo sistema de antiv�rus e
 acredita-se estar livre de perigo.



Re: [obm-l] Quebra do RSA por solução do problema de fatoração - Eric Campos Bastos Guedes

2022-01-11 Por tôpico Claudio Buffara
Eu diria que a melhor forma de avaliar seu trabalho é testando.
Apesar do “desafio RSA” já ter encerrado, os números ainda estão disponíveis.
Da uma olhada no verbete “RSA numbers” na Wikipédia.

Enviado do meu iPhone

> Em 11 de jan. de 2022, à(s) 15:03, Eric Campos Bastos Guedes 
>  escreveu:
> 
> 
> Proponho um algoritmo para quebrar o RSA. O algoritmo que eu propus antes 
> trabalhava com números muito grandes e por isso podia não funcionar 
> direito. Esse trabalha com números bem menores porque usa módulo N numa 
> etapa. O algoritmo e sua explicação estão no YouTube com o mesmo título 
> desse e-mail. São dois vídeos, o que conta é o mais recente deste ano de 
> 2022.
> 
> QUEBRA DO RSA - ALGORITMO N.2
> 
> PASSO 1: a=3
> 
> inicializando o valor de a
> 
> PASSO 2: N é o inteiro a ser fatorado 
> 
> N é o número usado no RSA. N é o produto de dois números primos grandes 
> não muito próximos. 
> 
> PASSO 3: M=N^512 (N elevado a 512)
> 
> M é um número grande mas não muito grande. O valor de P não vai 
> ultrapassar muito o valor de M. P é uma variável inteira que acumula 
> fatores primos. Aí você faz MDC(P, N) para tentar fatorar N.
> 
> PASSO 4: a=a+1
> 
> O valor de a é atualizado para a+1, isto é,  é  acrescentado 1 ao valor 
> de a
> 
> PASSO 5: P=a 
> 
> O valor de P é inicializado 
> 
> PASSO 6: b = número aleatório entre 0 e 1
> PASSO 7: Se b > 1/2 faça c=1 senão faça c=-1
> 
> O objetivo dos passos 6 e 7 é atribuir à variável c um valor que pode ser 
> 1 ou -1. Isso nem precisa ser feito de modo aleatório, mas acho que vai 
> funcionar melhor se for aleatório. 
> 
> PASSO 8: P=P(P+c)
> 
> É uma atribuição de valor. O novo valor de P passa a ser P(P+c). Note que 
> P+c é relativamente primo com P. Na prática são acrescentados novos 
> fatores primos a P que vai acumular fatores primos. 
> 
> PASSO 9: Se P < M vá para o PASSO 6
> 
> Esse passo determina um looping para acumular fatores em P.
> 
> PASSO 10: Se MDC(P, N) for diferente de 1 vá para o PASSO 14
> 
> Se MDC(P, N) for diferente de 1 ele pode ser um fator primo de N. Resta 
> verificar se ele não é o próprio N. Isso vai ser feito no PASSO 14.
> 
> PASSO 11: P = Resto da divisão de P por N
> 
> Esse passo é para trabalharmos com números menores. 
> 
> PASSO 12: Se P < 4 faça P=4
> 
> Talvez esse passo possa ser omitido
> 
> PASSO 13: vá para o PASSO 6
> 
> PASSO 14: Se MDC(P, N)=N vá para o PASSO 4
> 
> Se MDC(P, N) = N não foram encontrados fatores primos e algoritmo recomeça 
> do ponto apropriado. 
> 
> PASSO 15: MDC(P, N) é fator (primo) de N
> 
> FIM
> 
> Eu fui menção honrosa na Olimpíada Ibero-americana de Matemática 
> Universitária em 2006. Acho que este meu trabalho merece ser avaliado.
> 
> 
> 
> 
> -- 
> Esta mensagem foi verificada pelo sistema de antivírus e 
> acredita-se estar livre de perigo.

-- 
Esta mensagem foi verificada pelo sistema de antiv�rus e
 acredita-se estar livre de perigo.


=
Instru��es para entrar na lista, sair da lista e usar a lista em
http://www.mat.puc-rio.br/~obmlistas/obm-l.html
=


Re: [obm-l] CARA ou COROA com dez moedas

2021-12-22 Por tôpico Claudio Buffara
Uma pergunta mais interessante é: Qual o número esperado de lançamentos da
moeda até que um deles vença?

On Wed, Dec 22, 2021 at 12:00 PM jamil dasilva 
wrote:

> Duas pessoas disputam um CARA e COROA, jogando uma moeda honesta *CEM* 
> VEZES.Um
> deles aposta  que em todos os lançamentos ocorrerá  CARA e o outro, por sua
> vez, aposta que ocorrerá CARA *apenas*  nos *primeiros
> cinquenta lançamentos* e, consequentemente, cinquenta coroas nos
> cinquenta últimos.Qual deles tem maior probabilidade de ganhar nesse CARA e
> COROA ?
>
>
>
>
> --
> Esta mensagem foi verificada pelo sistema de antivírus e
> acredita-se estar livre de perigo.

-- 
Esta mensagem foi verificada pelo sistema de antiv�rus e
 acredita-se estar livre de perigo.



[obm-l] Número de matrizes 0-1

2021-12-20 Por tôpico Claudio Buffara
Num outro grupo, propuseram o problema de achar o número de matrizes 4x4
com entradas em {0,1} e cujo determinante seja ímpar.
Olhando mod 2, isso é equivalente a achar o número de matrizes 4x4
invertíveis com entradas em Z2 (o corpo com 2 elementos).
Este é um resultado conhecido: o número de tais matrizes é
(2^4-1)(2^4-2)(2^4-2^2)(2^4-2^3) = 15*14*12*8 = 20.160.
Provar isso pode ser um bom problema pra quem não conhece o "truque" (que
nada mais é do que usar uma caracterização alternativa de "matriz
invertível").

Daí surgiram duas dúvidas:
1) Quais os valores possíveis do determinante desta matriz?
2) Quantas matrizes existem com cada valor possível do determinante?

Não é difícil fazer um programa de computador pra calcular isso (afinal,
existem apenas 2^16 = 65.536 matrizes 4x4 com entradas em {0,1}).
Mas será que há uma forma "esperta" de calcular isso?
E que seja generalizável pra matrizes nxn?

[]s,
Claudio.

-- 
Esta mensagem foi verificada pelo sistema de antiv�rus e
 acredita-se estar livre de perigo.



[obm-l] Re: [obm-l] Invertíveis e Divisores de Zero

2021-11-30 Por tôpico Claudio Buffara
Z_4 x Z_5 é isomorfo a Z_20.
Talvez isso ajude.

On Tue, Nov 30, 2021 at 2:33 PM Pedro Júnior 
wrote:

> Quem puder ajudar...
> Encontre todos os invertíveis e divisores de zero em Z_4 x Z_5.
>
>
>
> --
> Esta mensagem foi verificada pelo sistema de antivírus e
> acredita-se estar livre de perigo.

-- 
Esta mensagem foi verificada pelo sistema de antiv�rus e
 acredita-se estar livre de perigo.



Re: [obm-l]

2021-11-14 Por tôpico Claudio Buffara
Por que vc não testa?

On Sun, Nov 14, 2021 at 9:53 AM Eric Campos Bastos Guedes <
ebastosgue...@gmail.com> wrote:

> Eu preciso de um retorno sobre o meu algoritmo que quebra o RSA resolvendo
> o problema de fatoracao.
>
> O Passo 8 talvez possa ser substituido por: PASSO 8': P = P(P+C)
>
> Em seg., 6 de set. de 2021 07:47, Eric Campos Bastos Guedes <
> ebastosgue...@gmail.com> escreveu:
>
>> Aparentemente minha conexão está raqueada por gente do Bolsonaro e eu não
>> estou recebendo respostas para minha postagem e também não estou
>> conseguindo acessar os sites de discussão sobre o RSA. Há pessoas se
>> fazendo passar por mim também.
>>
>> -- Forwarded message -
>> De: Eric Campos Bastos Guedes 
>> Date: sáb., 4 de set. de 2021 00:33
>> Subject: [obm-l]
>> To: 
>>
>>
>> Olá a todos. Gostaria de pedir licença para que vocês avaliem um
>> algoritmo que eu fiz para fatorar números grandes com fatores primos também
>> grandes. Eu acredito que esse algoritmo quebre o RSA
>>
>> O algoritmo é o seguinte:
>>
>> PASSO 1: faça A=3
>> PASSO 2: N é o inteiro a ser fatorado
>> PASSO 3: M = N**16 (potência)
>> PASSO 4: faça A=A+1
>> PASSO 5: faça P=A
>> PASSO 6: faça B=número aleatório entre 0 e 1
>> PASSO 7: se B eh maior que 0.5 faça C=1 senão faça C = -1
>> PASSO 8: faça P=(PP+CP)/2=P(P+C)/2
>> PASSO 9: se P eh menor que M  vá para o PASSO 6
>> PASSO 10: se mdc(P, N) = 1 faça M=MM e vá para o PASSO 4
>> PASSO 11: se mdc(P, N) = N faça M = raiz quadrada de M e vá para o PASSO 4
>> PASSO 12: mdc(P, N) é fator de N
>> FIM
>>
>> AUTOR: ERIC CAMPOS BASTOS GUEDES  (DIA 4 DE SETEMBRO)
>>
>> Creio ter resolvido o problema de fatoração. Alguém pode verificar isso
>> para mim.
>>
>> --
>> Esta mensagem foi verificada pelo sistema de antivírus e
>> acredita-se estar livre de perigo.
>>
>
> --
> Esta mensagem foi verificada pelo sistema de antivírus e
> acredita-se estar livre de perigo.

-- 
Esta mensagem foi verificada pelo sistema de antiv�rus e
 acredita-se estar livre de perigo.



Re: [obm-l] geometria

2021-09-27 Por tôpico Claudio Buffara
O caso LLL de congruência implica que, dados 3 segmentos que obedecem aa 
desigualdade triangular, o triângulo que os tem como lados é unicamente 
determinado, a menos de uma isometria.

Enviado do meu iPhone

> Em 27 de set. de 2021, à(s) 19:50, Israel Meireles Chrisostomo 
>  escreveu:
> 
> 
> 
> Olá pessoal. como faço para provar que o triângulo é um polígono rígido?
> 
> 
> Abraços, muito obrigado
> -- 
> Israel Meireles Chrisostomo
> 
> -- 
> Esta mensagem foi verificada pelo sistema de antivírus e 
> acredita-se estar livre de perigo.

-- 
Esta mensagem foi verificada pelo sistema de antiv�rus e
 acredita-se estar livre de perigo.


=
Instru��es para entrar na lista, sair da lista e usar a lista em
http://www.mat.puc-rio.br/~obmlistas/obm-l.html
=


Re: [obm-l] lei dos senos

2021-09-24 Por tôpico Claudio Buffara
Se os ângulos do triângulo são dados, então o triângulo fica determinado a
menos de uma semelhança.
Daí, dado um lado, os outros ficam unicamente determinados, e
necessariamente obedecem à lei dos senos.

Ou seja, dados a, b, c ângulos de um triângulo, e o lado de medida m,
oposto ao ângulo a, os lados de medidas n e o ficam unicamente
determinados, por:
n = m*sen(b)/sen(a)   e   o = m*sen(c)/sen(a).

On Fri, Sep 24, 2021 at 1:07 AM Israel Meireles Chrisostomo <
israelmchrisost...@gmail.com> wrote:

> Olá pessoal, eu estava resolvendo um problema  daí então surgiu uma
> dúvida.A dúvida é a seguinte: sejam a,b,c ângulos  de um triângulo e m,n,o
> lados de um triângulo qualquer , como provar que se m/sen(a)= n/sen(b)=
> o/sen(c) então a,b,c e m,n,o pertencem ao mesmo triângulo. ou seja vale a
> lei dos senos.
>
> --
> Israel Meireles Chrisostomo
>
> --
> Esta mensagem foi verificada pelo sistema de antivírus e
> acredita-se estar livre de perigo.

-- 
Esta mensagem foi verificada pelo sistema de antiv�rus e
 acredita-se estar livre de perigo.



Re: [obm-l] f(x + y) = f(x) + f(y)

2021-05-05 Por tôpico Claudio Buffara
f(x) = ax + b só satisfaz isso se b = 0.
Tente com x+1, por exemplo.
E mais: sem alguma outra condição (do tipo continuidade ou monotonicidade) 
ainda assim a expressão não implica que f(x) = ax.

Abs,
Cláudio.


Enviado do meu iPhone

> Em 5 de mai. de 2021, à(s) 09:13, joao pedro b menezes 
>  escreveu:
> 
> Eu estava fazendo um exercício de equações funcionais e me deparei com 
> essa expressão. Não sei o que aconteceu, mas tive uma crise existencial e 
> decidi provar que isso implica f(x) = ax + b( ou pelo menos acho que 
> implica). Essa prova estaria certa?:
> (obs: a função é definida nos racionais)
> f(x + 0) = f(x) + f(0) => f(0) = 0.
> f(x + h) = f(x) + f(h) ->
> (f(x + h) - f(x))/h = f(h)/h = (f(h) - f(0))/h
> agora basta fazer lim h -> 0 e obtemos 
> f’(x) = f’(0) . Mas f’(0) é uma constante, logo f(x) = ax + b
> (obs: tenho quase certeza que ela seria válida para os reais, porém como a 
> função é limitada aos racionais, estou em dúvida)
> 
> -- 
> Esta mensagem foi verificada pelo sistema de antivírus e 
> acredita-se estar livre de perigo.

-- 
Esta mensagem foi verificada pelo sistema de antiv�rus e
 acredita-se estar livre de perigo.


=
Instru��es para entrar na lista, sair da lista e usar a lista em
http://www.mat.puc-rio.br/~obmlistas/obm-l.html
=


Re: [obm-l] Re: [obm-l] Sequência Injetiva

2021-02-14 Por tôpico Claudio Buffara
Será que essa sequência é sobrejetiva (sobre os racionais positivos)?
Porque como a(2^n) = n+1, ela certamente atinge todos os naturais, de modo que 
é ilimitada, superiormente e inferiormente (já que a(2^n + 1) = 1/(n+1) ).
Mesmo que não seja, seria interessante descobrir que racionais positivos ela 
não atinge.
É suficiente provar que todos os racionais entre 0 e 1 são atingidos (no caso, 
pelos termos de ordem ímpar), mas não sei se isso facilita.
Vale uma exploração numérica, talvez com uma planilha.

Abs,
Claudio.

Enviado do meu iPhone

> Em 14 de fev. de 2021, à(s) 13:57, Anderson Torres 
>  escreveu:
> 
> 
> 
> 
> Em sáb., 13 de fev. de 2021 às 17:56, Jeferson Almir 
>  escreveu:
>> Amigos, peço ajuda em provar a injetividade dessa sequência que seria uma 
>> saída para provar a unica ocorrência do racional que aparece nela. Estou 
>> andando em círculos tentando montar uma possível indução.
>> 
>> Dado a sequência a_1 = 1 e a_2n = a_n  + 1 e a_2n+1 = 1/a_2n.
>> Prove que para todo racional positivo que ocorre na sequência, ocorre uma 
>> única vez.
>> 
> 
> Acho que e uma boa usar fracao continua aqui.
> 
> Se a_n = [c0; c1, c2, ..., ck], temos entao a_1 = [1] e
> 
> a_2n =Â [(1+c0); c1, c2, ..., ck] (chamemos isso de operacao E)
> a_2n+1 = [0; (1+c0), c1, c2, ..., ck] (chamemos isso de operacao O)
> 
> 
> A partir disso, acredito que a bijecao fica quase obvia, bastando formalizar 
> algumas inducoes marotas.
> 
> Primeiramente, nenhuma representacao da forma [...,N,1] vai surgir dai a 
> partir de a_2. Isso pode ser demonstrado por inducao mesmo: ck=1 somente no 
> caso [1], e depois dele a funcao a_n so modifica o comeco da cadeia, nunca o 
> final dela.
> 
> Assim sendo, temos certeza que nao tem como um racional aparecer uma vez na 
> forma canonica e outra na forma alternativa. E, por conseguinte, se duas 
> fracoes tem comprimentos diferentes, elas devem ser diferentes. E fracoes com 
> comprimentos iguais diferem se e somente se pelo menos um dos componentes 
> diferir.
> 
> Agora, a funcao recursiva age de duas formas. Uma delas altera o comprimento 
> em 1, e a outra mantém. A que altera, só altera acrescentando o 0 na 
> cabeceira. A que não altera, incrementa a cabeceira.
> 
> Desta forma, é possível gerar de maneira unica qualquer numero racional 
> comecando do 1.
> 
> - Qualquer fracao de comprimento 1 pode ser gerada simplesmente aplicando a 
> operacao E tantas vezes quantas forem necessarias. E tambem nao e possivel 
> fazer isso de outra maneira, pois a operacao O aumentara o comprimento de 
> maneira irreversivel.
> 
> - Dada uma fracao com comprimento K, temos duas sub inducoes para fazer:
> 
> + A fracao tem comprimento K e comeca com 0.
> 
> Â  Entao ela foi gerada por uma operacao O. O elemento que a gerou tinha 
> menos componentes, os quais satisfazem a hipotese de inducao.
> 
> + A fracao tem comprimento K e comeca com algo maior que 0.
> 
> Entao ela foi gerada por uma operacao E. A fracao da qual ela foi gerada 
> difere unicamente no primeiro elemento, o qual antes era menor. Assim sendo, 
> e possivel reduzir isso ate chegar no caso anterior.
> 
> E isso demonstra recursivamente a unicidade e existencia!
> 
> 
> 
>> -- 
>> Esta mensagem foi verificada pelo sistema de antivírus e 
>> acredita-se estar livre de perigo.
> 
> -- 
> Esta mensagem foi verificada pelo sistema de antivírus e 
> acredita-se estar livre de perigo.

-- 
Esta mensagem foi verificada pelo sistema de antiv�rus e
 acredita-se estar livre de perigo.



[obm-l] Re: [obm-l] Re: [obm-l] Re: [obm-l] Re: [obm-l] Sequência Injetiva

2021-02-14 Por tôpico Claudio Buffara
a(1) = 1
a(2n) = a(2n-1) + 1
a(2n+1) = 1/a(2n)

Fazendo a(n) = p(n)/q(n), obtemos duas sequências: p(n) e q(n).
E elas são tais que:
p(1) = q(1) = 1
p(2n) = p(2n-1) + q(2n-1)
q(2n) = q(2n-1)
p(2n+1) = q(2n)
q(2n+1) = p(2n)

Como as sequências começam com 1 e 1, que são primos entre si, e como
mdc(p,q) = mdc(q,p) = mdc(p+q,q), p(n) e q(n) sempre serão primos entre si.

Usando a definição de p e q:
p(2n+1) = q(2n) = q(2n-1) = p(2n-2) = p(2n-3) + q(2n-3) = p(2n-3) + q(2n-2)
= p(2n-3) + p(2n-1)
e
p(2n+2) = p(2n+1) + q(2n+1) = q(2n) + p(2n) = q(2n-1) + p(2n) = p(2n-2) +
p(2n)

Ou seja, os termos p(n) de ordem ímpar e de ordem par realmente formam uma
sequência de Fibonacci.
Os de ordem ímpar começam com p(1) = p(3) = 1 e os de ordem par com p(2) =
2 e p(4) = 3.

[]s,
Claudio.

On Sun, Feb 14, 2021 at 10:03 AM Claudio Buffara 
wrote:

> Ué!  Continua sendo. Só que é outra questão...
>
>
> On Sun, Feb 14, 2021 at 3:34 AM Ralph Costa Teixeira 
> wrote:
>
>> Sim, voce tem razao -- eu achei que era a_2n = a_{2n-1} +1. Que pena, era
>> uma boa questao com Fibonacci. :)
>>
>> On Sun, Feb 14, 2021 at 12:35 AM Claudio Buffara <
>> claudio.buff...@gmail.com> wrote:
>>
>>> Oi, Ralph:
>>>
>>> Eu posso ter entendido errado a definição da sequência, mas achei termos
>>> diferentes dos seus:
>>> 1:  1
>>> 2:  2
>>> 3:  1/2
>>> 4:  3
>>> 5:  1/3
>>> 6:  3/2
>>> 7:  2/3
>>> 8:  4
>>> 9:  1/4
>>> 10:  4/3
>>> 11:  3/4
>>> 12:  5/2
>>> 13:  2/5
>>> 14:  5/3
>>> 15:  3/5
>>> 16:  5
>>> ...
>>>
>>> []s,
>>> Claudio.
>>>
>>>
>>> On Sat, Feb 13, 2021 at 7:59 PM Ralph Costa Teixeira 
>>> wrote:
>>>
>>>> Meio enrolado, vou escrever meio vagamente.
>>>>
>>>> Eu sugiro olhar primeiro para os caras com indice impar. Sao eles:
>>>> a1=1/1
>>>> a3=1/2
>>>> a5=2/3
>>>> a7=3/5
>>>> a8=5/8
>>>> ...
>>>> Ou seja, mostre que eles sao quocientes de numeros de Fibonacci
>>>> consecutivos (os caras de indice par sao os inversos desses). Agora tem
>>>> varias maneiras de continuar:
>>>>
>>>> -- Voce pode mostrar que os numeros de Fibonacci consecutivos sao
>>>> primos entre si; portanto cada fracao dessas fica unicamente determinada
>>>> por numerador e denominador, e (como os numeros de Fibonacci formam uma
>>>> sequencia crescente) vao ser distintos entre si;
>>>> -- Se voce nao quiser entrar no merito do Fibonacci, tente mostrar
>>>> (pode ser por inducao) que a3 < a7 < a11 <...>>> a_(4k+1) < ... < a13 < a9 < a5 < 1  (phi ali seria (raiz(5)-1) / 2, acho).
>>>>
>>>> De qualquer forma, como a_(2n+1)<1, a1=1 e os "a_2n" sao os inversos
>>>> dos "a_2n+1, vao ser todos diferentes.
>>>>
>>>> Abraco, Ralph.
>>>>
>>>>
>>>> On Sat, Feb 13, 2021 at 5:56 PM Jeferson Almir <
>>>> jefersonram...@gmail.com> wrote:
>>>>
>>>>> Amigos, peço ajuda em provar a injetividade dessa sequência que seria
>>>>> uma saída para provar a unica ocorrência do racional que aparece nela.
>>>>> Estou andando em círculos tentando montar uma possível indução.
>>>>>
>>>>>
>>>>> Dado a sequência a_1 = 1 e a_2n = a_n  + 1 e a_2n+1 = 1/a_2n.
>>>>>
>>>>> Prove que para todo racional positivo que ocorre na sequência, ocorre
>>>>> uma única vez.
>>>>>
>>>>> --
>>>>> Esta mensagem foi verificada pelo sistema de antivírus e
>>>>> acredita-se estar livre de perigo.
>>>>
>>>>
>>>> --
>>>> Esta mensagem foi verificada pelo sistema de antivírus e
>>>> acredita-se estar livre de perigo.
>>>
>>>
>>> --
>>> Esta mensagem foi verificada pelo sistema de antivírus e
>>> acredita-se estar livre de perigo.
>>
>>
>> --
>> Esta mensagem foi verificada pelo sistema de antivírus e
>> acredita-se estar livre de perigo.
>
>

-- 
Esta mensagem foi verificada pelo sistema de antiv�rus e
 acredita-se estar livre de perigo.



[obm-l] Re: [obm-l] Re: [obm-l] Re: [obm-l] Re: [obm-l] Sequência Injetiva

2021-02-14 Por tôpico Claudio Buffara
Ué!  Continua sendo. Só que é outra questão...


On Sun, Feb 14, 2021 at 3:34 AM Ralph Costa Teixeira 
wrote:

> Sim, voce tem razao -- eu achei que era a_2n = a_{2n-1} +1. Que pena, era
> uma boa questao com Fibonacci. :)
>
> On Sun, Feb 14, 2021 at 12:35 AM Claudio Buffara <
> claudio.buff...@gmail.com> wrote:
>
>> Oi, Ralph:
>>
>> Eu posso ter entendido errado a definição da sequência, mas achei termos
>> diferentes dos seus:
>> 1:  1
>> 2:  2
>> 3:  1/2
>> 4:  3
>> 5:  1/3
>> 6:  3/2
>> 7:  2/3
>> 8:  4
>> 9:  1/4
>> 10:  4/3
>> 11:  3/4
>> 12:  5/2
>> 13:  2/5
>> 14:  5/3
>> 15:  3/5
>> 16:  5
>> ...
>>
>> []s,
>> Claudio.
>>
>>
>> On Sat, Feb 13, 2021 at 7:59 PM Ralph Costa Teixeira 
>> wrote:
>>
>>> Meio enrolado, vou escrever meio vagamente.
>>>
>>> Eu sugiro olhar primeiro para os caras com indice impar. Sao eles:
>>> a1=1/1
>>> a3=1/2
>>> a5=2/3
>>> a7=3/5
>>> a8=5/8
>>> ...
>>> Ou seja, mostre que eles sao quocientes de numeros de Fibonacci
>>> consecutivos (os caras de indice par sao os inversos desses). Agora tem
>>> varias maneiras de continuar:
>>>
>>> -- Voce pode mostrar que os numeros de Fibonacci consecutivos sao primos
>>> entre si; portanto cada fracao dessas fica unicamente determinada por
>>> numerador e denominador, e (como os numeros de Fibonacci formam uma
>>> sequencia crescente) vao ser distintos entre si;
>>> -- Se voce nao quiser entrar no merito do Fibonacci, tente mostrar (pode
>>> ser por inducao) que a3 < a7 < a11 <...>> < ... < a13 < a9 < a5 < 1  (phi ali seria (raiz(5)-1) / 2, acho).
>>>
>>> De qualquer forma, como a_(2n+1)<1, a1=1 e os "a_2n" sao os inversos dos
>>> "a_2n+1, vao ser todos diferentes.
>>>
>>> Abraco, Ralph.
>>>
>>>
>>> On Sat, Feb 13, 2021 at 5:56 PM Jeferson Almir 
>>> wrote:
>>>
>>>> Amigos, peço ajuda em provar a injetividade dessa sequência que seria
>>>> uma saída para provar a unica ocorrência do racional que aparece nela.
>>>> Estou andando em círculos tentando montar uma possível indução.
>>>>
>>>>
>>>> Dado a sequência a_1 = 1 e a_2n = a_n  + 1 e a_2n+1 = 1/a_2n.
>>>>
>>>> Prove que para todo racional positivo que ocorre na sequência, ocorre
>>>> uma única vez.
>>>>
>>>> --
>>>> Esta mensagem foi verificada pelo sistema de antivírus e
>>>> acredita-se estar livre de perigo.
>>>
>>>
>>> --
>>> Esta mensagem foi verificada pelo sistema de antivírus e
>>> acredita-se estar livre de perigo.
>>
>>
>> --
>> Esta mensagem foi verificada pelo sistema de antivírus e
>> acredita-se estar livre de perigo.
>
>
> --
> Esta mensagem foi verificada pelo sistema de antivírus e
> acredita-se estar livre de perigo.

-- 
Esta mensagem foi verificada pelo sistema de antiv�rus e
 acredita-se estar livre de perigo.



[obm-l] Re: [obm-l] Re: [obm-l] Sequência Injetiva

2021-02-13 Por tôpico Claudio Buffara
Oi, Ralph:

Eu posso ter entendido errado a definição da sequência, mas achei termos
diferentes dos seus:
1:  1
2:  2
3:  1/2
4:  3
5:  1/3
6:  3/2
7:  2/3
8:  4
9:  1/4
10:  4/3
11:  3/4
12:  5/2
13:  2/5
14:  5/3
15:  3/5
16:  5
...

[]s,
Claudio.


On Sat, Feb 13, 2021 at 7:59 PM Ralph Costa Teixeira 
wrote:

> Meio enrolado, vou escrever meio vagamente.
>
> Eu sugiro olhar primeiro para os caras com indice impar. Sao eles:
> a1=1/1
> a3=1/2
> a5=2/3
> a7=3/5
> a8=5/8
> ...
> Ou seja, mostre que eles sao quocientes de numeros de Fibonacci
> consecutivos (os caras de indice par sao os inversos desses). Agora tem
> varias maneiras de continuar:
>
> -- Voce pode mostrar que os numeros de Fibonacci consecutivos sao primos
> entre si; portanto cada fracao dessas fica unicamente determinada por
> numerador e denominador, e (como os numeros de Fibonacci formam uma
> sequencia crescente) vao ser distintos entre si;
> -- Se voce nao quiser entrar no merito do Fibonacci, tente mostrar (pode
> ser por inducao) que a3 < a7 < a11 <... < ... < a13 < a9 < a5 < 1  (phi ali seria (raiz(5)-1) / 2, acho).
>
> De qualquer forma, como a_(2n+1)<1, a1=1 e os "a_2n" sao os inversos dos
> "a_2n+1, vao ser todos diferentes.
>
> Abraco, Ralph.
>
>
> On Sat, Feb 13, 2021 at 5:56 PM Jeferson Almir 
> wrote:
>
>> Amigos, peço ajuda em provar a injetividade dessa sequência que seria uma
>> saída para provar a unica ocorrência do racional que aparece nela. Estou
>> andando em círculos tentando montar uma possível indução.
>>
>>
>> Dado a sequência a_1 = 1 e a_2n = a_n  + 1 e a_2n+1 = 1/a_2n.
>>
>> Prove que para todo racional positivo que ocorre na sequência, ocorre uma
>> única vez.
>>
>> --
>> Esta mensagem foi verificada pelo sistema de antivírus e
>> acredita-se estar livre de perigo.
>
>
> --
> Esta mensagem foi verificada pelo sistema de antivírus e
> acredita-se estar livre de perigo.

-- 
Esta mensagem foi verificada pelo sistema de antiv�rus e
 acredita-se estar livre de perigo.



[obm-l] Re: [obm-l] Sequência Injetiva

2021-02-13 Por tôpico Claudio Buffara
Se a sequência é:
a(1) = 1
a(2n) = a(n) + 1
a(2n+1) = 1/a(2n),
então:
Como os termos da sequência são positivos, os termos de ordem par são
maiores do que 1 e os de ordem ímpar (e maior do que 1) são menores do que
1.
Se houver alguma repetição, então o primeiro termo a(n) a ser repetido
deverá índice n par, já que cada termo de ordem ímpar > 1 é simplesmente o
inverso do termo anterior.
Assim, suponhamos que m seja o menor natural tal que a(2m) = a(2m+2k), para
algum k > 0.
Mas isso implica que a(m) + 1 = a(m+k) + 1 ==> a(m) = a(m+k).
Se m for par, isso é uma contradição à escolha de m.
Se m for ímpar, então k é par (já que m+k terá que ser ímpar também) e,
pela definição da sequência, a(m-1) = a(m+k-1) ==> contradição à escolha de
m.
Logo, a sequência é injetiva.

[]s,
Claudio.


On Sat, Feb 13, 2021 at 5:56 PM Jeferson Almir 
wrote:

> Amigos, peço ajuda em provar a injetividade dessa sequência que seria uma
> saída para provar a unica ocorrência do racional que aparece nela. Estou
> andando em círculos tentando montar uma possível indução.
>
>
> Dado a sequência a_1 = 1 e a_2n = a_n  + 1 e a_2n+1 = 1/a_2n.
>
> Prove que para todo racional positivo que ocorre na sequência, ocorre uma
> única vez.
>
> --
> Esta mensagem foi verificada pelo sistema de antivírus e
> acredita-se estar livre de perigo.

-- 
Esta mensagem foi verificada pelo sistema de antiv�rus e
 acredita-se estar livre de perigo.



Re: [obm-l] prova por inducao

2021-01-29 Por tôpico Claudio Buffara
Ponha a = raiz(2).
Então, vc precisa provar que, para n >= 2, a^(2n) > 1 + n*a^(n-1) <==> a^n
> 1/a^n + n/a.
Pra n = 2 isso é verdade.
Suponha que, para um dado n >= 2, 1/a^n + n/a < a^n  (H.I.)
Então 1/a^(n+1) + (n+1)/a < 1/a^n + 1/a + n/a = 1/a + (1/a^n + n/a) < 1/a +
a^n (pela H.I.)

Agora, resta provar que 1/a + a^n < a^(n+1), para n >= 2.
Mas isso é equivalente a   1 + a^(n+1) < a*a^(n+1) <==> (a-1)*a^(n+1) > 1.
Só que:
(a-1)*a^(n+1) >= (a-1)*a^3 = a^4 - a^3 = 4 - 2*raiz(2) > 1,  pois  raiz(2)
< 3/2.

[]s,
Claudio.

On Fri, Jan 29, 2021 at 3:56 PM Phablo dos Santos 
wrote:

> Mostre que 2ⁿ > 1 + n√(2ⁿ⁻¹), para todo n≥2.
>
> Eu sei a prova desse problema partindo do caminho da indução, porém estou
> tendo problemas tentando prová-lo pelo caminho da hipótese e gostaria da
> ajuda de vcs nele. Vou postar aqui até onde cheguei com minha solução:
>
> Caso inicial n=2:   2² > 1+2√2
> Hip: n, n>2 :   2ⁿ > 1+n√(2ⁿ⁻¹)
> Ind: n+1, n>2 :   2ⁿ⁺¹ > 1+(n+1)√(2ⁿ)
> Pela hipótese:.  2ⁿ*2 > (1+n√(2ⁿ⁻¹))*2
> ∴ 2ⁿ⁺¹ > 2 + 2n√(2ⁿ⁻¹)=2+(√2)n√(2ⁿ) > 1+n√(2ⁿ).
> A partir daí eu n consigo mais desenvolver. Desde já agradeço pela ajuda
>
>


Re: [obm-l] Amigo secreto ENEM

2021-01-27 Por tôpico Claudio Buffara
Mas daí me parece que temos 3 conjuntos distintos (supondo que ninguém se
auto-presenteia):
1) o dos desarranjos de N pessoas;
2) o das sequências de N presenteados;
3) o dos diferentes jogos de amigo oculto com N pessoas (que o seu exemplo
mostrou ser diferente de (2): duas sequências idênticas de presenteados,
com uma requerendo 1 sorteio intermediário e a outra requerendo 2 sorteios
intermediários pra retomar o jogo)

(1) tem !N elementos.

Quantos elementos têm (2) e (3)?

[]s,
Claudio.

On Wed, Jan 27, 2021 at 12:12 PM Claudio Buffara 
wrote:

> Muito obrigado, Ralph!
>
> Muito interessante!
> Meu caso particular foi pequeno demais.
> Daí eu só vi a situação em que um dado desarranjo origina duas (ou mais)
> sequências distintas de presenteados.
>
> Mas, como vc bem mostrou, com 6 ou mais participantes pode ocorrer a
> situação "dual": uma mesma sequência de presenteados sendo oriunda de dois
> desarranjos distintos:
> Com A sorteado pra começar, a sequência de presenteados B-A-C-D-E-F pode
> vir de:
> (AB)(CD)(EF), com D e F sendo sorteados pra retomar o jogo (após A e D
> serem presenteados, respectivamente)
> ou de:
> (AB)(CDEF), com F sendo sorteado pra retomar o jogo (após A ser
> presenteado)
>
> Acho que isso dá um bom artigo.
>
> []s,
> Claudio.
>
> On Tue, Jan 26, 2021 at 10:01 PM Ralph Costa Teixeira 
> wrote:
>
>> Oi, Claudio.
>>
>> Primeiro, parece que o video supõe que NÃO podem haver "auto-sorteios"
>> (isto fica implícito quando ele diz que a primeira a entregar não pode ser
>> a primeira e receber nem a penúltima, evitando que o último se de um
>> presente). Vou supor isso daqui para a frente.
>>
>> Mas o problema é que o video usa implicitamente que o amigo secreto todo
>> (SORTEIO + ENTREGA) com N pessoas fica determinado por:
>> (i) A primeira pessoa que entrega.
>> E
>> (ii) A sequência de N pessoas que recebem.
>>
>> Bom, não funciona para N acima de, huh, 5 eu acho. Deixa eu dar um
>> exemplo com N=6 para facilitar. Se a gente tivesse:
>> A + BACDEF
>> O que significa isso? Vou usar ">" para indicar "deu presente para". Note
>> as possíveis interpretações disso:
>> A>B B>A; C>D D>E E>F F>C
>> ou
>> A>B B>A; C>D D>C; E>F F>E
>> Ou seja, esta sequência em particular representa DOIS possíveis sorteios.
>> Por outro lado:
>> A + BCDEFA
>> tem uma unica interpretação possível:
>> A>B B>C C>D D>E E>F F>A
>> Por causa disso, as "sequências" que ele criou não são equiprováveis, e
>> isso derruba o argumento.
>>
>> (Vou escrever isso no canal dele)
>>
>> Abraço, Ralph.
>>
>> On Tue, Jan 26, 2021 at 6:37 PM Claudio Buffara <
>> claudio.buff...@gmail.com> wrote:
>>
>>> Oi, Ralph:
>>>
>>> Onde está o erro da solução apresentada no vídeo abaixo?
>>> https://www.youtube.com/watch?v=c-t_BAMASKE=youtu.be
>>>
>>> Eu entendo que se um dado desarranjo tiver 2 ou mais ciclos, então
>>> quando cada ciclo até o penúltimo for "exaurido", uma nova pessoa deverá
>>> ser sorteada (dentre aquelas que ainda não deram nem receberam presentes)
>>> para continuar o jogo.
>>> Neste caso, um mesmo desarranjo pode dar origem a várias sequências
>>> distintas de presenteados.
>>> Por exemplo, com 4 pessoas (numeradas de 1 a 4), se o desarranjo for
>>> (12)(34) e a pessoa 1 for sorteada para começar, então:
>>> 1 presenteia 2 que presenteia 1.
>>> Daí, uma nova pessoa deverá ser sorteada (no caso, 3 ou 4) e a
>>> brincadeira poderá continuar de 2 maneiras diferentes:
>>> - 3 presenteia 4 que presenteia 3
>>> ou
>>> - 4 presenteia 3 que presenteia 4.
>>> Mas ambas correspondem ao mesmo desarranjo (12)(34).
>>>
>>> A necessidade destes sorteios intermediários para continuar o jogo
>>> parece complicar bastante a análise com base em desarranjos.
>>> Daí eu achei interessante o raciocínio apresentado no vídeo, que leva em
>>> conta apenas a pessoa A sorteada pra dar o primeiro presente e a sequências
>>> de presenteados, e toma o cuidado de excluir dos casos possíveis as
>>> sequências de presenteados que têm A na primeira posição (para evitar que A
>>> se auto-presenteie) e na penúltima posição (para evitar que o último
>>> presenteado se auto-presenteie).
>>> Não consegui ver onde está o erro.
>>>
>>> []s,
>>> Claudio.
>>>
>>> On Tue, Jan 26, 2021 at 5:26 PM Ralph Costa Teixeira 
>>>

Re: [obm-l] Amigo secreto ENEM

2021-01-27 Por tôpico Claudio Buffara
Muito obrigado, Ralph!

Muito interessante!
Meu caso particular foi pequeno demais.
Daí eu só vi a situação em que um dado desarranjo origina duas (ou mais)
sequências distintas de presenteados.

Mas, como vc bem mostrou, com 6 ou mais participantes pode ocorrer a
situação "dual": uma mesma sequência de presenteados sendo oriunda de dois
desarranjos distintos:
Com A sorteado pra começar, a sequência de presenteados B-A-C-D-E-F pode
vir de:
(AB)(CD)(EF), com D e F sendo sorteados pra retomar o jogo (após A e D
serem presenteados, respectivamente)
ou de:
(AB)(CDEF), com F sendo sorteado pra retomar o jogo (após A ser presenteado)

Acho que isso dá um bom artigo.

[]s,
Claudio.

On Tue, Jan 26, 2021 at 10:01 PM Ralph Costa Teixeira 
wrote:

> Oi, Claudio.
>
> Primeiro, parece que o video supõe que NÃO podem haver "auto-sorteios"
> (isto fica implícito quando ele diz que a primeira a entregar não pode ser
> a primeira e receber nem a penúltima, evitando que o último se de um
> presente). Vou supor isso daqui para a frente.
>
> Mas o problema é que o video usa implicitamente que o amigo secreto todo
> (SORTEIO + ENTREGA) com N pessoas fica determinado por:
> (i) A primeira pessoa que entrega.
> E
> (ii) A sequência de N pessoas que recebem.
>
> Bom, não funciona para N acima de, huh, 5 eu acho. Deixa eu dar um exemplo
> com N=6 para facilitar. Se a gente tivesse:
> A + BACDEF
> O que significa isso? Vou usar ">" para indicar "deu presente para". Note
> as possíveis interpretações disso:
> A>B B>A; C>D D>E E>F F>C
> ou
> A>B B>A; C>D D>C; E>F F>E
> Ou seja, esta sequência em particular representa DOIS possíveis sorteios.
> Por outro lado:
> A + BCDEFA
> tem uma unica interpretação possível:
> A>B B>C C>D D>E E>F F>A
> Por causa disso, as "sequências" que ele criou não são equiprováveis, e
> isso derruba o argumento.
>
> (Vou escrever isso no canal dele)
>
> Abraço, Ralph.
>
> On Tue, Jan 26, 2021 at 6:37 PM Claudio Buffara 
> wrote:
>
>> Oi, Ralph:
>>
>> Onde está o erro da solução apresentada no vídeo abaixo?
>> https://www.youtube.com/watch?v=c-t_BAMASKE=youtu.be
>>
>> Eu entendo que se um dado desarranjo tiver 2 ou mais ciclos, então quando
>> cada ciclo até o penúltimo for "exaurido", uma nova pessoa deverá ser
>> sorteada (dentre aquelas que ainda não deram nem receberam presentes) para
>> continuar o jogo.
>> Neste caso, um mesmo desarranjo pode dar origem a várias sequências
>> distintas de presenteados.
>> Por exemplo, com 4 pessoas (numeradas de 1 a 4), se o desarranjo for
>> (12)(34) e a pessoa 1 for sorteada para começar, então:
>> 1 presenteia 2 que presenteia 1.
>> Daí, uma nova pessoa deverá ser sorteada (no caso, 3 ou 4) e a
>> brincadeira poderá continuar de 2 maneiras diferentes:
>> - 3 presenteia 4 que presenteia 3
>> ou
>> - 4 presenteia 3 que presenteia 4.
>> Mas ambas correspondem ao mesmo desarranjo (12)(34).
>>
>> A necessidade destes sorteios intermediários para continuar o jogo parece
>> complicar bastante a análise com base em desarranjos.
>> Daí eu achei interessante o raciocínio apresentado no vídeo, que leva em
>> conta apenas a pessoa A sorteada pra dar o primeiro presente e a sequências
>> de presenteados, e toma o cuidado de excluir dos casos possíveis as
>> sequências de presenteados que têm A na primeira posição (para evitar que A
>> se auto-presenteie) e na penúltima posição (para evitar que o último
>> presenteado se auto-presenteie).
>> Não consegui ver onde está o erro.
>>
>> []s,
>> Claudio.
>>
>> On Tue, Jan 26, 2021 at 5:26 PM Ralph Costa Teixeira 
>> wrote:
>>
>>> Deixa eu copiar o que escrevi em outro lugar... :D :D
>>>
>>> Primeiro: não fica claro do enunciado se "auto-sorteios" (alguém sortear
>>> o próprio nome) são permitidos ou não, e isto ALTERA a resposta. :(
>>>
>>> Vejamos possíveis respostas corretas:
>>>
>>> ---///---
>>>
>>> SE AUTO-SORTEIOS FOREM PERMITIDOS:
>>> Em resumo, temos 1/10 de chance de A iniciar o sorteio, e 1/10 de chance
>>> de B terminar (1/10 sim, pois A *pode* terminar). Assim, a resposta seria
>>> 1/10*1/10*2=1/50.
>>>
>>> Com mais detalhes para justificar o segundo "1/10":
>>> -- Número de sorteios possíveis = 10!
>>> -- Número de sorteios que formam um único ciclo de tamanho 10 = 9!
>>> -- Note que ter um ciclo de tamanho 10 equivale a terminar com quem
>>> inicia; portanto, a chance de o amigo secreto terminar

Re: [obm-l] Amigo secreto ENEM

2021-01-26 Por tôpico Claudio Buffara
Oi, Ralph:

Onde está o erro da solução apresentada no vídeo abaixo?
https://www.youtube.com/watch?v=c-t_BAMASKE=youtu.be

Eu entendo que se um dado desarranjo tiver 2 ou mais ciclos, então quando
cada ciclo até o penúltimo for "exaurido", uma nova pessoa deverá ser
sorteada (dentre aquelas que ainda não deram nem receberam presentes) para
continuar o jogo.
Neste caso, um mesmo desarranjo pode dar origem a várias sequências
distintas de presenteados.
Por exemplo, com 4 pessoas (numeradas de 1 a 4), se o desarranjo for
(12)(34) e a pessoa 1 for sorteada para começar, então:
1 presenteia 2 que presenteia 1.
Daí, uma nova pessoa deverá ser sorteada (no caso, 3 ou 4) e a brincadeira
poderá continuar de 2 maneiras diferentes:
- 3 presenteia 4 que presenteia 3
ou
- 4 presenteia 3 que presenteia 4.
Mas ambas correspondem ao mesmo desarranjo (12)(34).

A necessidade destes sorteios intermediários para continuar o jogo parece
complicar bastante a análise com base em desarranjos.
Daí eu achei interessante o raciocínio apresentado no vídeo, que leva em
conta apenas a pessoa A sorteada pra dar o primeiro presente e a sequências
de presenteados, e toma o cuidado de excluir dos casos possíveis as
sequências de presenteados que têm A na primeira posição (para evitar que A
se auto-presenteie) e na penúltima posição (para evitar que o último
presenteado se auto-presenteie).
Não consegui ver onde está o erro.

[]s,
Claudio.

On Tue, Jan 26, 2021 at 5:26 PM Ralph Costa Teixeira 
wrote:

> Deixa eu copiar o que escrevi em outro lugar... :D :D
>
> Primeiro: não fica claro do enunciado se "auto-sorteios" (alguém sortear o
> próprio nome) são permitidos ou não, e isto ALTERA a resposta. :(
>
> Vejamos possíveis respostas corretas:
>
> ---///---
>
> SE AUTO-SORTEIOS FOREM PERMITIDOS:
> Em resumo, temos 1/10 de chance de A iniciar o sorteio, e 1/10 de chance
> de B terminar (1/10 sim, pois A *pode* terminar). Assim, a resposta seria
> 1/10*1/10*2=1/50.
>
> Com mais detalhes para justificar o segundo "1/10":
> -- Número de sorteios possíveis = 10!
> -- Número de sorteios que formam um único ciclo de tamanho 10 = 9!
> -- Note que ter um ciclo de tamanho 10 equivale a terminar com quem
> inicia; portanto, a chance de o amigo secreto terminar com o mesmo que
> iniciou seria 9!/10!=1/10 (*que é independente de quem começa*).
>
> Assim:
> -- Chance de A iniciar = 1/10;
> Agora, DADO QUE A INICIOU:
>  Chance de A terminar = 9!/10! = 1/10
>  Portanto, chance de não terminar com A: 9/10
>  Chance de B terminar (por simetria): (9/10) /9 = 1/10
>
> Isso nos dá 1/10*1/10 = 1/100 de chance do amigo secreto começar por A e
> terminar com B. Portanto a resposta seria o dobro, 1/50.
>
> ---///---
> SE AUTO-SORTEIOS SÃO PROIBIDOS:
> -- Número de sorteios (desarranjos) possíveis = !10 (vou escrever K=!10
> daqui por diante);
> -- Número de sorteios que formam um único ciclo de tamanho 10 = 9!
> -- Portanto, a chance de o amigo secreto terminar com o mesmo que iniciou
> seria 9!/K (que é independente de quem começa).
>
> Assim:
> -- Chance de A iniciar = 1/10;
> Agora, DADO QUE A INICIOU:
>  Chance de A terminar = 9!/K
>  Portanto, chance de não terminar com A: 1-9!/K
>  Chance de B terminar (por simetria): (1-9!/K) /9 = (K-9!)/(9K)
>
> Isso nos dá 1/10* (K-9!)/(9K) = (K-9!)/(90K) de chance do amigo secreto
> começar por A e terminar com B. Portanto a resposta seria o dobro,
> (K-9!)/(45K). Fazendo a conta com a ajuda do computador, achei 12001/741645.
>
> Abraço, Ralph.
>
>
>
> On Tue, Jan 26, 2021 at 1:45 PM Professor Vanderlei Nemitz <
> vanderma...@gmail.com> wrote:
>
>> Oi, pessoal!
>>
>> Com certeza vocês estão acompanhando desde domingo as resoluções da
>> questão do ENEM do amigo secreto.
>> Além da resposta proposta, *1/45*, que parece não estar correta, já vi
>> outras duas, *12001/741645* (ETAPA e ANGLO), que consideram também que o
>> sorteio anterior para definir "quem presenteia quem", e *7/360*, do
>> vídeo a seguir:
>>
>> https://www.youtube.com/watch?v=c-t_BAMASKE
>>
>> Gostaria da opinião (e se possível, uma resolução) dos especialistas da
>> lista (Ralph e cia :))
>>
>> Muito obrigado!
>>
>>
>>
>>
>>
>


[obm-l] Re: [obm-l] Ângulos de um triângulo

2020-12-04 Por tôpico Claudio Buffara
Aliás, de posse da expressão para BAD e CAD, um exercício razoavelmente
fácil de programação (até em planilha), é descobrir para quais triângulos
isósceles com ângulos inteiros (em graus) e quais ângulos DBC e DCB
inteiros, BAD (e obviamente CAD) também são inteiros.

Daí, um problema (não mais um exercício!) é descobrir o padrão por trás
destes triângulos especiais.

On Fri, Dec 4, 2020 at 1:42 PM Claudio Buffara 
wrote:

> Usando áreas - em particular, área(ABC) = (1/2)*AB*AC*sen(A) - você
> consegue, com alguma facilidade, expressar a tangente de DAC em termos de
> senos e cossenos dos ângulos dados.   Daí, é só calcular (com calculadora
> ou computador - eu uso Excel ou Wolfram Alpha).  E, de fato, AD divide BAC,
> que mede 48 graus, em dois ângulos: um medindo 30 e o outro 18 graus.
>
> O que não dá é - em 2020 - ficar manipulando aquelas fórmulas de
> prostaférese ou identidades trigonométricas obscuras envolvendo ângulos
> múltiplos de 3 graus. Isso é coisa do século 19...
>
> []s,
> Claudio.
>
> On Mon, Nov 30, 2020 at 7:28 PM Professor Vanderlei Nemitz <
> vanderma...@gmail.com> wrote:
>
>> Boa noite!
>> Alguém conhece uma saída para o seguinte problema?
>> Muito obrigado!
>>
>> *Num triângulo isósceles ABC, AB = AC.*
>> *Seja D um ponto interno tal que os ângulos DBC, DCB, DBA e DCA medem,
>> respectivamente, 12°, 18°, 54° e 48°. *
>> *Determine a medida do ângulo DAC.*
>>
>>
>> <https://www.avast.com/sig-email?utm_medium=email_source=link_campaign=sig-email_content=webmail>
>>  Livre
>> de vírus. www.avast.com
>> <https://www.avast.com/sig-email?utm_medium=email_source=link_campaign=sig-email_content=webmail>.
>>
>> <#m_-4747407596740689255_m_4608836649714424769_DAB4FAD8-2DD7-40BB-A1B8-4E2AA1F9FDF2>
>>
>


[obm-l] Re: [obm-l] Ângulos de um triângulo

2020-12-04 Por tôpico Claudio Buffara
Usando áreas - em particular, área(ABC) = (1/2)*AB*AC*sen(A) - você
consegue, com alguma facilidade, expressar a tangente de DAC em termos de
senos e cossenos dos ângulos dados.   Daí, é só calcular (com calculadora
ou computador - eu uso Excel ou Wolfram Alpha).  E, de fato, AD divide BAC,
que mede 48 graus, em dois ângulos: um medindo 30 e o outro 18 graus.

O que não dá é - em 2020 - ficar manipulando aquelas fórmulas de
prostaférese ou identidades trigonométricas obscuras envolvendo ângulos
múltiplos de 3 graus. Isso é coisa do século 19...

[]s,
Claudio.

On Mon, Nov 30, 2020 at 7:28 PM Professor Vanderlei Nemitz <
vanderma...@gmail.com> wrote:

> Boa noite!
> Alguém conhece uma saída para o seguinte problema?
> Muito obrigado!
>
> *Num triângulo isósceles ABC, AB = AC.*
> *Seja D um ponto interno tal que os ângulos DBC, DCB, DBA e DCA medem,
> respectivamente, 12°, 18°, 54° e 48°. *
> *Determine a medida do ângulo DAC.*
>
>
> 
>  Livre
> de vírus. www.avast.com
> .
> <#m_4608836649714424769_DAB4FAD8-2DD7-40BB-A1B8-4E2AA1F9FDF2>
>


Re: [obm-l] Eliminar parâmetro t

2020-11-17 Por tôpico Claudio Buffara
Você quer eliminar t em algo como:
x = at + b/t
y = ct + d/t

Pra começar, faça u = x/b e v = y/d.
Daí vem:
u = pt + 1/t
v = qt + 1/t

Isso é um sistema linear nas variáveis t e 1/t, cuja solução é:
t = (u-v)/(p-q)
1/t = (qu-pv)/(q-p)

Multiplicando as duas equações acima e eliminando denominadores...
(u-v)(qu-pv) + (p-q)^2 = 0

Agora é só voltar às variáveis originais x e y.

[]s,
Claudio 

> Em 16 de nov. de 2020, à(s) 21:25, Luís Lopes  
> escreveu:
> 
> 
> Sauda,c~oes, 
> 
> Num problema de encontrar o lugar geométrico do vértice 
> A de um triângulo, encontrei como valores das coordenadas 
> x_A e y_A as seguintes expressões: 
> 
> A(x_A,y_A) com  x_A = N/D,  y_A=P/D, onde N=m(v^2+t^2); 
> D=t(1+m^2); P=m^2 v^2 - t^2.
> 
> Fora t, que tem que ser eliminado, todos os outros parâmetros 
> são fixos e conhecidos. 
> 
> Não consegui fazer, obtendo sempre uma identidade 0=0. 
> 
> Um amigo que já me ajudou nessas questões mandou a resposta, 
> obtida por computador. Para facilitar meus cálculos, tinha feito a=2v. 
> Daí o  na fórmula enviada:
> 
> -a^2 m + 4 m x^2 - 4 x y + 4 m^2 x y - 4 m y^2 = 0.
> 
> O lugar geométrico é uma hipérbole equilátera. O locus está correto. 
> 
> Como fazer isso ? Outras eliminações mais difíceis que ele me enviou 
> eu nem tentaria fazer à mão. Mas essa não parecia difícil. 
> Como fazer ? Qual a técnica ? Deve haver uma para o computador e 
> casos complicados. 
> 
> Luís
> 


[obm-l] Re: [obm-l] Re: [obm-l] Re: [obm-l] Re: [obm-l] Re: [obm-l] Re: [obm-l] Relação de girard

2020-11-16 Por tôpico Claudio Buffara
Sugestão: proponha pra eles o problema de determinar se é possível atribuir
sinais "+" ou "-" a cada um dos números:
1  2  3  4  5  6  7  8  9  10
de modo que a soma algébrica (com sinal) destes números seja igual a zero.
Isso é um desafio e é razoavelmente lúdico, apesar de envolver conceitos
que uma criança de 8 anos entenderia.

On Sat, Nov 14, 2020 at 4:22 PM Israel Meireles Chrisostomo <
israelmchrisost...@gmail.com> wrote:

> Desculpe é q eu queria propor algo q fosse lúdico, mais um desafio,
> voltada para jovens adolescentes, algo descompromissado, sem muitas
> complicações com formalidades
>
> Em qui, 12 de nov de 2020 09:10, Anderson Torres <
> torres.anderson...@gmail.com> escreveu:
>
>>
>>
>> Em sáb., 7 de nov. de 2020 às 16:44, Israel Meireles Chrisostomo <
>> israelmchrisost...@gmail.com> escreveu:
>>
>>>   o objetivo dessa proposta é recriar o ambiente vivido por Euler na
>>> época.
>>>
>>
>> E naquele tempo eles não usavam indução? Formalização é algo bem recente
>> na matemática.
>>
>> Sua exigência me parece algo tão surreal quanto exigir rigor na geometria
>> do tempo de Euclides.
>>
>>
>>
>>>
>>> Em sáb., 7 de nov. de 2020 às 15:10, Israel Meireles Chrisostomo <
>>> israelmchrisost...@gmail.com> escreveu:
>>>
 Na verdade eu estava elaborando um problema que dependia disso.O
 problema é esse aqui:

 Desafio do ano: resolver o problema da Basiléia sem usar derivadas,
 integrais, série de potências, produto infinito do seno ou cosseno, ou
 mesmo indução ou números complexos.

 Em sáb., 7 de nov. de 2020 às 15:07, Israel Meireles Chrisostomo <
 israelmchrisost...@gmail.com> escreveu:

> Na verdade eu estava elaborando um problema que dependia disso.O
> problema é esse aqui:
>
> Desafio do ano: resolver o problema da Basiléia sem usar derivadas,
> integrais, série de potências, produto infinito do seno ou cosseno, ou
> mesmo indução.
>
> Em sáb., 7 de nov. de 2020 às 14:47, Israel Meireles Chrisostomo <
> israelmchrisost...@gmail.com> escreveu:
>
>> conheço uma que usa o teorema de d'lambert
>>
>> Em sáb., 7 de nov. de 2020 às 12:50, Bernardo Freitas Paulo da Costa <
>> bernardo...@gmail.com> escreveu:
>>
>>> On Thu, Nov 5, 2020 at 9:26 PM Artur Costa Steiner
>>>  wrote:
>>> >
>>> > Para facilitar, suponhamos que o polinômio de grau n P seja
>>> mônico. Sejam z_1, , z_n suas n raízes não necessariamente 
>>> distintas.
>>> Para todo complexo z, temos que
>>> >
>>> > P(z)  = ( z - z_1) (z - z_n)
>>> >
>>> > Desenvolvendo e aplicando o chamado produto de Stevin, vc tem as
>>> relações de Girard.
>>>
>>> Eu não conhecia o produto de Stevin, mas de forma geral quando você
>>> usa "..." tem, muitas vezes, um argumento por indução que está
>>> subentendido.  Pode ser que o produto de Stevin "faça a indução pra
>>> você" (calculando os termos \sum \prod z_i que vão aparecer como
>>> coeficientes dos monômios z^k), mas é "quase" como se você estivesse
>>> empurrando a indução um andar abaixo ;-)
>>>
>>>
>>> Israel: qual a demonstração por indução que você conhece?  E porque
>>> você gostaria de outra??
>>>
>>> Abraços,
>>> --
>>> Bernardo Freitas Paulo da Costa
>>>
>>>
>>> =
>>> Instru�ões para entrar na lista, sair da lista e usar a lista em
>>> http://www.mat.puc-rio.br/~obmlistas/obm-l.html
>>>
>>> =
>>>
>>
>>
>> --
>> Israel Meireles Chrisostomo
>>
>
>
> --
> Israel Meireles Chrisostomo
>


 --
 Israel Meireles Chrisostomo

>>>
>>>
>>> --
>>> Israel Meireles Chrisostomo
>>>
>>


[obm-l] Re: [obm-l] [obm-l] Teoria dos Números

2020-10-06 Por tôpico Claudio Buffara
Há outros dois: (1,2,2) e (2,3,6).


On Tue, Oct 6, 2020 at 5:14 PM Marcos Duarte 
wrote:

> Boa tarde!
>
> Encontre todos os números naturais a,b,c tais que a<=b<=c e a soma 1/a +
> 1/b + 1/c seja um inteiro.
>
> O único limitante que encontrei é que a < 4, pois 1/4 + 1/4 + 1/4 = 3/4 <
> 1 e já que a + 1 > a => 1/(a+1) < 1/a, temos que para a > 4 a soma continua
> menor que 1. Além disso, (1,1,1) e (3,3,3) satisfazem.
>
> --
> Esta mensagem foi verificada pelo sistema de antivírus e
> acredita-se estar livre de perigo.

-- 
Esta mensagem foi verificada pelo sistema de antiv�rus e
 acredita-se estar livre de perigo.



[obm-l] Re: [obm-l] Sequência das médias ponderadas

2020-08-26 Por tôpico Claudio Buffara
Acho que isso tá mal formulado.
Por exemplo,quanto é s_3?

On Tue, Aug 25, 2020 at 3:49 PM Artur Costa Steiner <
artur.costa.stei...@gmail.com> wrote:

> Isso me foi dado como verdadeiro, mas ainda não cheguei a uma conclusão.
>
> Sejam (a_ n) uma sequência de reais positivos e (s_n) a sequência das
> médias ponderadas de (a_n,) com relação aos pesos positivos (p_n).
> Suponhamos que lim p_n = p, 0 < p < oo, e que a sequência das médias
> aritméticas de (a_n) convirja para o real a. Então, s_n --> a.
>
> Abraços
> Artur
>
> --
> Esta mensagem foi verificada pelo sistema de antivírus e
> acredita-se estar livre de perigo.

-- 
Esta mensagem foi verificada pelo sistema de antiv�rus e
 acredita-se estar livre de perigo.



[obm-l] Re: [obm-l] Re: [obm-l] Re: [obm-l] Elipse e lugar geométrico

2020-08-24 Por tôpico Claudio Buffara
Tem um artigo do (saudoso) Morgado na RPM sobre este assunto. Está aqui:
http://www.rpm.org.br/cdrpm/43/5.htm

[]s,
Claudio.

On Sat, Aug 22, 2020 at 9:14 PM Professor Vanderlei Nemitz <
vanderma...@gmail.com> wrote:

> Demorei para responder, mas queria dizer que foi muito boa sua resolução,
> como sempre, Ralph!
> Eu desconhecia o fato de as coordenadas do incentro serem dadas daquela
> forma.
>
> Muitíssimo obrigado!
>
> Vanderlei
>
>
> 
>  Livre
> de vírus. www.avast.com
> .
> <#m_3828508563874758992_DAB4FAD8-2DD7-40BB-A1B8-4E2AA1F9FDF2>
>
> Em qui., 20 de ago. de 2020 às 00:37, Ralph Costa Teixeira <
> ralp...@gmail.com> escreveu:
>
>> As coordenadas do incentro sao a media ponderada das coordenadas dos
>> vertices, usando os lados como pesos. Ou seja, se escrevo P=(5cost,4sint),
>> F1=(-3,0), F2=(3,0) e Incentro=(x,y):
>>
>> x = (30cost + (-3)b + 3c) / 16
>> y = (24sint + 0 + 0) / 16
>>
>> onde b=d(P,F2) e c=d(P,F1). Note que b+c=eixo maior = 10.
>>
>> Mais especificamente:
>>
>> b^2=d(P,F2)^2=(5cost-3)^2+(4sint)^2=9(cost)^2-30cost+25=(3cost-5)^2, ou
>> seja, b=5-3cost, portanto c=5+3cost.
>>
>> Jogando na formula de x e y:
>>
>> x= 3cost ; y=3sint/2. Outra elipse, a saber, (x^2)/9+(y^2)/(9/4)=1
>> (talvez tirando os pontos onde tudo degenera, para ser chato).
>>
>> Abraço, Ralph.
>>
>>
>>
>> Hmm Assim:
>>
>> On Wed, Aug 19, 2020 at 11:58 PM Professor Vanderlei Nemitz <
>> vanderma...@gmail.com> wrote:
>>
>>> Oi!
>>> Venho com mais uma envolvendo incentro.
>>>
>>> *O ponto P pertence a uma elipse de focos F1 e F2 e de equação (x^2)/25
>>> + (y^2)/16 = 1. Determine o lugar geométrico do incentro do triângulo
>>> PF1F2.*
>>>
>>> Muito obrigado!
>>>
>>>
>>> 
>>>  Livre
>>> de vírus. www.avast.com
>>> .
>>>
>>> <#m_3828508563874758992_m_6041639077674691514_m_2344932968934913062_DAB4FAD8-2DD7-40BB-A1B8-4E2AA1F9FDF2>
>>>
>>> --
>>> Esta mensagem foi verificada pelo sistema de antivírus e
>>> acredita-se estar livre de perigo.
>>
>>
>> --
>> Esta mensagem foi verificada pelo sistema de antivírus e
>> acredita-se estar livre de perigo.
>
>
> --
> Esta mensagem foi verificada pelo sistema de antivírus e
> acredita-se estar livre de perigo.

-- 
Esta mensagem foi verificada pelo sistema de antiv�rus e
 acredita-se estar livre de perigo.



[obm-l] Re: [obm-l] Re: [obm-l] Re: [obm-l] Re: [obm-l] Geometria plana com desigualdade de médias?

2020-08-18 Por tôpico Claudio Buffara
Realmente, não era isso que eu estava procurando...  mas valeu! É outra
solução.


On Tue, Aug 18, 2020 at 7:51 PM Pedro José  wrote:

> Boa noite!
> Cláudio,
> não consegui nada geométrico.
> O máximo que atingi foi:
> a/ha + b/hb + c/hc= [cotg(A1) +cotg (A2)]  + [cotg(B1) +cotg (B2)] +
> co[tg(C1) +cotg (C2)] com A1 + A2 = A; B1 + B2 + B e C1 + C2 = C.
> Para ser mínimo cada termo entre colchetes deve ser mínimo, o que ocorre
> quando A1 = A2; B1 = B2 e C1 = C2. Logo P seria o encontro das bissetrizes
> e logo I.
> Onde: A1= PAB e A2=PAC; B1=PBA e B2=PBC; C1=PCA e C2=PCB.
>
> Saudações,
> PJMS
>
> Em ter., 18 de ago. de 2020 às 11:34, Claudio Buffara <
> claudio.buff...@gmail.com> escreveu:
>
>> Será que tem uma demonstração mais geométrica e menos algébrica disso? E
>> que torne o resultado mais intuitivo?
>> É razoável que o ponto P não esteja muito próximo de qualquer dos lados,
>> pois neste caso, se P se aproximasse do lado a, por exemplo,
>> a/h_a cresceria e a expressão se afastaria do valor mínimo.
>> Mas, com lados não necessariamente congruentes, não é óbvio, a priori,
>> que P deva ser equidistante dos três.
>> De fato, seria razoável esperar que P estivesse mais próximo do maior
>> lado e conjecturar, por exemplo, que o P que minimiza a expressão é tal que
>> a/h_a = b/h_b = c/h_c.
>> O fato de P ser o incentro não me parece a conjectura mais evidente neste
>> caso.
>>
>>
>> On Sun, Aug 16, 2020 at 10:11 AM Matheus Secco 
>> wrote:
>>
>>> Olá, Vanderlei.
>>> Por Cauchy-Schwarz, temos
>>>
>>> (a/ha + b/hb + c/hc) * (a*ha + b*hb + c*hc) >= (a+b+c)^2.  (#)
>>>
>>> Como (a*ha + b*hb + c*hc) = 2S, onde S é a área de ABC, segue que a
>>> expressão a/ha + b/hb + c/hc é pelo menos 2p^2/S, onde p é o
>>> semi-perimetro.
>>>
>>> Por outro lado, a igualdade em (#) ocorre se, e somente se, ha = hb =
>>> hc, ou seja, quando P é o incentro do triângulo
>>>
>>> Abraços,
>>> Matheus
>>>
>>> Em dom, 16 de ago de 2020 08:59, Professor Vanderlei Nemitz <
>>> vanderma...@gmail.com> escreveu:
>>>
>>>> Bom dia!
>>>>
>>>> Tentei utilizar alguma desigualdade de médias aqui, mas não tive êxito.
>>>> Alguém ajuda?
>>>> Muito agradecido!
>>>>
>>>> Seja P um ponto no interior de um triângulo e sejam ha, hb e hc as
>>>> distâncias de P aos lados a, b e c, respectivamente. Mostre que o valor
>>>> mínimo de (a/ha) + (b/hb) + (c/hc) ocorre quando P é o incentivo do
>>>> triângulo ABC.
>>>>
>>>> --
>>>> Esta mensagem foi verificada pelo sistema de antivírus e
>>>> acredita-se estar livre de perigo.
>>>
>>>
>>> --
>>> Esta mensagem foi verificada pelo sistema de antivírus e
>>> acredita-se estar livre de perigo.
>>
>>
>> --
>> Esta mensagem foi verificada pelo sistema de antivírus e
>> acredita-se estar livre de perigo.
>
>
> --
> Esta mensagem foi verificada pelo sistema de antivírus e
> acredita-se estar livre de perigo.

-- 
Esta mensagem foi verificada pelo sistema de antiv�rus e
 acredita-se estar livre de perigo.



[obm-l] Re: [obm-l] Re: [obm-l] Geometria plana com desigualdade de médias?

2020-08-18 Por tôpico Claudio Buffara
Será que tem uma demonstração mais geométrica e menos algébrica disso? E
que torne o resultado mais intuitivo?
É razoável que o ponto P não esteja muito próximo de qualquer dos lados,
pois neste caso, se P se aproximasse do lado a, por exemplo,
a/h_a cresceria e a expressão se afastaria do valor mínimo.
Mas, com lados não necessariamente congruentes, não é óbvio, a priori, que
P deva ser equidistante dos três.
De fato, seria razoável esperar que P estivesse mais próximo do maior lado
e conjecturar, por exemplo, que o P que minimiza a expressão é tal que
a/h_a = b/h_b = c/h_c.
O fato de P ser o incentro não me parece a conjectura mais evidente neste
caso.


On Sun, Aug 16, 2020 at 10:11 AM Matheus Secco 
wrote:

> Olá, Vanderlei.
> Por Cauchy-Schwarz, temos
>
> (a/ha + b/hb + c/hc) * (a*ha + b*hb + c*hc) >= (a+b+c)^2.  (#)
>
> Como (a*ha + b*hb + c*hc) = 2S, onde S é a área de ABC, segue que a
> expressão a/ha + b/hb + c/hc é pelo menos 2p^2/S, onde p é o
> semi-perimetro.
>
> Por outro lado, a igualdade em (#) ocorre se, e somente se, ha = hb = hc,
> ou seja, quando P é o incentro do triângulo
>
> Abraços,
> Matheus
>
> Em dom, 16 de ago de 2020 08:59, Professor Vanderlei Nemitz <
> vanderma...@gmail.com> escreveu:
>
>> Bom dia!
>>
>> Tentei utilizar alguma desigualdade de médias aqui, mas não tive êxito.
>> Alguém ajuda?
>> Muito agradecido!
>>
>> Seja P um ponto no interior de um triângulo e sejam ha, hb e hc as
>> distâncias de P aos lados a, b e c, respectivamente. Mostre que o valor
>> mínimo de (a/ha) + (b/hb) + (c/hc) ocorre quando P é o incentivo do
>> triângulo ABC.
>>
>> --
>> Esta mensagem foi verificada pelo sistema de antivírus e
>> acredita-se estar livre de perigo.
>
>
> --
> Esta mensagem foi verificada pelo sistema de antivírus e
> acredita-se estar livre de perigo.

-- 
Esta mensagem foi verificada pelo sistema de antiv�rus e
 acredita-se estar livre de perigo.



[obm-l] Re: [obm-l] Re: [obm-l] Re: [obm-l] Re: [obm-l] Re: [obm-l] Re: [obm-l] polinômio irredutível

2020-08-17 Por tôpico Claudio Buffara
Eu acho que o Eisenstein inventou este critério pra polinômios da forma
x^n + a ou, mais geralmente, pra polinômios ciclotômicos.
Daí funciona bem.

On Mon, Aug 17, 2020 at 11:02 AM Esdras Muniz 
wrote:

> E se p=3, e p divide N^2+9, então p^2 divide N^2+9.
>
> Então o critério de Eisenstein realmente não é tão abrangente. Será que
> tem algum outro critério que cubra casos em que o de Eisenstein não cubra?
>
> Em seg, 17 de ago de 2020 09:46, Claudio Buffara <
> claudio.buff...@gmail.com> escreveu:
>
>> Boa! Se p <> 3 mas p divide 3N e 3N^2, então p divide N ==> p não divide
>> N^3 + 9.
>>
>> On Sun, Aug 16, 2020 at 10:51 PM Esdras Muniz 
>> wrote:
>>
>>> Tenta com x^3+9.
>>>
>>> Em dom, 16 de ago de 2020 15:24, Claudio Buffara <
>>> claudio.buff...@gmail.com> escreveu:
>>>
>>>> f(x) em Z[x], bem entendido...
>>>>
>>>>
>>>> On Sun, Aug 16, 2020 at 3:08 PM Claudio Buffara <
>>>> claudio.buff...@gmail.com> wrote:
>>>>
>>>>> Que tal essa aqui?
>>>>> Prove ou disprove que, dado um polinômio f(x), irredutível sobre Q,
>>>>> existe um inteiro N tal que a irredutibilidade de f pode ser provada pelo
>>>>> critério de Eisenstein aplicado a f(x+N).
>>>>>
>>>>> On Sun, Aug 16, 2020 at 2:31 PM Matheus Secco 
>>>>> wrote:
>>>>>
>>>>>> O melhor jeito é pensar na contrapositiva (supondo que você esteja
>>>>>> falando sobre irredutibilidade em Z[x] ou até em Q[x]): se f(x) fatora 
>>>>>> como
>>>>>> g(x)*h(x), então f(x+a) fatora como g(x+a) *h(x+a) e é claro que uma vez
>>>>>> que g(x) e h(x) têm coeficientes inteiros, então g(x+a) e h(x+a) também
>>>>>> têm. A recíproca é essencialmente idêntica.
>>>>>>
>>>>>> Abraços
>>>>>>
>>>>>> Em dom, 16 de ago de 2020 14:11, Luís Lopes 
>>>>>> escreveu:
>>>>>>
>>>>>>> Sauda,c~oes,
>>>>>>>
>>>>>>> Como provar que um polinômio f(x) tendo como coeficientes números
>>>>>>> inteiros
>>>>>>> é irredutível se e somente se f(x+a) é irredutível para algum 
>>>>>>> inteiro ?
>>>>>>>
>>>>>>> Luís
>>>>>>>
>>>>>>>
>>>>>>>
>>>>>>>
>>>>>>> --
>>>>>>> Esta mensagem foi verificada pelo sistema de antivírus e
>>>>>>> acredita-se estar livre de perigo.
>>>>>>>
>>>>>>
>>>>>> --
>>>>>> Esta mensagem foi verificada pelo sistema de antivírus e
>>>>>> acredita-se estar livre de perigo.
>>>>>
>>>>>
>>>> --
>>>> Esta mensagem foi verificada pelo sistema de antivírus e
>>>> acredita-se estar livre de perigo.
>>>
>>>
>>> --
>>> Esta mensagem foi verificada pelo sistema de antivírus e
>>> acredita-se estar livre de perigo.
>>
>>
>> --
>> Esta mensagem foi verificada pelo sistema de antivírus e
>> acredita-se estar livre de perigo.
>
>
> --
> Esta mensagem foi verificada pelo sistema de antivírus e
> acredita-se estar livre de perigo.

-- 
Esta mensagem foi verificada pelo sistema de antiv�rus e
 acredita-se estar livre de perigo.



[obm-l] Re: [obm-l] Re: [obm-l] Re: [obm-l] Re: [obm-l] polinômio irredutível

2020-08-17 Por tôpico Claudio Buffara
Boa! Se p <> 3 mas p divide 3N e 3N^2, então p divide N ==> p não divide
N^3 + 9.

On Sun, Aug 16, 2020 at 10:51 PM Esdras Muniz 
wrote:

> Tenta com x^3+9.
>
> Em dom, 16 de ago de 2020 15:24, Claudio Buffara <
> claudio.buff...@gmail.com> escreveu:
>
>> f(x) em Z[x], bem entendido...
>>
>>
>> On Sun, Aug 16, 2020 at 3:08 PM Claudio Buffara <
>> claudio.buff...@gmail.com> wrote:
>>
>>> Que tal essa aqui?
>>> Prove ou disprove que, dado um polinômio f(x), irredutível sobre Q,
>>> existe um inteiro N tal que a irredutibilidade de f pode ser provada pelo
>>> critério de Eisenstein aplicado a f(x+N).
>>>
>>> On Sun, Aug 16, 2020 at 2:31 PM Matheus Secco 
>>> wrote:
>>>
>>>> O melhor jeito é pensar na contrapositiva (supondo que você esteja
>>>> falando sobre irredutibilidade em Z[x] ou até em Q[x]): se f(x) fatora como
>>>> g(x)*h(x), então f(x+a) fatora como g(x+a) *h(x+a) e é claro que uma vez
>>>> que g(x) e h(x) têm coeficientes inteiros, então g(x+a) e h(x+a) também
>>>> têm. A recíproca é essencialmente idêntica.
>>>>
>>>> Abraços
>>>>
>>>> Em dom, 16 de ago de 2020 14:11, Luís Lopes 
>>>> escreveu:
>>>>
>>>>> Sauda,c~oes,
>>>>>
>>>>> Como provar que um polinômio f(x) tendo como coeficientes números
>>>>> inteiros
>>>>> é irredutível se e somente se f(x+a) é irredutível para algum 
>>>>> inteiro ?
>>>>>
>>>>> Luís
>>>>>
>>>>>
>>>>>
>>>>>
>>>>> --
>>>>> Esta mensagem foi verificada pelo sistema de antivírus e
>>>>> acredita-se estar livre de perigo.
>>>>>
>>>>
>>>> --
>>>> Esta mensagem foi verificada pelo sistema de antivírus e
>>>> acredita-se estar livre de perigo.
>>>
>>>
>> --
>> Esta mensagem foi verificada pelo sistema de antivírus e
>> acredita-se estar livre de perigo.
>
>
> --
> Esta mensagem foi verificada pelo sistema de antivírus e
> acredita-se estar livre de perigo.

-- 
Esta mensagem foi verificada pelo sistema de antiv�rus e
 acredita-se estar livre de perigo.



[obm-l] Re: [obm-l] Re: [obm-l] polinômio irredutível

2020-08-16 Por tôpico Claudio Buffara
f(x) em Z[x], bem entendido...


On Sun, Aug 16, 2020 at 3:08 PM Claudio Buffara 
wrote:

> Que tal essa aqui?
> Prove ou disprove que, dado um polinômio f(x), irredutível sobre Q, existe
> um inteiro N tal que a irredutibilidade de f pode ser provada pelo critério
> de Eisenstein aplicado a f(x+N).
>
> On Sun, Aug 16, 2020 at 2:31 PM Matheus Secco 
> wrote:
>
>> O melhor jeito é pensar na contrapositiva (supondo que você esteja
>> falando sobre irredutibilidade em Z[x] ou até em Q[x]): se f(x) fatora como
>> g(x)*h(x), então f(x+a) fatora como g(x+a) *h(x+a) e é claro que uma vez
>> que g(x) e h(x) têm coeficientes inteiros, então g(x+a) e h(x+a) também
>> têm. A recíproca é essencialmente idêntica.
>>
>> Abraços
>>
>> Em dom, 16 de ago de 2020 14:11, Luís Lopes 
>> escreveu:
>>
>>> Sauda,c~oes,
>>>
>>> Como provar que um polinômio f(x) tendo como coeficientes números
>>> inteiros
>>> é irredutível se e somente se f(x+a) é irredutível para algum 
>>> inteiro ?
>>>
>>> Luís
>>>
>>>
>>>
>>>
>>> --
>>> Esta mensagem foi verificada pelo sistema de antivírus e
>>> acredita-se estar livre de perigo.
>>>
>>
>> --
>> Esta mensagem foi verificada pelo sistema de antivírus e
>> acredita-se estar livre de perigo.
>
>

-- 
Esta mensagem foi verificada pelo sistema de antiv�rus e
 acredita-se estar livre de perigo.



[obm-l] Re: [obm-l] Re: [obm-l] polinômio irredutível

2020-08-16 Por tôpico Claudio Buffara
Que tal essa aqui?
Prove ou disprove que, dado um polinômio f(x), irredutível sobre Q, existe
um inteiro N tal que a irredutibilidade de f pode ser provada pelo critério
de Eisenstein aplicado a f(x+N).

On Sun, Aug 16, 2020 at 2:31 PM Matheus Secco 
wrote:

> O melhor jeito é pensar na contrapositiva (supondo que você esteja falando
> sobre irredutibilidade em Z[x] ou até em Q[x]): se f(x) fatora como
> g(x)*h(x), então f(x+a) fatora como g(x+a) *h(x+a) e é claro que uma vez
> que g(x) e h(x) têm coeficientes inteiros, então g(x+a) e h(x+a) também
> têm. A recíproca é essencialmente idêntica.
>
> Abraços
>
> Em dom, 16 de ago de 2020 14:11, Luís Lopes 
> escreveu:
>
>> Sauda,c~oes,
>>
>> Como provar que um polinômio f(x) tendo como coeficientes números
>> inteiros
>> é irredutível se e somente se f(x+a) é irredutível para algum  inteiro
>> ?
>>
>> Luís
>>
>>
>>
>>
>> --
>> Esta mensagem foi verificada pelo sistema de antivírus e
>> acredita-se estar livre de perigo.
>>
>
> --
> Esta mensagem foi verificada pelo sistema de antivírus e
> acredita-se estar livre de perigo.

-- 
Esta mensagem foi verificada pelo sistema de antiv�rus e
 acredita-se estar livre de perigo.



[obm-l] Re: [obm-l] Funções complexas - número de zeros em C

2020-07-30 Por tôpico Claudio Buffara
Será que fazendo w = 1/z  e  w -> 0 ajuda?

On Thu, Jul 30, 2020 at 7:24 AM Artur Costa Steiner <
artur.costa.stei...@gmail.com> wrote:

> Sejam f e g funções inteiras tais que lim |z| ---> oo f(z)/g(z) = 1.
> Mostre que f e g tem um número finito de zeros em C e que o número de zeros
> de f é igual ao número de zeros de  g.
>
> Abs
>
> Artur
>
> --
> Esta mensagem foi verificada pelo sistema de antivírus e
> acredita-se estar livre de perigo.

-- 
Esta mensagem foi verificada pelo sistema de antiv�rus e
 acredita-se estar livre de perigo.



Re: [obm-l] Desafio de probabilidade

2020-07-25 Por tôpico Claudio Buffara
É isso mesmo. Tem que sair 3 vezes o MESMO NÚMERO e não 3 vezes a MESMA
PARIDADE.

[]s,
Claudio.

On Sat, Jul 25, 2020 at 3:53 PM Ralph Costa Teixeira 
wrote:

> Oi, Claudio
>
> Eu também pensei em trocar o dado por uma moeda, mas se entendi bem o
> enunciado, não podemos! O problema eh que, se o dado der 2,4,6,2,4,6,1,1,1,
> quem ganha eh Umberto; trocando pela moeda, vemos par,par,par e vamos dar o
> trofeu para o Ze Roberto... Muda o jogo!
>
> On Sat, Jul 25, 2020 at 3:24 PM Claudio Buffara 
> wrote:
>
>> Pra facilitar, podemos substituir o dado por uma moeda, com cara = par =
>> 0 e coroa = ímpar = 1, já que o que importa é apenas a paridade do número
>> na face superior do dado lançado e, neste caso, P(par) = P(ímpar) = 1/2.
>>
>> Como 3 caras seguidas ou 3 coroas seguidas encerra o jogo, basta
>> considerar os dois últimos lançamentos.
>>
>> Suponha que dois lançamentos seguidos tenham sido 1 e 0 (cara e coroa).
>> Após sair o 0, digamos que a probabilidade de ZR vencer seja p.
>>
>> Se o terceiro lançamento for 0, a probabilidade de ZR vencer aumentará
>> para q  (p e q são incógnitas a serem determinadas), e q é justamente a
>> probabilidade desejada, já que é a probabilidade de ZR vencer dado que os
>> dois últimos lançamentos foram 0 e 0.
>> Se o quarto lançamento for 0, ZR vence. Mas se for 1, sua probabilidade
>> de vencer cai para 1-p pois, neste caso, por simetria, Umberto passa a ter
>> probabilidade p de vencer, em virtude dos dois últimos lançamentos terem
>> sido 0 e 1.
>> Ou seja, q = (1/2)*(1 + (1-p))  <==>  p + 2q = 2.
>>
>> Se o terceiro lançamento for 1, a probabilidade de ZR vencer cai para 1-p.
>> Neste caso, podemos escrever p = (1/2)*(q + (1-p))  <==>  3p - q = 1.
>>
>> Resolvendo este sistema, achamos p = 4/7 e q = 5/7.
>>
>> Na verdade, isso tudo fica mais fácil de ver se você fizer uma árvore.
>>
>> []s,
>> Claudio.
>>
>> On Sat, Jul 25, 2020 at 2:03 PM Professor Vanderlei Nemitz <
>> vanderma...@gmail.com> wrote:
>>
>>> Então meu raciocínio foi muito errado, pois pensei assim:
>>> Seja p a probabilidade de Zé Roberto vender. Podemos considerar que o
>>> jogo "começa" com Zé Roberto precisando obter um 6 para vencer.
>>> Assim, a probabilidade de Humberto vencer é:
>>> q = (3/6).(1/6).p, ou seja, p = 12q
>>> Assim, p = 12/13 e q = 1/13
>>>
>>> Prezado Cláudio, você pode explicar sua resolução?
>>>
>>> Muito obrigado!
>>>
>>>
>>>
>>>
>>>
>>>
>>> Em sáb., 25 de jul. de 2020 às 13:43, Claudio Buffara <
>>> claudio.buff...@gmail.com> escreveu:
>>>
>>>> Eu achei 5/7.
>>>>
>>>> On Sat, Jul 25, 2020 at 7:28 AM Professor Vanderlei Nemitz <
>>>> vanderma...@gmail.com> wrote:
>>>>
>>>>> Bom dia!
>>>>> O problema a seguir encontra-se em uma prova de desafios da PUC-RJ,
>>>>> muito boas!!!
>>>>> Acho que são organizadas pelo professor Nicolau Saldanha.
>>>>> Encontrei uma resposta bem alta, mais de 90%. Será que está correto?
>>>>> Muito obrigado!
>>>>>
>>>>> Zé Roberto e Umberto gostam de jogar par ou ímpar; Zé Roberto sempre
>>>>> pede par e Umberto sempre pede íımpar. Eles gostam de inventar novas
>>>>> maneiras de jogar. A última maneira que eles inventaram usa um dado comum,
>>>>> com seis faces numeradas de 1 a 6. Eles jogam o dado várias vezes até que
>>>>> um número saia três vezes seguidas; Zé Roberto ganha se este número for
>>>>> par, Umberto ganha se for ímpar. Sábado de manhã o dado teve os 
>>>>> resultados:
>>>>> 5, 3, 4, 2, 6, 1, 1, 3, 1, 4, 2, 3, 5, 6, 3, 4, 5, 4, 4, 4 e neste ponto 
>>>>> Zé
>>>>> Roberto se declarou vitorioso. Sábado de tarde o dado teve os resultados:
>>>>> 6, 1, 4, 2, 3, 5, 6, 6; neste momento o jogo foi interrompido pela queda 
>>>>> de
>>>>> um meteorito. Quando a situação se acalmou, eles concordaram em continuar
>>>>> do ponto em que estavam. Qual é a probabilidade de que Zé Roberto seja o
>>>>> vencedor?
>>>>>
>>>>> --
>>>>> Esta mensagem foi verificada pelo sistema de antivírus e
>>>>> acredita-se estar livre de perigo.
>>>>
>>>>
>>>> --
>>>> Esta mensagem foi verificada pelo sistema de antivírus e
>>>> acredita-se estar livre de perigo.
>>>
>>>
>>> --
>>> Esta mensagem foi verificada pelo sistema de antivírus e
>>> acredita-se estar livre de perigo.
>>
>>
>> --
>> Esta mensagem foi verificada pelo sistema de antivírus e
>> acredita-se estar livre de perigo.
>
>
> --
> Esta mensagem foi verificada pelo sistema de antivírus e
> acredita-se estar livre de perigo.

-- 
Esta mensagem foi verificada pelo sistema de antiv�rus e
 acredita-se estar livre de perigo.



Re: [obm-l] Desafio de probabilidade

2020-07-25 Por tôpico Claudio Buffara
Por favor desconsiderem.
Reli o enunciado e vi que errei.
Pro ZR ganhar, tem que sair o mesmo número par 3 vezes seguidas.
E minha solução é para o caso (bem mais fácil!) em que ele ganha se saírem
3 números pares seguidos.

[]s,
Claudio.


On Sat, Jul 25, 2020 at 3:08 PM Claudio Buffara 
wrote:

> Pra facilitar, podemos substituir o dado por uma moeda, com cara = par = 0
> e coroa = ímpar = 1, já que o que importa é apenas a paridade do número na
> face superior do dado lançado e, neste caso, P(par) = P(ímpar) = 1/2.
>
> Como 3 caras seguidas ou 3 coroas seguidas encerra o jogo, basta
> considerar os dois últimos lançamentos.
>
> Suponha que dois lançamentos seguidos tenham sido 1 e 0 (cara e coroa).
> Após sair o 0, digamos que a probabilidade de ZR vencer seja p.
>
> Se o terceiro lançamento for 0, a probabilidade de ZR vencer aumentará
> para q  (p e q são incógnitas a serem determinadas), e q é justamente a
> probabilidade desejada, já que é a probabilidade de ZR vencer dado que os
> dois últimos lançamentos foram 0 e 0.
> Se o quarto lançamento for 0, ZR vence. Mas se for 1, sua probabilidade de
> vencer cai para 1-p pois, neste caso, por simetria, Umberto passa a ter
> probabilidade p de vencer, em virtude dos dois últimos lançamentos terem
> sido 0 e 1.
> Ou seja, q = (1/2)*(1 + (1-p))  <==>  p + 2q = 2.
>
> Se o terceiro lançamento for 1, a probabilidade de ZR vencer cai para 1-p.
> Neste caso, podemos escrever p = (1/2)*(q + (1-p))  <==>  3p - q = 1.
>
> Resolvendo este sistema, achamos p = 4/7 e q = 5/7.
>
> Na verdade, isso tudo fica mais fácil de ver se você fizer uma árvore.
>
> []s,
> Claudio.
>
> On Sat, Jul 25, 2020 at 2:03 PM Professor Vanderlei Nemitz <
> vanderma...@gmail.com> wrote:
>
>> Então meu raciocínio foi muito errado, pois pensei assim:
>> Seja p a probabilidade de Zé Roberto vender. Podemos considerar que o
>> jogo "começa" com Zé Roberto precisando obter um 6 para vencer.
>> Assim, a probabilidade de Humberto vencer é:
>> q = (3/6).(1/6).p, ou seja, p = 12q
>> Assim, p = 12/13 e q = 1/13
>>
>> Prezado Cláudio, você pode explicar sua resolução?
>>
>> Muito obrigado!
>>
>>
>>
>>
>>
>>
>> Em sáb., 25 de jul. de 2020 às 13:43, Claudio Buffara <
>> claudio.buff...@gmail.com> escreveu:
>>
>>> Eu achei 5/7.
>>>
>>> On Sat, Jul 25, 2020 at 7:28 AM Professor Vanderlei Nemitz <
>>> vanderma...@gmail.com> wrote:
>>>
>>>> Bom dia!
>>>> O problema a seguir encontra-se em uma prova de desafios da PUC-RJ,
>>>> muito boas!!!
>>>> Acho que são organizadas pelo professor Nicolau Saldanha.
>>>> Encontrei uma resposta bem alta, mais de 90%. Será que está correto?
>>>> Muito obrigado!
>>>>
>>>> Zé Roberto e Umberto gostam de jogar par ou ímpar; Zé Roberto sempre
>>>> pede par e Umberto sempre pede íımpar. Eles gostam de inventar novas
>>>> maneiras de jogar. A última maneira que eles inventaram usa um dado comum,
>>>> com seis faces numeradas de 1 a 6. Eles jogam o dado várias vezes até que
>>>> um número saia três vezes seguidas; Zé Roberto ganha se este número for
>>>> par, Umberto ganha se for ímpar. Sábado de manhã o dado teve os resultados:
>>>> 5, 3, 4, 2, 6, 1, 1, 3, 1, 4, 2, 3, 5, 6, 3, 4, 5, 4, 4, 4 e neste ponto Zé
>>>> Roberto se declarou vitorioso. Sábado de tarde o dado teve os resultados:
>>>> 6, 1, 4, 2, 3, 5, 6, 6; neste momento o jogo foi interrompido pela queda de
>>>> um meteorito. Quando a situação se acalmou, eles concordaram em continuar
>>>> do ponto em que estavam. Qual é a probabilidade de que Zé Roberto seja o
>>>> vencedor?
>>>>
>>>> --
>>>> Esta mensagem foi verificada pelo sistema de antivírus e
>>>> acredita-se estar livre de perigo.
>>>
>>>
>>> --
>>> Esta mensagem foi verificada pelo sistema de antivírus e
>>> acredita-se estar livre de perigo.
>>
>>
>> --
>> Esta mensagem foi verificada pelo sistema de antivírus e
>> acredita-se estar livre de perigo.
>
>

-- 
Esta mensagem foi verificada pelo sistema de antiv�rus e
 acredita-se estar livre de perigo.



Re: [obm-l] Desafio de probabilidade

2020-07-25 Por tôpico Claudio Buffara
Pra facilitar, podemos substituir o dado por uma moeda, com cara = par = 0
e coroa = ímpar = 1, já que o que importa é apenas a paridade do número na
face superior do dado lançado e, neste caso, P(par) = P(ímpar) = 1/2.

Como 3 caras seguidas ou 3 coroas seguidas encerra o jogo, basta considerar
os dois últimos lançamentos.

Suponha que dois lançamentos seguidos tenham sido 1 e 0 (cara e coroa).
Após sair o 0, digamos que a probabilidade de ZR vencer seja p.

Se o terceiro lançamento for 0, a probabilidade de ZR vencer aumentará para
q  (p e q são incógnitas a serem determinadas), e q é justamente a
probabilidade desejada, já que é a probabilidade de ZR vencer dado que os
dois últimos lançamentos foram 0 e 0.
Se o quarto lançamento for 0, ZR vence. Mas se for 1, sua probabilidade de
vencer cai para 1-p pois, neste caso, por simetria, Umberto passa a ter
probabilidade p de vencer, em virtude dos dois últimos lançamentos terem
sido 0 e 1.
Ou seja, q = (1/2)*(1 + (1-p))  <==>  p + 2q = 2.

Se o terceiro lançamento for 1, a probabilidade de ZR vencer cai para 1-p.
Neste caso, podemos escrever p = (1/2)*(q + (1-p))  <==>  3p - q = 1.

Resolvendo este sistema, achamos p = 4/7 e q = 5/7.

Na verdade, isso tudo fica mais fácil de ver se você fizer uma árvore.

[]s,
Claudio.

On Sat, Jul 25, 2020 at 2:03 PM Professor Vanderlei Nemitz <
vanderma...@gmail.com> wrote:

> Então meu raciocínio foi muito errado, pois pensei assim:
> Seja p a probabilidade de Zé Roberto vender. Podemos considerar que o jogo
> "começa" com Zé Roberto precisando obter um 6 para vencer.
> Assim, a probabilidade de Humberto vencer é:
> q = (3/6).(1/6).p, ou seja, p = 12q
> Assim, p = 12/13 e q = 1/13
>
> Prezado Cláudio, você pode explicar sua resolução?
>
> Muito obrigado!
>
>
>
>
>
>
> Em sáb., 25 de jul. de 2020 às 13:43, Claudio Buffara <
> claudio.buff...@gmail.com> escreveu:
>
>> Eu achei 5/7.
>>
>> On Sat, Jul 25, 2020 at 7:28 AM Professor Vanderlei Nemitz <
>> vanderma...@gmail.com> wrote:
>>
>>> Bom dia!
>>> O problema a seguir encontra-se em uma prova de desafios da PUC-RJ,
>>> muito boas!!!
>>> Acho que são organizadas pelo professor Nicolau Saldanha.
>>> Encontrei uma resposta bem alta, mais de 90%. Será que está correto?
>>> Muito obrigado!
>>>
>>> Zé Roberto e Umberto gostam de jogar par ou ímpar; Zé Roberto sempre
>>> pede par e Umberto sempre pede íımpar. Eles gostam de inventar novas
>>> maneiras de jogar. A última maneira que eles inventaram usa um dado comum,
>>> com seis faces numeradas de 1 a 6. Eles jogam o dado várias vezes até que
>>> um número saia três vezes seguidas; Zé Roberto ganha se este número for
>>> par, Umberto ganha se for ímpar. Sábado de manhã o dado teve os resultados:
>>> 5, 3, 4, 2, 6, 1, 1, 3, 1, 4, 2, 3, 5, 6, 3, 4, 5, 4, 4, 4 e neste ponto Zé
>>> Roberto se declarou vitorioso. Sábado de tarde o dado teve os resultados:
>>> 6, 1, 4, 2, 3, 5, 6, 6; neste momento o jogo foi interrompido pela queda de
>>> um meteorito. Quando a situação se acalmou, eles concordaram em continuar
>>> do ponto em que estavam. Qual é a probabilidade de que Zé Roberto seja o
>>> vencedor?
>>>
>>> --
>>> Esta mensagem foi verificada pelo sistema de antivírus e
>>> acredita-se estar livre de perigo.
>>
>>
>> --
>> Esta mensagem foi verificada pelo sistema de antivírus e
>> acredita-se estar livre de perigo.
>
>
> --
> Esta mensagem foi verificada pelo sistema de antivírus e
> acredita-se estar livre de perigo.

-- 
Esta mensagem foi verificada pelo sistema de antiv�rus e
 acredita-se estar livre de perigo.



Re: [obm-l] Desafio de probabilidade

2020-07-25 Por tôpico Claudio Buffara
Eu achei 5/7.

On Sat, Jul 25, 2020 at 7:28 AM Professor Vanderlei Nemitz <
vanderma...@gmail.com> wrote:

> Bom dia!
> O problema a seguir encontra-se em uma prova de desafios da PUC-RJ, muito
> boas!!!
> Acho que são organizadas pelo professor Nicolau Saldanha.
> Encontrei uma resposta bem alta, mais de 90%. Será que está correto?
> Muito obrigado!
>
> Zé Roberto e Umberto gostam de jogar par ou ímpar; Zé Roberto sempre pede
> par e Umberto sempre pede íımpar. Eles gostam de inventar novas maneiras de
> jogar. A última maneira que eles inventaram usa um dado comum, com seis
> faces numeradas de 1 a 6. Eles jogam o dado várias vezes até que um número
> saia três vezes seguidas; Zé Roberto ganha se este número for par, Umberto
> ganha se for ímpar. Sábado de manhã o dado teve os resultados: 5, 3, 4, 2,
> 6, 1, 1, 3, 1, 4, 2, 3, 5, 6, 3, 4, 5, 4, 4, 4 e neste ponto Zé Roberto se
> declarou vitorioso. Sábado de tarde o dado teve os resultados: 6, 1, 4, 2,
> 3, 5, 6, 6; neste momento o jogo foi interrompido pela queda de um
> meteorito. Quando a situação se acalmou, eles concordaram em continuar do
> ponto em que estavam. Qual é a probabilidade de que Zé Roberto seja o
> vencedor?
>
> --
> Esta mensagem foi verificada pelo sistema de antivírus e
> acredita-se estar livre de perigo.

-- 
Esta mensagem foi verificada pelo sistema de antiv�rus e
 acredita-se estar livre de perigo.



[obm-l] Re: [obm-l] Ajuda em teoria dos números

2020-07-24 Por tôpico Claudio Buffara
Pelo que entendi, a solução é a porção dessa curva algébrica situada no 1o
quadrante.
Dá pra fazer isso no Wolfram Alpha, com o comando plot (x*y-7)^2 - x^2 -
y^2 = 0.

[]s,
Claudio.

On Fri, Jul 24, 2020 at 9:58 AM Prof. Douglas Oliveira <
profdouglaso.del...@gmail.com> wrote:

> Preciso de ajuda para encontrar todas as soluções não negativas da equação
> (xy-7)^2=x^2+y^2.
>
> Desde já agradeço a ajuda
> Douglas Oliveira
>
> --
> Esta mensagem foi verificada pelo sistema de antivírus e
> acredita-se estar livre de perigo.

-- 
Esta mensagem foi verificada pelo sistema de antiv�rus e
 acredita-se estar livre de perigo.



[obm-l] Re: [obm-l] Números complexos e equações

2020-06-17 Por tôpico Claudio Buffara
Aquele 1+i sugere que se forme uma equação em z, onde z = (1+i)/raiz(2) *
x, ou seja, cujas raízes sejam as da equação original giradas de 45 graus
no sentido anti-horário e sem coeficientes complexos.
z = (1+i)/raiz(2) * x ==> x = (1-i)/raiz(2) * z
Assim, x^4 + 4(1+i)x + 1 = 0 ==> -z^4 + 4*raiz(2)*z + 1 = 0
Consideremos f(z) = z^4 - 4*raiz(2)*z - 1  (multiplicar os coeficientes por
-1 não altera as raízes).
f(-1) = 4*raiz(2) > 0
f(0) = -1 < 0
f(raiz(2)) = -5 < 0
f(2) =15 - 8*raiz(2) > 0 ==> f tem (pelo menos) duas raízes reais: uma
entre -1 e 0 e outra entre raiz(2) e 2.
Mas f'(z) = 4z^3 - 4*raiz(2) ==> f'(z) < 0 para z < 2^(1/6) (logo, para z <
0) e f'(z) > 0 para z > 2^(1/6) (logo, para z > raiz(2)), de modo que estas
são as únicas raízes reais de f.
Se duas das raízes da equação original, ao serem giradas de 45 graus no
sentido anti-horário, se tornam reais, então aquelas raízes estavam na reta
Im(z) = -Re(z).
Além disso, como, após giradas, uma se tornou negativa e a outra positiva,
isso significa que a primeira está no 2o quadrante e a segunda no 4o
quadrante.

Dadas as magnitudes das raízes giradas (a primeira entre -1 e 0 e a segunda
maior do que raiz(2)), também concluímos que a soma delas está no 4o
quadrante, ou seja, é da forma p*(1-i), com p > 0.
Além disso, o produto delas é da forma (1/q)*i, com q > 0.

Chame as outras duas raízes da equação original de a e b.
Então, como a soma das raízes é zero, vale a+b = -p(1-i) = p(-1+i): um
ponto do 2o quadrante sobre a reta Im(z) = -Re(z)   (1)
Como o produto das raízes é 1, vale a*b = -q*i, um ponto do eixo imaginário
negativo
A localização do produto a*b implica que a/|a| e b/|b| são números
complexos (de módulo unitário) e simétricos em relação à reta Im(z) =
-Re(z)   (2)
(1) e (2) implicam que a e b têm o mesmo módulo R

(2) também implica que, sobre a e b:
OU ambos pertencem ao 2o quadrante
OU um deles pertence ao 1o e o outro ao 3o quadrante
OU ambos pertencem ao 4o quadrante.

De cara dá pra eliminar a última alternativa, já que isso implicaria que
a+b pertence ao 4o quadrante, o que não é o caso.

Resta eliminar a 1a alternativa.
Assim, suponhamos que a e b pertencem ao 2o quadrante.

Neste caso, a+b = p(-1+i) ==> |a+b| = p*raiz(2) > R*raiz(2) ==> 2*p^2 >
2*R^2
E também, de qualquer jeito, ab = -qi ==> q = R^2

Da equação, também sabemos que ab + ac + ad + bc + bd + cd = 0 ==>
ab + cd + (a+b)(c+d) = 0 ==>
-q*i + (1/q)*i + p(-1+i)*p*(1-i) = 0 ==>
1/q - q + 2p^2 = 0
1/q - q + 2R^2 < 0 ==>
1/R^2 - R^2 + 2*R^2 < 1/R^2 + < 0 ==> contradição ==> a e b não pertencem
ao 2o quadrante.

Logo, temos que concluir que, sobre as outras duas raízes, que uma pertence
ao 1o e a outra ao 3o quadrante.

[]s,
Claudio.


On Wed, Jun 17, 2020 at 9:01 AM Prof. Douglas Oliveira <
profdouglaso.del...@gmail.com> wrote:

> Olá, gostaria de uma ajuda para localizar as raízes da
> equação x^4+4(1+i)x+1=0, saber em qual quadrante estão, joguei no MAPLE e
> percebi que existe uma em cada quadrante.
>
> Mas não consigo achar uma saída.
>
> Obrigado.
> Douglas Oliveira
>
> --
> Esta mensagem foi verificada pelo sistema de antivírus e
> acredita-se estar livre de perigo.

-- 
Esta mensagem foi verificada pelo sistema de antiv�rus e
 acredita-se estar livre de perigo.



Re: [obm-l] construção geométrica

2020-06-10 Por tôpico Claudio Buffara
Se o triângulo for equilátero, qualquer ponto do arco AB serve.

Enviado do meu iPhone

> Em 10 de jun de 2020, à(s) 17:24, Luís Lopes  escreveu:
> 

-- 
Esta mensagem foi verificada pelo sistema de antiv�rus e
 acredita-se estar livre de perigo.


=
Instru��es para entrar na lista, sair da lista e usar a lista em
http://www.mat.puc-rio.br/~obmlistas/obm-l.html
=


Re: [obm-l] site enem matemática

2020-06-05 Por tôpico Claudio Buffara
Pra mim, a melhor forma de se preparar é baixar as provas passadas do site do 
INEP e resolver as questões. Se vc resolver as provas dos últimos 5 ou 6 anos, 
estará bem preparado.

Se empacar em alguma questão, poste a dúvida aqui que alguém poderá responder 
(apesar deste ser um grupo de olimpíadas, pelo menos em tese).

O YouTube deve ter vídeos com soluções de todas as questões dos últimos anos.

E se você quiser a teoria também há sites pra isso. O maior deles é o 
Descomplica.

Abs

> Em 5 de jun de 2020, à(s) 09:41, carlos h Souza  
> escreveu:
> 
> 
> Bom dia,
> 
> Alguém sabe de algum site para treinamento de questões do ENEM, só de 
> matemática e raciocínio lógico ???
> 
> Obrigado
> 
> -- 
> Esta mensagem foi verificada pelo sistema de antivírus e 
> acredita-se estar livre de perigo.

-- 
Esta mensagem foi verificada pelo sistema de antiv�rus e
 acredita-se estar livre de perigo.


=
Instru��es para entrar na lista, sair da lista e usar a lista em
http://www.mat.puc-rio.br/~obmlistas/obm-l.html
=


[obm-l] Re: [obm-l] Cálculo do Volume de um Sólido

2020-02-11 Por tôpico Claudio Buffara
O sólido é a região do 1o octante (todas as coordenadas positivas)
compreendida entre os planos x-z e y-z, acima do plano z = (x+y)/2 e abaixo
da z = raiz(x+y).
A superfície e o plano se intersectam numa reta:
raiz(x+y) = (x+y)/2 ==> x+y = (x+y)^2/4 ==> x+y = 4, contida no plano z = 2.

Assim, o volume pode ser dado pela diferença entre duas integrais duplas,
calculadas sobre o domínio D, no plano x-y, dado por x > 0, y > 0 e x+y = 4.
Volume = Integral(D) raiz(x+y)*dA - Integral(D) (x+y)/2*dA.

Usando coordenadas cartesianas, a primeira integral fica:
Integral(x=0...4)Integral(y=0...4-x)*raiz(x+y)*dy*dx
= Integral(0...4) (2/3)*(4^(3/2) - x^(3/2))*dx
= Integral(0...4) (16/3 - (2/3)*x^(3/2))
= 64/3 - (4/15)*4^(5/2)
= 64/3 - 128/15
= 64/5

A segunda integral é:
Integral(x=0...4)Integral(y=0...4-x) (x+y)/2*dy*dx
= Integral(x=0...4) (1/2)*(x*(4-x) + (4-x)^2/2)*dx
= Integral(0...4) (4 - x^2/4)*dx
= 32/3

Logo, o volume é 64/5 - 32/3 = 32/15  (se não errei nenhuma conta...)

[]s,
Claudio.


On Mon, Feb 3, 2020 at 8:55 PM Luiz Antonio Rodrigues 
wrote:

> Olá, pessoal!
> Tudo bem?
> Estou tentando resolver o seguinte problema:
>
> Ache o volume da região tridimensional definida por:
>
> z^2
> Sendo que:
> x>0 e y>0 e z>0
>
> Com o auxílio de um software eu consegui visualizar o sólido em questão.
> Eu calculei o volume do sólido girando em torno do eixo z e dividindo o
> resultado por 4.
> A resposta que eu obtive foi (16*pi)/15, que não está correta.
> Já refiz os cálculos muitas vezes e chego sempre na mesma resposta.
> Alguém pode me ajudar?
> Muito obrigado e um abraço!
>
> --
> Esta mensagem foi verificada pelo sistema de antivírus e
> acredita-se estar livre de perigo.

-- 
Esta mensagem foi verificada pelo sistema de antiv�rus e
 acredita-se estar livre de perigo.



Re: [obm-l] Uma soma

2020-01-16 Por tôpico Claudio Buffara
O termo geral é k*(n+1-k), com k variando de 1 a n

Enviado do meu iPhone

> Em 16 de jan de 2020, à(s) 17:27, Claudio Buffara  
> escreveu:
> 
> Faz uma tabela
> 1
> 1   2
> 1   2   3
> 1   2   3  4
> 
> 4*1 + 3*2 + 2*3 + 1*4
> 
> Deu pra pegar o padrão?
> 
> Enviado do meu iPhone
> 
>> Em 16 de jan de 2020, à(s) 16:13, marcone augusto araújo borges 
>>  escreveu:
>> 
>>  Como calcular 1 + (1+2) + (1+2+3) +... +(1+2+...+n)? 
>> -- 
>> Esta mensagem foi verificada pelo sistema de antivírus e 
>> acredita-se estar livre de perigo.

-- 
Esta mensagem foi verificada pelo sistema de antiv�rus e
 acredita-se estar livre de perigo.


=
Instru��es para entrar na lista, sair da lista e usar a lista em
http://www.mat.puc-rio.br/~obmlistas/obm-l.html
=


Re: [obm-l] Uma soma

2020-01-16 Por tôpico Claudio Buffara
Faz uma tabela
1
1   2
1   2   3
1   2   3  4

4*1 + 3*2 + 2*3 + 1*4

Deu pra pegar o padrão?

Enviado do meu iPhone

> Em 16 de jan de 2020, à(s) 16:13, marcone augusto araújo borges 
>  escreveu:
> 
>  Como calcular 1 + (1+2) + (1+2+3) +... +(1+2+...+n)? 
> -- 
> Esta mensagem foi verificada pelo sistema de antivírus e 
> acredita-se estar livre de perigo.

-- 
Esta mensagem foi verificada pelo sistema de antiv�rus e
 acredita-se estar livre de perigo.


=
Instru��es para entrar na lista, sair da lista e usar a lista em
http://www.mat.puc-rio.br/~obmlistas/obm-l.html
=


Re: [obm-l] Soma de Riemann

2020-01-14 Por tôpico Claudio Buffara
Os livros são estes mesmo.

O artigo é esse aqui:
https://epocanegocios.globo.com/Informacao/Dilemas/noticia/2014/12/elas-precisam-de-reengenharia.html

É de 2014, mas ino que a situação não tenha mudado muito de lá pra cá.

[]s,
Claudio.


On Tue, Jan 14, 2020 at 7:09 PM Luiz Antonio Rodrigues <
rodrigue...@gmail.com> wrote:

> Olá, Claudio!
> Tudo bem?
> Muito obrigado pelas sugestões.
> Eu vi na Amazon os títulos:
>
> A Problem Book in Algebra - Krechmar
>
> Problems in Higher Algebra - Faddeev & Sominskii
>
> São esses?
> O que você disse é verdade, muitas vezes eu recorro aos softwares para
> verificar minhas respostas.
> Eu gostaria bastante de ler o artigo que você citou.
> Muito obrigado!
> Abs.
>
> Em ter, 14 de jan de 2020 5:01 PM, Claudio Buffara <
> claudio.buff...@gmail.com> escreveu:
>
>> Estas somas trigonométricas (e várias outras) são obtidas sem grandes
>> dificuldades, mas com alguma álgebra, usando números complexos.
>>
>> O melhor caminho, a meu ver, seria vc conseguir um daqueles livros russos
>> clássicos - Krechmar ou Faddev-Sominski - que contém coletâneas de
>> problemas resolvidos sobre este tema e muitos outros.
>>
>> Agora, me parece que a habilidade de computar estas somas “na mão”,
>> usando complexos e/ou alguns truques algébricos (ou até mesmo integrais)
>> tem se desvalorizado recentemente devido à existência e ampla
>> disponibilidade de softwares gratuitos tais como o Wolfram Alpha, que
>> calculam qualquer soma dessas.
>>
>> Recentemente li um artigo que toca um pouco neste tema, da necessidade de
>> modernização dos cursos de exatas. Vou procurar e postarei aqui.
>>
>> Abs
>>
>>
>> Enviado do meu iPhone
>>
>> Em 14 de jan de 2020, à(s) 12:07, Luiz Antonio Rodrigues <
>> rodrigue...@gmail.com> escreveu:
>>
>> 
>> Olá, Artur!
>> Tudo bem?
>> Agradeço sua resposta.
>> O problema diz:
>>
>> É dado o somatório de:
>>
>> sen(k*b/n)
>>
>> Onde k varia de 1 até n.
>>
>> Calcule o limite deste somatório dividido por n, quando n tende a
>> infinito.
>>
>> O problema pede que se relacione este limite com uma soma de Riemann.
>>
>> Seguindo a sugestão do Claudio, calculei o somatório dos senos em P.A.
>> Depois eu calculei o limite solicitado.
>> Cheguei n mesma resposta do Claudio, que está correta.
>> Aproveito para pedir uma indicação de material sobre este assunto, que
>> considero bastante interessante.
>> Muito obrigado!
>> Luiz
>>
>>
>> Em ter, 14 de jan de 2020 1:32 AM, Artur Costa Steiner <
>> artur.costa.stei...@gmail.com> escreveu:
>>
>>> Este somatório não é uma soma de Riemann. Seria se fosse
>>>
>>> S(n) = 1/n Soma(k = 1, n) sen(k*b/n).Â
>>>
>>> Mas é S(n) = Soma(k = 1, n) sen(k*b/n). Não se divide por n.
>>>
>>> Tem ceteza de que pelo outro processo vc chegou no seu somatório Ã
>>> expressão correspondente ao caso da soma de Riemann?
>>>
>>> Se fizermos b = pi/2, no seu somatorio temos para todo n que S(n) >
>>> sen(pi/2) = 1.Logo, se o limite com n indo para oo existir, será >= 1.Mas
>>> entrando com b = pi/2 na fórmula da soma de Riemann, obtemos 2/pi < 1.
>>>
>>> Me corrija se eu tiver cometido algum erro.
>>>
>>> Abraços
>>>
>>> Artur
>>>
>>> Em seg, 13 de jan de 2020 18:04, Luiz Antonio Rodrigues <
>>> rodrigue...@gmail.com> escreveu:
>>>
>>>> Olá, Claudio!
>>>> Tudo bem?
>>>> Sim, foi esse resultado que eu achei!
>>>> Muito obrigado pela ajuda!
>>>>
>>>> Em seg, 13 de jan de 2020 5:02 PM, Claudio Buffara <
>>>> claudio.buff...@gmail.com> escreveu:
>>>>
>>>>> É a soma de n retângulos, todos com base 1/n e o k-esimo com altura
>>>>> sen(kb/n): logo, o limite e’ a integral superior (portanto, a integral
>>>>> definida) de sen(bx) no intervalo [0,1].
>>>>>
>>>>> A antiderivada é (-1/b)*cos(bx).
>>>>>
>>>>> Logo, a integral é (1 - cos(b))/b.
>>>>>
>>>>> Enviado do meu iPhone
>>>>>
>>>>> Em 13 de jan de 2020, Ã (s) 07:04, Esdras Muniz <
>>>>> esdrasmunizm...@gmail.com> escreveu:
>>>>>
>>>>> 
>>>>> Esse limite vai ser a integral inferior de sen(x) de 0 a b. Daí,
>>>>> como Sen Ã

Re: [obm-l] Soma de Riemann

2020-01-14 Por tôpico Claudio Buffara
Estas somas trigonométricas (e várias outras) são obtidas sem grandes 
dificuldades, mas com alguma álgebra, usando números complexos. 

O melhor caminho, a meu ver, seria vc conseguir um daqueles livros russos 
clássicos - Krechmar ou Faddev-Sominski - que contém coletâneas de problemas 
resolvidos sobre este tema e muitos outros.

Agora, me parece que a habilidade de computar estas somas “na mão”, usando 
complexos e/ou alguns truques algébricos (ou até mesmo integrais) tem se 
desvalorizado recentemente devido à existência e ampla disponibilidade de 
softwares gratuitos tais como o Wolfram Alpha, que calculam qualquer soma 
dessas.

Recentemente li um artigo que toca um pouco neste tema, da necessidade de 
modernização dos cursos de exatas. Vou procurar e postarei aqui.

Abs


Enviado do meu iPhone

> Em 14 de jan de 2020, à(s) 12:07, Luiz Antonio Rodrigues 
>  escreveu:
> 
> Olá, Artur!
> Tudo bem?
> Agradeço sua resposta.
> O problema diz:
> 
> É dado o somatório de:
> 
> sen(k*b/n)
> 
> Onde k varia de 1 até n.
> 
> Calcule o limite deste somatório dividido por n, quando n tende a infinito.
> 
> O problema pede que se relacione este limite com uma soma de Riemann.
> 
> Seguindo a sugestão do Claudio, calculei o somatório dos senos em P.A.
> Depois eu calculei o limite solicitado.
> Cheguei n mesma resposta do Claudio, que está correta.
> Aproveito para pedir uma indicação de material sobre este assunto, que 
> considero bastante interessante.
> Muito obrigado!
> Luiz
> 
> 
> Em ter, 14 de jan de 2020 1:32 AM, Artur Costa Steiner 
>  escreveu:
>> Este somatório não é uma soma de Riemann. Seria se fosse
>> 
>> S(n) = 1/n Soma(k = 1, n) sen(k*b/n). 
>> 
>> Mas é S(n) = Soma(k = 1, n) sen(k*b/n). Não se divide por n.
>> 
>> Tem ceteza de que pelo outro processo vc chegou no seu somatório à 
>> expressão correspondente ao caso da soma de Riemann?
>> 
>> Se fizermos b = pi/2, no seu somatorio temos para todo n que S(n) > 
>> sen(pi/2) = 1.Logo, se o limite com n indo para oo existir, será >= 1.Mas 
>> entrando com b = pi/2 na fórmula da soma de Riemann, obtemos 2/pi < 1.
>> 
>> Me corrija se eu tiver cometido algum erro.
>> 
>> Abraços
>> 
>> Artur
>> 
>> Em seg, 13 de jan de 2020 18:04, Luiz Antonio Rodrigues 
>>  escreveu:
>>> Olá, Claudio!
>>> Tudo bem?
>>> Sim, foi esse resultado que eu achei!
>>> Muito obrigado pela ajuda!
>>> 
>>> Em seg, 13 de jan de 2020 5:02 PM, Claudio Buffara 
>>>  escreveu:
>>>> É a soma de n retângulos, todos com base 1/n e o k-esimo com altura 
>>>> sen(kb/n): logo, o limite e’ a integral superior (portanto, a integral 
>>>> definida) de sen(bx) no intervalo [0,1].
>>>> 
>>>> A antiderivada é (-1/b)*cos(bx).
>>>> 
>>>> Logo, a integral é (1 - cos(b))/b.
>>>> 
>>>> Enviado do meu iPhone
>>>> 
>>>>> Em 13 de jan de 2020, Ã (s) 07:04, Esdras Muniz 
>>>>>  escreveu:
>>>>> 
>>>>> Esse limite vai ser a integral inferior de sen(x) de 0 a b. Daí, como 
>>>>> Sen é integravel, esse limite vai ser Sen(b).
>>>>> 
>>>>> Em dom, 12 de jan de 2020 19:19, Luiz Antonio Rodrigues 
>>>>>  escreveu:
>>>>>> Olá, pessoal!
>>>>>> Tudo bem?
>>>>>> Estou pensando neste problema há vários dias e não consigo 
>>>>>> descobrir onde está meu erro.
>>>>>> Alguém pode me ajudar?
>>>>>> 
>>>>>> O problema é o seguinte:
>>>>>> 
>>>>>> É dado o somatório de:
>>>>>> 
>>>>>> sen(k*b/n)
>>>>>> 
>>>>>> Onde k varia de 1 até n.
>>>>>> 
>>>>>> Preciso calcular o limite deste somatório dividido por n, quando n 
>>>>>> tende a infinito.
>>>>>> 
>>>>>> O problema pede que se relacione este limite com uma soma de Riemann.
>>>>>> 
>>>>>> Eu cheguei no valor zero, que está errado.
>>>>>> O problema parece simples...
>>>>>> Agradeço desde já!
>>>>>> Luiz
>>>>>> 
>>>>>> 
>>>>>> -- 
>>>>>> Esta mensagem foi verificada pelo sistema de antivírus e 
>>>>>> acredita-se estar livre de perigo.
>>>>> 
>>>>> -- 
>>>>> Esta mensagem foi verificada pelo sistema de antivírus e 
>>>>> acredita-se estar livre de perigo.
>>>> 
>>>> -- 
>>>> Esta mensagem foi verificada pelo sistema de antivírus e 
>>>> acredita-se estar livre de perigo.
>>> 
>>> -- 
>>> Esta mensagem foi verificada pelo sistema de antivírus e 
>>> acredita-se estar livre de perigo.
>> 
>> -- 
>> Esta mensagem foi verificada pelo sistema de antivírus e 
>> acredita-se estar livre de perigo.
> 
> -- 
> Esta mensagem foi verificada pelo sistema de antivírus e 
> acredita-se estar livre de perigo.

-- 
Esta mensagem foi verificada pelo sistema de antiv�rus e
 acredita-se estar livre de perigo.



Re: [obm-l] Soma de Riemann

2020-01-13 Por tôpico Claudio Buffara
É a soma de n retângulos, todos com base 1/n e o k-esimo com altura sen(kb/n): 
logo, o limite e’ a integral superior (portanto, a integral definida) de 
sen(bx) no intervalo [0,1].

A antiderivada é (-1/b)*cos(bx).

Logo, a integral é (1 - cos(b))/b.

Enviado do meu iPhone

> Em 13 de jan de 2020, à(s) 07:04, Esdras Muniz  
> escreveu:
> 
> 
> Esse limite vai ser a integral inferior de sen(x) de 0 a b. Daí, como Sen é 
> integravel, esse limite vai ser Sen(b).
> 
> Em dom, 12 de jan de 2020 19:19, Luiz Antonio Rodrigues 
>  escreveu:
>> Olá, pessoal!
>> Tudo bem?
>> Estou pensando neste problema há vários dias e não consigo descobrir onde 
>> está meu erro.
>> Alguém pode me ajudar?
>> 
>> O problema é o seguinte:
>> 
>> É dado o somatório de:
>> 
>> sen(k*b/n)
>> 
>> Onde k varia de 1 até n.
>> 
>> Preciso calcular o limite deste somatório dividido por n, quando n tende a 
>> infinito.
>> 
>> O problema pede que se relacione este limite com uma soma de Riemann.
>> 
>> Eu cheguei no valor zero, que está errado.
>> O problema parece simples...
>> Agradeço desde já!
>> Luiz
>> 
>> 
>> -- 
>> Esta mensagem foi verificada pelo sistema de antivírus e 
>> acredita-se estar livre de perigo.
> 
> -- 
> Esta mensagem foi verificada pelo sistema de antivírus e 
> acredita-se estar livre de perigo.

-- 
Esta mensagem foi verificada pelo sistema de antiv�rus e
 acredita-se estar livre de perigo.



Re: [obm-l] Soma de Riemann

2020-01-12 Por tôpico Claudio Buffara
Você sabe como somar os senos de arcos cujas medidas formam uma PA?
Use e^(ix) = cos(x) = i*sen(x).

On Sun, Jan 12, 2020 at 7:19 PM Luiz Antonio Rodrigues <
rodrigue...@gmail.com> wrote:

> Olá, pessoal!
> Tudo bem?
> Estou pensando neste problema há vários dias e não consigo descobrir onde
> está meu erro.
> Alguém pode me ajudar?
>
> O problema é o seguinte:
>
> É dado o somatório de:
>
> sen(k*b/n)
>
> Onde k varia de 1 até n.
>
> Preciso calcular o limite deste somatório dividido por n, quando n tende a
> infinito.
>
> O problema pede que se relacione este limite com uma soma de Riemann.
>
> Eu cheguei no valor zero, que está errado.
> O problema parece simples...
> Agradeço desde já!
> Luiz
>
>
> --
> Esta mensagem foi verificada pelo sistema de antivírus e
> acredita-se estar livre de perigo.

-- 
Esta mensagem foi verificada pelo sistema de antiv�rus e
 acredita-se estar livre de perigo.



[obm-l] Re: [obm-l] Máximo

2020-01-12 Por tôpico Claudio Buffara
Oi, Gilberto:

Que mal eu pergunte, de onde veio este problema?
E por que um aluno de EM teria que resolver um problema desses (e sem usar
cálculo)?

[]s,
Claudio.


On Sun, Jan 12, 2020 at 6:33 PM gilberto azevedo 
wrote:

> Se a e b são números que satisfazem a equação :
> 17(a²+b²) - 30ab - 16 = 0
> Determinar o máximo de :
> √(16a² + 4b² - 16ab - 12a + 6b + 9)
> Sem utilizar lagrange e nada que envolva ensino superior . Não sei oq
> utilizar, se a sacada é enxergar uma fatoração... Enfim aceitando ideias.
>
> --
> Esta mensagem foi verificada pelo sistema de antivírus e
> acredita-se estar livre de perigo.

-- 
Esta mensagem foi verificada pelo sistema de antiv�rus e
 acredita-se estar livre de perigo.



[obm-l] Re: [obm-l] Re: [obm-l] Teoria dos números

2019-12-13 Por tôpico Claudio Buffara
Em tese, nada impede...  a == b (mod m) <==> (a - b)/m é inteiro.
Por exemplo, em trigonometria trabalha-se muito com congruência mod 2*pi.
sen x = sen y  e  cos x = cos y <==> x == y (mod 2*pi)

On Fri, Dec 13, 2019 at 3:54 PM Esdras Muniz 
wrote:

> Existe congruência com números que não são inteiros?
>
> Em sex, 13 de dez de 2019 11:57, Prof. Douglas Oliveira <
> profdouglaso.del...@gmail.com> escreveu:
>
>> Olá caros amigos,
>> preciso de uma ajuda pra criar uma fórmula que seja congruente (módulo p)
>> ao somatório
>> S_a=sum{(a^k)/k}, com k de 1 a p-1, sendo p primo ímpar.
>>
>> Saudações
>> Douglas Oliveira
>>
>> --
>> Esta mensagem foi verificada pelo sistema de antivírus e
>> acredita-se estar livre de perigo.
>
>
> --
> Esta mensagem foi verificada pelo sistema de antivírus e
> acredita-se estar livre de perigo.

-- 
Esta mensagem foi verificada pelo sistema de antiv�rus e
 acredita-se estar livre de perigo.



Re: [obm-l]

2019-11-29 Por tôpico Claudio Buffara
Acho que com números complexos e alguma álgebra sai.

Se os vértices do triângulo forem R, Rw  e Rw^2 (onde w = cis(2pi/3) e R é
um real positivo) e P = z, então:
a = |z - R|, b = |z - Rw|; c = |z - Rw^2| ==>
a^2 + b^2 + c^2 = |z - R|^2 + |z - Rw|^2 + |z - Rw^2|^2 = 3*|z|^2 + 3*R^2
 (se não errei nenhuma conta)

Neste caso, L^2 = 3*R^2, de modo que o lado direito da expressão do
enunciado será igual a (3*|z|^2 + 6*R^2)^2 = 9*(|z|^4 + 4*R^2*|z|^2 +
4*R^4).

O lado esquerdo deve dar um pouco mais de trabalho...


On Tue, Nov 26, 2019 at 7:00 PM gilberto azevedo 
wrote:

> Pesquisando achei uma relação muito interessante, mas não achei nenhuma
> demonstração dela na web.
> Pra quem se interessar Seja um ponto P no interior de um triângulo
> equilátero de lado l, e a,b,c a distância desse ponto aos vértices do
> triângulo. Provar que :
> 3( a⁴ + b⁴ + c⁴ + l⁴) = ( a² + b² + c² + l²)²
>
> --
> Esta mensagem foi verificada pelo sistema de antivírus e
> acredita-se estar livre de perigo.

-- 
Esta mensagem foi verificada pelo sistema de antiv�rus e
 acredita-se estar livre de perigo.



Re: [obm-l] Triângulos.

2019-11-22 Por tôpico Claudio Buffara
Do jeito que está escrito, uma infinidade.

Enviado do meu iPhone

> Em 22 de nov de 2019, à(s) 19:18, Guilherme Abbehusen 
>  escreveu:
> 
> 
> Olá, 
>   Preciso de ajuda com a seguinte questão: 
> 
> Tendo em vista a leis dos Cossenos, marque a quantidade de triângulos 
> obtusângulos que podemos formar com lados menores do que 7.
> a) 6
> b) 7
> c) 8Â 
> d) 9
> e) 10
> 
> -- 
> Esta mensagem foi verificada pelo sistema de antivírus e 
> acredita-se estar livre de perigo.

-- 
Esta mensagem foi verificada pelo sistema de antiv�rus e
 acredita-se estar livre de perigo.


=
Instru��es para entrar na lista, sair da lista e usar a lista em
http://www.mat.puc-rio.br/~obmlistas/obm-l.html
=


Re: [obm-l]

2019-11-22 Por tôpico Claudio Buffara
MG >= MH decorre de MA >= MG.
Pois 1/MH(a1,a2,...,an) = (1/a1 + 1/a2 + ... + 1/an)/n =
MA(1/a1,1/a2,...,1/an) >= MG(1/a1,1/a2,...,1/an) = 1/MG(a1,a2,...,an) ==>
MH(a1,a2,...,an) <= MG(a1,a2,...,an)


On Fri, Nov 22, 2019 at 6:39 PM Esdras Muniz 
wrote:

> Eu usei mg>= mh
>
> Em sex, 22 de nov de 2019 17:04, Claudio Buffara <
> claudio.buff...@gmail.com> escreveu:
>
>> Que podemos elevar ao quadrado, obtendo x^6/(x - 12).
>>
>> Ou seja, o problema se torna achar o valor mínimo de x^6/(x - 12), com x
>> > 12  (não pode ser "=" ...).
>> Depois, é só tirar a raiz quadrada.
>>
>> Agora, usamos a sugestão do Julio: y^6 = x - 12 ==> x^6 = (y^6 + 12)^6.
>> E a expressão a ser minimizada passa a ser (y^6 + 12)^6/y^6 = ((y^6 +
>> 12)/y)^6 = (y^5 + 12/y)^6.
>>
>> y^5 + 12/y = 6*(y^5 + 5*(2,4/y))/6 >= 6*(y^5*(2,4/y)^5)^(1/6)  (MA >= MG)
>> = 6*2,4^(5/6).
>>
>> Ou seja, o valor mínimo de y^5 + 12/y é igual a 6*(12/5)^(5/6), e é
>> obtido quando y^5 = 2,4/y <==> y = (12/5)^(1/6).
>> ==> y^6 = 12/5 = x - 12
>> ==> x = 12 + 12/5 = 72/5
>> ==> x^6/(x-12) = (72/5)^6/(12/5) = 6^6*12^5/5^5 = 2^16*3^11/5^5
>> ==> o valor mínimo de raiz(x^6/(x-12)) é igual a 2^8*3^(11/2)/5^(5/2) e é
>> atingido quando x = 72/5.
>>
>> Moral da história: se souber usar derivada, use derivada...
>>
>> []s,
>> Claudio.
>>
>> On Fri, Nov 22, 2019 at 4:22 PM Claudio Buffara <
>> claudio.buff...@gmail.com> wrote:
>>
>>> Melhor reescrever a expressão.
>>> Como x - 12 >= 0, podemos supor que x >= 12.
>>> Nesse caso, a expressão a ser minimizada fica x^3/raiz(x-12), certo?
>>>
>>> On Fri, Nov 22, 2019 at 4:20 PM gilberto azevedo 
>>> wrote:
>>>
>>>> Não vejo com isso ajuda. Eu tava pensando em usa AM - MG , mas n ajudou
>>>> mt.
>>>>
>>>> Em sex, 22 de nov de 2019 10:10, Julio César Saldaña Pumarica <
>>>> saldana...@pucp.edu.pe> escreveu:
>>>>
>>>>> mudando a variável:
>>>>>
>>>>> x-12 = y^6
>>>>>
>>>>> El vie., 22 nov. 2019 a las 2:40, gilberto azevedo (<
>>>>> gil159...@gmail.com>) escribió:
>>>>>
>>>>>> Como achar o mínimo de :
>>>>>> x² * √(x²/(x-12)) , usando apenas desigualdades comuns ?
>>>>>>
>>>>>> --
>>>>>> Esta mensagem foi verificada pelo sistema de antivírus e
>>>>>> acredita-se estar livre de perigo.
>>>>>
>>>>>
>>>>> --
>>>>> Esta mensagem foi verificada pelo sistema de antivírus e
>>>>> acredita-se estar livre de perigo.
>>>>
>>>>
>>>> --
>>>> Esta mensagem foi verificada pelo sistema de antivírus e
>>>> acredita-se estar livre de perigo.
>>>
>>>
>> --
>> Esta mensagem foi verificada pelo sistema de antivírus e
>> acredita-se estar livre de perigo.
>
>
> --
> Esta mensagem foi verificada pelo sistema de antivírus e
> acredita-se estar livre de perigo.

-- 
Esta mensagem foi verificada pelo sistema de antiv�rus e
 acredita-se estar livre de perigo.



Re: [obm-l]

2019-11-22 Por tôpico Claudio Buffara
Que podemos elevar ao quadrado, obtendo x^6/(x - 12).

Ou seja, o problema se torna achar o valor mínimo de x^6/(x - 12), com x >
12  (não pode ser "=" ...).
Depois, é só tirar a raiz quadrada.

Agora, usamos a sugestão do Julio: y^6 = x - 12 ==> x^6 = (y^6 + 12)^6.
E a expressão a ser minimizada passa a ser (y^6 + 12)^6/y^6 = ((y^6 +
12)/y)^6 = (y^5 + 12/y)^6.

y^5 + 12/y = 6*(y^5 + 5*(2,4/y))/6 >= 6*(y^5*(2,4/y)^5)^(1/6)  (MA >= MG) =
6*2,4^(5/6).

Ou seja, o valor mínimo de y^5 + 12/y é igual a 6*(12/5)^(5/6), e é obtido
quando y^5 = 2,4/y <==> y = (12/5)^(1/6).
==> y^6 = 12/5 = x - 12
==> x = 12 + 12/5 = 72/5
==> x^6/(x-12) = (72/5)^6/(12/5) = 6^6*12^5/5^5 = 2^16*3^11/5^5
==> o valor mínimo de raiz(x^6/(x-12)) é igual a 2^8*3^(11/2)/5^(5/2) e é
atingido quando x = 72/5.

Moral da história: se souber usar derivada, use derivada...

[]s,
Claudio.

On Fri, Nov 22, 2019 at 4:22 PM Claudio Buffara 
wrote:

> Melhor reescrever a expressão.
> Como x - 12 >= 0, podemos supor que x >= 12.
> Nesse caso, a expressão a ser minimizada fica x^3/raiz(x-12), certo?
>
> On Fri, Nov 22, 2019 at 4:20 PM gilberto azevedo 
> wrote:
>
>> Não vejo com isso ajuda. Eu tava pensando em usa AM - MG , mas n ajudou
>> mt.
>>
>> Em sex, 22 de nov de 2019 10:10, Julio César Saldaña Pumarica <
>> saldana...@pucp.edu.pe> escreveu:
>>
>>> mudando a variável:
>>>
>>> x-12 = y^6
>>>
>>> El vie., 22 nov. 2019 a las 2:40, gilberto azevedo ()
>>> escribió:
>>>
>>>> Como achar o mínimo de :
>>>> x² * √(x²/(x-12)) , usando apenas desigualdades comuns ?
>>>>
>>>> --
>>>> Esta mensagem foi verificada pelo sistema de antivírus e
>>>> acredita-se estar livre de perigo.
>>>
>>>
>>> --
>>> Esta mensagem foi verificada pelo sistema de antivírus e
>>> acredita-se estar livre de perigo.
>>
>>
>> --
>> Esta mensagem foi verificada pelo sistema de antivírus e
>> acredita-se estar livre de perigo.
>
>

-- 
Esta mensagem foi verificada pelo sistema de antiv�rus e
 acredita-se estar livre de perigo.



Re: [obm-l]

2019-11-22 Por tôpico Claudio Buffara
Melhor reescrever a expressão.
Como x - 12 >= 0, podemos supor que x >= 12.
Nesse caso, a expressão a ser minimizada fica x^3/raiz(x-12), certo?

On Fri, Nov 22, 2019 at 4:20 PM gilberto azevedo 
wrote:

> Não vejo com isso ajuda. Eu tava pensando em usa AM - MG , mas n ajudou mt.
>
> Em sex, 22 de nov de 2019 10:10, Julio César Saldaña Pumarica <
> saldana...@pucp.edu.pe> escreveu:
>
>> mudando a variável:
>>
>> x-12 = y^6
>>
>> El vie., 22 nov. 2019 a las 2:40, gilberto azevedo ()
>> escribió:
>>
>>> Como achar o mínimo de :
>>> x² * √(x²/(x-12)) , usando apenas desigualdades comuns ?
>>>
>>> --
>>> Esta mensagem foi verificada pelo sistema de antivírus e
>>> acredita-se estar livre de perigo.
>>
>>
>> --
>> Esta mensagem foi verificada pelo sistema de antivírus e
>> acredita-se estar livre de perigo.
>
>
> --
> Esta mensagem foi verificada pelo sistema de antivírus e
> acredita-se estar livre de perigo.

-- 
Esta mensagem foi verificada pelo sistema de antiv�rus e
 acredita-se estar livre de perigo.



Re: [obm-l] Re: [obm-l] Qual o 2020º termo da sequência abaixo ?

2019-11-17 Por tôpico Claudio Buffara
Pela definição da sequência.
Quando a a(n) + a(n+1) > 40, a(n+2) = resto da divisão de a(n) + a(n+1) por 40, 
sendo que neste caso os restos vão de 1 a 40 (ao invés de 0 a 39).

Enviado do meu iPhone

> Em 17 de nov de 2019, à(s) 18:59, Jamil Silva  
> escreveu:
> 
> Por que mod40 ?
> 
> 17.11.2019, 14:36, "Claudio Buffara" :
>> Me parece que basta calcular o 2020o termo sem a restrição de ser mod 40 
>> (é uma sequência de Fibonacci começando por 5 e 2) e depois ver quanto 
>> e’ a(2020) mod 40, sendo que na redução mod 40, ao invés dos restos 
>> serem 0, 1, ..., 39, eles serão 1, 2, ..., 40.
>> 
>> Enviado do meu iPhone
>> 
>>> Â Em 17 de nov de 2019, Ã (s) 08:15, Jamil Silva  
>>> escreveu:
>>> 
>>>  5, 2, 7, 9, 16, 25, 1, 26, 27, 13, 40, 13, 13, 26, 39, 25, 24,...
>>> 
>>>  Sua lei de formação é a seguinte:
>>> 
>>> Â a(1) = 5
>>> Â a(2) = 2
>>>  a(n+2) = [a(n+1)+a(n)], sse [a(n+1) + a(n)] ≤ 40
>>> Â a(n+2) = [a(n+1)+a(n)] - 40, sse [a(n+1) + a(n)] > 40
>>> 
>>> Â --
>>>  Esta mensagem foi verificada pelo sistema de antivírus e
>>> Â acredita-se estar livre de perigo.
>>> 
>>> Â =
>>>  Instruções para entrar na lista, sair da lista e usar a lista em
>>> Â http://www.mat.puc-rio.br/~obmlistas/obm-l.html
>>> Â =
>> 
>> --
>> Esta mensagem foi verificada pelo sistema de antiv?rus e
>> Â acredita-se estar livre de perigo.
>> 
>> =
>> Instru??es para entrar na lista, sair da lista e usar a lista em
>> http://www.mat.puc-rio.br/~obmlistas/obm-l.html
>> =
> 
> -- 
> Esta mensagem foi verificada pelo sistema de antivírus e
> acredita-se estar livre de perigo.
> 
> =
> Instruções para entrar na lista, sair da lista e usar a lista em
> http://www.mat.puc-rio.br/~obmlistas/obm-l.html
> =

-- 
Esta mensagem foi verificada pelo sistema de antiv�rus e
 acredita-se estar livre de perigo.


=
Instru��es para entrar na lista, sair da lista e usar a lista em
http://www.mat.puc-rio.br/~obmlistas/obm-l.html
=


Re: [obm-l] Qual o 2020º termo da sequência abaixo ?

2019-11-17 Por tôpico Claudio Buffara
Me parece que basta calcular o 2020o termo sem a restrição de ser mod 40 (é uma 
sequência de Fibonacci começando por 5 e 2) e depois ver quanto e’ a(2020) mod 
40, sendo que na redução mod 40, ao invés dos restos serem 0, 1, ..., 39, eles 
serão 1, 2, ..., 40.

Enviado do meu iPhone

> Em 17 de nov de 2019, à(s) 08:15, Jamil Silva  
> escreveu:
> 
> 5, 2, 7, 9, 16, 25, 1, 26, 27, 13, 40, 13, 13, 26, 39, 25, 24,...
> 
> Sua lei de formação é a seguinte:
> 
> a(1) = 5 
> a(2) = 2
> a(n+2) = [a(n+1)+a(n)],  sse [a(n+1) + a(n)] ≤ 40
> a(n+2) = [a(n+1)+a(n)] - 40, sse [a(n+1) + a(n)] > 40
> 
> -- 
> Esta mensagem foi verificada pelo sistema de antivírus e
> acredita-se estar livre de perigo.
> 
> =
> Instruções para entrar na lista, sair da lista e usar a lista em
> http://www.mat.puc-rio.br/~obmlistas/obm-l.html
> =

-- 
Esta mensagem foi verificada pelo sistema de antiv�rus e
 acredita-se estar livre de perigo.


=
Instru��es para entrar na lista, sair da lista e usar a lista em
http://www.mat.puc-rio.br/~obmlistas/obm-l.html
=


Re: [obm-l] Qual o 2020º termo da sequência abaixo ?

2019-11-17 Por tôpico Claudio Buffara
Eu também usaria uma planilha pra checar o resultado.

Enviado do meu iPhone

> Em 17 de nov de 2019, à(s) 11:56, Claudio Buffara  
> escreveu:
> 
> Me parece que basta calcular o 2020o termo sem a restrição de ser mod 40 (é 
> uma sequência de Fibonacci começando por 5 e 2) e depois ver quanto e’ 
> a(2020) mod 40, sendo que na redução mod 40, ao invés dos restos serem 0, 1, 
> ..., 39, eles serão 1, 2, ..., 40.
> 
> Enviado do meu iPhone
> 
>> Em 17 de nov de 2019, à(s) 08:15, Jamil Silva  
>> escreveu:
>> 
>> 5, 2, 7, 9, 16, 25, 1, 26, 27, 13, 40, 13, 13, 26, 39, 25, 24,...
>> 
>> Sua lei de formação é a seguinte:
>> 
>> a(1) = 5 
>> a(2) = 2
>> a(n+2) = [a(n+1)+a(n)],  sse [a(n+1) + a(n)] ≤ 40
>> a(n+2) = [a(n+1)+a(n)] - 40, sse [a(n+1) + a(n)] > 40
>> 
>> -- 
>> Esta mensagem foi verificada pelo sistema de antivírus e
>> acredita-se estar livre de perigo.
>> 
>> =
>> Instruções para entrar na lista, sair da lista e usar a lista em
>> http://www.mat.puc-rio.br/~obmlistas/obm-l.html
>> =

-- 
Esta mensagem foi verificada pelo sistema de antiv�rus e
 acredita-se estar livre de perigo.


=
Instru��es para entrar na lista, sair da lista e usar a lista em
http://www.mat.puc-rio.br/~obmlistas/obm-l.html
=


[obm-l] Re: [obm-l] Minha solução para o item c) do problema 3 da prova da OBM-2017

2019-11-08 Por tôpico Claudio Buffara
Uma forma de ver se sua solução está certa é tentar dar outra solução,
essencialmente diferente da primeira.

A meu ver, a solução mais elementar é por enumeração pura e simples e usa
apenas o princípio multiplicativo, sem nenhuma "sacada brilhante".

Há 5 números de um algarismo no sistema Impa (1, 3, 5, 7, 9);
5^2 = 25 números de dois algarismos;
5^3 = 125 de três;
5^4 = 625 de quatro.
Isso significa que , o maior número de quatro algarismos no sistema
Impa, ocupa o lugar de número 5+25+125+625 = 780 na sequência e, portanto,
corresponde ao número 780 no sistema decimal.

5^5 = 3125 > 2017, o que implica que 2017 corresponde a um número Impa de
cinco algarismos.
Há 625 números Impa de 5 algarismos começando com 1 (de 1 a 1) ==>
1 corresponde a 780+625 = 1405
Há 625 deles começando com 3 (de 3 a 3) ==> 3 corresponde a
1405+780 = 2185.
Ou seja, o número desejado (correspondente a 2017) começa com "3".

Voltando ao 1 (correspondente ao nosso 1405) ...
O número seguinte é 3 (1406).
Há:
- 125 números começando com 31 (de 3 a 31999): 1405+125 = 1530;
- 125 começando com 33 (33111 a 33999): 1530+125 = 1655
- 125 começando com 35: 1655+125 = 1780
- 125 começando com 37: 1780+125 = 1905
- 125 começando com 39: 1905+125 = 2030 ==> o número desejado começa com
"39".

Tomemos, então, o Impa 37999 (correspondente a 1905). O seguinte é o 39111.
Há:
- 25 números começando com 391 (39111 a 39199): 1905+25 = 1930
- 25 começando com 393: 1930+25 = 1955
- 25 começando com 395: 1955+25 = 1980
- 25 começando com 397: 1980+25 = 2005 ==> 2005 corresponde ao Impa 39799
==> 2006 é 39911

E, por fim, há:
- 5 números começando com 3991: 2005+5 = 2010
- 5 começando com 3993: 2010+5 = 2015 ==> 2015 corresponde ao Impa 39939
==> 2016 é 39951 ==> 2017 é 39953.

[]s,
Claudio.


On Thu, Nov 7, 2019 at 12:36 PM Cauã DSR  wrote:

>
> Tenho um pequeno problema, eu fiz o item C) do problema 3 da prova da OBM
> de 2017, mas não tenho certeza sobre seu resultado, então achei uma boa
> fazer minha primeira aparição no grupo perguntando se o que fiz está certo.
>
> 3. Na Terra dos Impas, somente os algarismos ímpares são utilizados para
> contar e escrever números. Assim, em vez dos numeros 1, 2, 3, 4, 5, 6, 7,
> 8, 9, 10, 11, 12,. . . os Impas tem os números correspondentes 1, 3, 5, 7,
> 9, 11, 13, 15, 17, 19, 31, 33, . . . (note que os números dos Impas tem
> somente algarismos ímpares). Por exemplo, se
> uma criança tem 11 anos, os Impas diriam que ela tem 31 anos.
>
> c) Escreva, na linguagem dos Impas, o numero que na nossa representação
> decimal é escrito como 2017.
>
> Minha solução:
> Como no problema só temos Ímpares para usar como algarismo {1,3,5,7,9},
> temos um sistema de numeração de base 5, porém com os algarismos ímpares ao
> invés da base 5 comumente usada {0,1,2,3,4}. Ao analisar isso decidi
> transformar 2017 em um número de Base 5 {1,2,3,4,5}, ao usar esta base,
> percebi que para transformar um número de Base Decimal em um de Base 5
> {1,2,3,4,5} é quase o mesmo processo para transformá-lo em um número de
> Base 5 {0,1,2,3,4}, onde a única diferença é que podemos usar 5x5^n e que
> quando tivermos 0x5^n apenas basta ignorá-lo e partir para a próxima
> potência de 5 (5^n-1).
> Ao fazer isto obtive o seguinte:
>
> 2017= 3x5^4+1x5^3+3x5^1+2x5^0
> 2017= 3132
>
> Agora, saibam que tem como transformar um número n de base 5 {1,2,3,4,5}
> em um número x de base 5 {1,3,5,7,9} apenas mudando os algarismos
> correspondentes, uma vez que os dois tem base 5.
> então temos os seguinte correspondentes das Bases 5 {1,2,3,4,5} e
> {1,3,5,7,9} respectivamente
> 1=1
> 2=3
> 3=5
> 4=7
> 5=9
>
> Portanto o número 3132 da Base 5 {1,2,3,4,5} vira 5153 da Base 5
> {1,3,5,7,9}
>
> --
> Esta mensagem foi verificada pelo sistema de antivírus e
> acredita-se estar livre de perigo.

-- 
Esta mensagem foi verificada pelo sistema de antiv�rus e
 acredita-se estar livre de perigo.



Re: [obm-l] Problema 5 OBMU 2018

2019-10-31 Por tôpico Claudio Buffara
Mudando um pouco a notação...
Ponha: Df(x) = f(x+1) - f(x).

Para todo x em R+, e todo inteiro positivo k, existe (pelo TVM) y_k entre x
e x+1 tal que (Df)^(k)(x) = f^(k)(x+1) - f^(k)(x) = f'^(k+1)(y_k) > 0.
Logo, Df satisfaz a primeira condição do enunciado.
Além disso, como f' é positiva para todo x em R+, para todo m em R+, existe
(pelo TVM) a_m entre m e m+1 tal que Df(m) = f(m+1) - f(m) = f'(a_m) > 0
==> Df(m) é inteiro positivo.
Logo, Df satisfaz a condição 2 do enunciado.

Suponha que o resultado valha para Df (ou seja, que Df(n) >= 2^(n-1) para
todo inteiro positivo n).
Pela condição 2 do enunciado, f(1) >= 1.
Além disso, para todo n > 1, f(n) = f(1) + Df(1) + Df(2) + ... + Df(n-1) >=
1 + 2^0 + 2^1 + ... + 2^(n-2) = 1 + (2^(n-1) - 1) = 2^(n-1)
Portanto, se o resultado valer para Df, então valerá para f.

Pelo mesmo argumento, as funções D^2f, D^3f, ..., D^(n-1)f, dadas por:
D^2f(x) = Df(x+1) - Df(x),
D^3f(x) = D^2f(x+1) - D^2f(x)
...
D^(n-1)f(x) = D^(n-2)f(x+1) - D^(n-2)f(x)
também satisfazem as duas condições do enunciado.

Logo, se o resultado valer para D^2f, então valerá para Df (e, portanto,
pelo que foi feito acima, para f).
E assim por diante...
se valer para D^3f, então valerá para D^2f,
...
se valer para D^(n-1)f, então valerá para D^(n-2)f.

Agora, D^(n-1)f(1) = 1 (já que D^(n-1)f satisfaz a condição 2 do enunciado).
E assim...
D^(n-2)f(2) = D^(n-2)f(1) + D^(n-1)f(1)  >= 1 + 1 = 2
D^(n-3)f(3) = D^(n-3)f(1) + D^(n-2)f(1) + D^(n-2)f(2) >= 1 + 1 + 2 = 1 + 3
= 4
...
D^2f(n-2) = D^2f(1) + D^3f(1) + ... + D^3f(n-3) >= 1 + 1 + ... + 2^(n-4) =
1 + (2^(n-3) - 1) = 2^(n-3)
Df(n-1) = Df(1) + D^2f(1) + D^2f(2) + ... D^2f(n-2) >= 1 + 1 + 2 + ... +
2^(n-3) = 1 + (2^(n-2) - 1) = 2^(n-2)

Finalmente...
f(n) = f(1) + Df(1) + Df(2) + ... + Df(n-1) >= 1 + 1 + 2^1 + ... + 2^(n-2)
= 1 + (2^(n-1) - 1) = 2^(n-1)

E acabou!

[]s,
Claudio.



On Wed, Oct 30, 2019 at 5:38 PM Ernesto Rodrigues 
wrote:

> Pense um pouco sobre g(x)=f(x+1)-f(x), essa questão é bem tricky, o
> segredo é que a g satisfaz as condições da questão, logo, por indução, vale
> que g(n) é maior ou igual a dois elevado a n menos um, mas isto implica que
> o mesmo vale para f(n+1), completando a indução (tem que pensar bastante
> para sacar a ideia).
>
> Em qua, 30 de out de 2019 13:47, Lucas Dantas 
> escreveu:
>
>> Meu grupo da faculdade estamos com dificuldade de resolver o problema 5
>> da segunda fase da OBM-U 2018.
>>
>> Enunciado: Sejam R+ o conjunto dos números reais positivos e f:R+->R+ uma
>> função infinitamente diferenciável tal que:
>>
>> 1) Para todo k inteiro positivo e para todo real positivo x, f^(k)(x)>0 .
>> (Onde f^(k) representa a k-esima derivada).
>>
>> 2) para todo m inteiro positivo, f(m) é inteiro positivo.
>>
>> Prove que para todo inteiro positivo n, f(n)>= 2^(n-1)
>>
>>
>>
>> Estávamos pensando que qualquer função com todas as derivadas positivas
>> cresceria mais rápido que uma exponencial do tipo b^x com uma base maior
>> que um. Mas daí conseguimos o contraexemplo vish(sqrt(x)). Agora estamos
>> pensando em pensar em algo do tipo que f(x+1)>=2f(x). Mas mesmo assim não
>> sei como continuar. Se alguém tiver alguma sugestão de como resolver ou
>> alguma ideia pra nos ajudar.
>>
>> --
>> Esta mensagem foi verificada pelo sistema de antivírus e
>> acredita-se estar livre de perigo.
>
>
> --
> Esta mensagem foi verificada pelo sistema de antivírus e
> acredita-se estar livre de perigo.

-- 
Esta mensagem foi verificada pelo sistema de antiv�rus e
 acredita-se estar livre de perigo.



Re: [obm-l] Re: [obm-l] Re: [obm-l] Re: [obm-l] Domínio de uma Função

2019-10-29 Por tôpico Claudio Buffara
Eu uso Excel.
Muito útil pra analisar gráficos e gerar conjecturas em cálculo, teoria dos 
números e combinatória.

Abs

Enviado do meu iPhone

> Em 29 de out de 2019, à(s) 18:54, Luiz Antonio Rodrigues 
>  escreveu:
> 
> 
> Oi, Claudio!
> Tudo bem?
> Você sugere uma planilha tipo Excel ou Numbers?
> Eu nunca pensei nisso...
> Acho que é uma ideia excelente!
> 
> 
>> On Tue, Oct 29, 2019, 12:29 PM Claudio Buffara  
>> wrote:
>> Use uma planilha. Eu acho melhor pra analisar funções.
>> 
>> Enviado do meu iPhone
>> 
>>> Em 29 de out de 2019, Ã (s) 11:23, Luiz Antonio Rodrigues 
>>>  escreveu:
>>> 
>>> 
>>> Olá, Claudio!
>>> Bom dia!
>>> Foi assim que eu pensei também...
>>> Não entendi por que a calculadora gráfica indicou domínio [0, + 
>>> infinito).
>>> Vou verificar tudo novamente...
>>> Muito obrigado pela ajuda! 
>>> Abraço!
>>> Luiz
>>> 
>>>> On Tue, Oct 29, 2019, 10:49 AM Claudio Buffara  
>>>> wrote:
>>>> Estritamente falando, o domínio da função não foi definido.
>>>> Nestes casos, o usual é tomar por domínio o maior subconjunto de R 
>>>> no qual a fórmula faz sentido.
>>>> E, neste caso específico, a fórmula faz sentido para todo x real.
>>>> 
>>>> O gráfico de h(x) = x^(2/3) tem uma "ponta" em x = 0, de modo que  a 
>>>> derivada h'(x) não é definida na origem.
>>>> 
>>>> Mas não deveria haver problema algum em x = -1.
>>>> 
>>>> 
>>>>> On Tue, Oct 29, 2019 at 4:57 AM Luiz Antonio Rodrigues 
>>>>>  wrote:
>>>>> Olá, pessoal!
>>>>> Tudo bem?
>>>>> Estou tentando descobrir os pontos  de máximo e mínimo da 
>>>>> função:
>>>>> 
>>>>> f(x)=1.5*(x)^(2/3)+x
>>>>> 
>>>>> A primeira derivada se anula em x=-1.
>>>>> Mas porque -1 não pertence ao domínio da função?
>>>>> Vi isso numa calculadora gráfica.
>>>>> Eu não consigo entender isso...
>>>>> Não estou tirando a raiz cúbica de um número ao quadrado?
>>>>> Alguém pode me ajudar?
>>>>> Muito obrigado!
>>>>> Luiz
>>>>> 
>>>>> -- 
>>>>> Esta mensagem foi verificada pelo sistema de antivírus e 
>>>>> acredita-se estar livre de perigo.
>>>> 
>>>> -- 
>>>> Esta mensagem foi verificada pelo sistema de antivírus e 
>>>> acredita-se estar livre de perigo.
>>> 
>>> -- 
>>> Esta mensagem foi verificada pelo sistema de antivírus e 
>>> acredita-se estar livre de perigo.
>> 
>> -- 
>> Esta mensagem foi verificada pelo sistema de antivírus e 
>> acredita-se estar livre de perigo.
> 
> -- 
> Esta mensagem foi verificada pelo sistema de antivírus e 
> acredita-se estar livre de perigo.

-- 
Esta mensagem foi verificada pelo sistema de antiv�rus e
 acredita-se estar livre de perigo.



Re: [obm-l] Re: [obm-l] Re: [obm-l] Domínio de uma Função

2019-10-29 Por tôpico Claudio Buffara
Use uma planilha. Eu acho melhor pra analisar funções.

Enviado do meu iPhone

> Em 29 de out de 2019, à(s) 11:23, Luiz Antonio Rodrigues 
>  escreveu:
> 
> 
> Olá, Claudio!
> Bom dia!
> Foi assim que eu pensei também...
> Não entendi por que a calculadora gráfica indicou domínio [0, + infinito).
> Vou verificar tudo novamente...
> Muito obrigado pela ajuda! 
> Abraço!
> Luiz
> 
>> On Tue, Oct 29, 2019, 10:49 AM Claudio Buffara  
>> wrote:
>> Estritamente falando, o domínio da função não foi definido.
>> Nestes casos, o usual é tomar por domínio o maior subconjunto de R no qual 
>> a fórmula faz sentido.
>> E, neste caso específico, a fórmula faz sentido para todo x real.
>> 
>> O gráfico de h(x) = x^(2/3) tem uma "ponta" em x = 0, de modo que  a 
>> derivada h'(x) não é definida na origem.
>> 
>> Mas não deveria haver problema algum em x = -1.
>> 
>> 
>>> On Tue, Oct 29, 2019 at 4:57 AM Luiz Antonio Rodrigues 
>>>  wrote:
>>> Olá, pessoal!
>>> Tudo bem?
>>> Estou tentando descobrir os pontos  de máximo e mínimo da função:
>>> 
>>> f(x)=1.5*(x)^(2/3)+x
>>> 
>>> A primeira derivada se anula em x=-1.
>>> Mas porque -1 não pertence ao domínio da função?
>>> Vi isso numa calculadora gráfica.
>>> Eu não consigo entender isso...
>>> Não estou tirando a raiz cúbica de um número ao quadrado?
>>> Alguém pode me ajudar?
>>> Muito obrigado!
>>> Luiz
>>> 
>>> -- 
>>> Esta mensagem foi verificada pelo sistema de antivírus e 
>>> acredita-se estar livre de perigo.
>> 
>> -- 
>> Esta mensagem foi verificada pelo sistema de antivírus e 
>> acredita-se estar livre de perigo.
> 
> -- 
> Esta mensagem foi verificada pelo sistema de antivírus e 
> acredita-se estar livre de perigo.

-- 
Esta mensagem foi verificada pelo sistema de antiv�rus e
 acredita-se estar livre de perigo.



[obm-l] Re: [obm-l] Domínio de uma Função

2019-10-29 Por tôpico Claudio Buffara
Estritamente falando, o domínio da função não foi definido.
Nestes casos, o usual é tomar por domínio o maior subconjunto de R no qual
a fórmula faz sentido.
E, neste caso específico, a fórmula faz sentido para todo x real.

O gráfico de h(x) = x^(2/3) tem uma "ponta" em x = 0, de modo que  a
derivada h'(x) não é definida na origem.

Mas não deveria haver problema algum em x = -1.


On Tue, Oct 29, 2019 at 4:57 AM Luiz Antonio Rodrigues <
rodrigue...@gmail.com> wrote:

> Olá, pessoal!
> Tudo bem?
> Estou tentando descobrir os pontos  de máximo e mínimo da função:
>
> f(x)=1.5*(x)^(2/3)+x
>
> A primeira derivada se anula em x=-1.
> Mas porque -1 não pertence ao domínio da função?
> Vi isso numa calculadora gráfica.
> Eu não consigo entender isso...
> Não estou tirando a raiz cúbica de um número ao quadrado?
> Alguém pode me ajudar?
> Muito obrigado!
> Luiz
>
> --
> Esta mensagem foi verificada pelo sistema de antivírus e
> acredita-se estar livre de perigo.

-- 
Esta mensagem foi verificada pelo sistema de antiv�rus e
 acredita-se estar livre de perigo.



Re: [obm-l] Área

2019-10-27 Por tôpico Claudio Buffara
Area = 0, dado que é a intersecção de 4 segmentos. Logo, só pode ser um 
segmento, um ponto ou vazia.

Enviado do meu iPhone

> Em 27 de out de 2019, à(s) 10:23, gilberto azevedo  
> escreveu:
> 
> 
> Dado um paralelogramo abcd de área 1 e a' , b' , c' , d' os pontos médios 
> de ab, bc, cd , ad respectivamente. Calcule a área da figura formada pela 
> intercessão de ab', cd' , da' , bc'.
> 
> -- 
> Esta mensagem foi verificada pelo sistema de antivírus e 
> acredita-se estar livre de perigo.

-- 
Esta mensagem foi verificada pelo sistema de antiv�rus e
 acredita-se estar livre de perigo.


=
Instru��es para entrar na lista, sair da lista e usar a lista em
http://www.mat.puc-rio.br/~obmlistas/obm-l.html
=


Re: [obm-l] Re: Problema 19 da OMDF de 2018.

2019-10-25 Por tôpico Claudio Buffara
E qual a relação entre a e b para que o problema tenha solução?

Enviado do meu iPhone

> Em 25 de out de 2019, à(s) 12:29, Prof. Douglas Oliveira 
>  escreveu:
> 
> 
> Vamos fazer por complexos.
> 
> 1) Coloque os eixos real e imaginário com origem no vértice A.
> 
> 2) Chame de z1 o complexo APÂ  e de z2 o complexo AQ.
> 
> 3)Faca uma rotação de 60 graus, z1cis(60)=z2.
> 
> 4) Igualando as partes real e imaginaria teremos para resposta 2b-a3^(1/2)
> 
> Abraço 
> ProfDouglasOliveira
> 
> Em qui, 24 de out de 2019 23:44, Guilherme Abbehusen 
>  escreveu:
>> Aliás, esqueci de avisar que a resposta deve ser em função de a e b. As 
>> alternativas seriam: A) 2a - b*3^1/2Â  Â  B) a - 2b*3^1/2Â  Â  Â C) 3b - 
>> a*3^1/2Â  Â  D) 2b - a*3^1/2Â  Â  Â E) b - a*3^1/2
>> 
>> 
>> Em qui, 24 de out de 2019 Ã s 23:06, Guilherme Abbehusen 
>>  escreveu:
>>> Olá, alguém poderia me ajudar com essa questão?
>>> 
>>> Azambuja tem uma folha retangular ABCD de dimensões AB = a e BC = b , na 
>>> qual quer efetuar três cortes para obter um triângulo equilátero. 
>>> Portanto, escolhe o ponto P sobre BC e o ponto Q sobre CD, obtendo o 
>>> triângulo equilátero APQ. Qual é o comprimento do segmento BP?
>>> 
>>> Agradeço desde já.
>> 
>> -- 
>> Esta mensagem foi verificada pelo sistema de antivírus e 
>> acredita-se estar livre de perigo.
> 
> -- 
> Esta mensagem foi verificada pelo sistema de antivírus e 
> acredita-se estar livre de perigo.

-- 
Esta mensagem foi verificada pelo sistema de antiv�rus e
 acredita-se estar livre de perigo.



[obm-l] Re: [obm-l] Re: [obm-l] Triângulos

2019-09-26 Por tôpico Claudio Buffara
Que tal quebrar uma vareta em 3 pedaços e calcular a probabilidade
CONDICIONAL do pedaço mais longo exceder o mais curto em não mais do que
10%, DADO QUE é possível formar um triângulo com estes pedaços?

Outro problema interessante (talvez até mais do que o original) é explicar
PORQUE estas escolhas levam a respostas diferentes.

Nesta linha, existe o paradoxo de Bertrand. Vide aqui:
http://www.rpm.org.br/cdrpm/34/6.htm

[]s,
Claudio.



On Thu, Sep 26, 2019 at 8:30 PM Ralph Teixeira  wrote:

> Hmmm... Esse enunciado, como estah , nao funciona... O problema eh definir
> o que significa escolher um triangulo "ao acaso". Algumas opcoes:
>
> -- Escolher 3 numeros uniformemente na regiao do R^3 definida por
> 0 -- Escolher 3 pontos uniformemente dentro de um circulo de raio R, que
> seriam os vertices. (Se necessario, tome R-> Inf depois.)
> -- Escolher 3 pontos uniformemente dentro de um quadrado de lado R, que
> seriam os vertices. (Se necessario, tome R-> Inf depois.)
> -- Escolher 3 pontos uniformemente numa circunferencia de raio R, que
> seriam os vertices. (Se necessario, tome R-> Inf depois.)
>
> Todas estas opcoes sao razoaveis para interpretar "ao acaso", mas nao
> levam aa mesma resposta... :(
>
> Abraco, Ralph.
>
> On Thu, Sep 26, 2019 at 5:12 PM João Maldonado <
> joao_maldona...@hotmail.com> wrote:
>
>> Eaí galera.
>> Fica um problema legal de probabilidade pra vocês resolverem (e me
>> ajudarem).
>>
>> Um triângulo é dito aproximadamente equilátero quando o maior de seus
>> lados não excede o menor por 10%. Um triângulo é selecionado ao acaso. Qual
>> a chance de ele ser aproximadamente equilátero?
>>
>> Pensei em prosseguir da seguinte forma.
>> Sendo c o maior lado, b o do meio e a o menor. Sendo P1 a probabilidade
>> pedida.
>> Temos que:
>> P1=6*P(a<= b<= c<= mín(1.1a, a+b))= 6*P(a<=b<=c<=1.1a)
>>
>> Aí eu pensei em pegar um cubo de lado 1 e fazer uma integral tripla com
>> esses limites.
>> O problema é que um cubo de lado 1 não é um subespaço amostral
>> equivalente. Pense comigo:
>> Se tivermos um dos lados valendo 0.9  o outro 0.7, o terceiro poderia
>> valer até 1.6, e isso estaria fora do cubo, mesmo os dois primeiros estando
>> dentro. Dessa forma não saberíamos qual seria o "denominador" do nosso P.
>>
>> Alguém consegue me ajudar?
>>
>>
>> --
>> Esta mensagem foi verificada pelo sistema de antivírus e
>> acredita-se estar livre de perigo.
>>
>
> --
> Esta mensagem foi verificada pelo sistema de antivírus e
> acredita-se estar livre de perigo.

-- 
Esta mensagem foi verificada pelo sistema de antiv�rus e
 acredita-se estar livre de perigo.



Re: [obm-l] Complexos

2019-09-08 Por tôpico Claudio Buffara
Esta tem uma demonstração bonitinha usando um retângulo dividido em 6 quadrados 
congruentes da forma óbvia (2x3).

Enviado do meu iPhone

Em 8 de set de 2019, à(s) 19:57, Maikel Andril Marcelino 
 escreveu:

> Calcule (2+i)(3+i) e deduza que pi/4 = arctg(1/2) + arctg(1/3)
> -- 
> Esta mensagem foi verificada pelo sistema de antivírus e
> acredita-se estar livre de perigo.
> 
> 
> =
> Instruções para entrar na lista, sair da lista e usar a lista em
> http://www.mat.puc-rio.br/~obmlistas/obm-l.html
> =

-- 
Esta mensagem foi verificada pelo sistema de antiv�rus e
 acredita-se estar livre de perigo.


=
Instru��es para entrar na lista, sair da lista e usar a lista em
http://www.mat.puc-rio.br/~obmlistas/obm-l.html
=


Re: [obm-l] Trigonometria

2019-09-02 Por tôpico Claudio Buffara
Tudo o que você precisa está nas primeiras duas páginas daqui:
http://people.math.sc.edu/filaseta/gradcourses/TheMath784Notes.pdf

On Mon, Sep 2, 2019 at 8:34 AM Israel Meireles Chrisostomo <
israelmchrisost...@gmail.com> wrote:

> Alguém sabe se existe sen(pi/n) racional para n suficientemente grande?
> --
> Israel Meireles Chrisostomo
>
>
> 
>  Livre
> de vírus. www.avg.com
> .
> <#m_829828656712733292_DAB4FAD8-2DD7-40BB-A1B8-4E2AA1F9FDF2>
>
> --
> Esta mensagem foi verificada pelo sistema de antivírus e
> acredita-se estar livre de perigo.

-- 
Esta mensagem foi verificada pelo sistema de antiv�rus e
 acredita-se estar livre de perigo.



Re: [obm-l]

2019-08-30 Por tôpico Claudio Buffara
Acho que também dá pra se inscrever como aluno de cursos livres. Os
créditos assim adquiridos poderão ser usados posteriormente caso você
decida se inscrever no mestrado.

On Fri, Aug 30, 2019 at 4:40 PM Michel Torres  wrote:

> Ola,
> Até onde eu sei, voce deve se inscrever no programa de verão do IMPA e
> depois solicitar a inscrição no programa de mestrado.
> pelo menos era assim.
> mas se não me engano ( faz muito tempo que não acesso o sistema academico)
> voce pode se inscrever direto no programa de mestrado mas tem que preencher
> alguns requisitos.
>
> ante de mais nada vc deve se cadastrar no sistema academico mesmo para
> participar do programa de verão
>
> abs
>
> On Fri, Aug 30, 2019 at 4:21 PM Gabriel Lopes  wrote:
>
>> Prezados, boa tarde.Estou um pouco atrasado na minha formação mas
>> gostaria de recuperar um pouco desse tempo fazendo materias de mestrado no
>> IMPA, alguem sabe se isso é possivel e qual seria os procedimentos e pré
>> -requisitos para tal.
>>
>> Att. Gabriel
>>
>> --
>> Esta mensagem foi verificada pelo sistema de antivírus e
>> acredita-se estar livre de perigo.
>
>
> --
> Esta mensagem foi verificada pelo sistema de antivírus e
> acredita-se estar livre de perigo.

-- 
Esta mensagem foi verificada pelo sistema de antiv�rus e
 acredita-se estar livre de perigo.



[obm-l] Re: [obm-l] Re: [obm-l] Re: [obm-l] Números eficientes

2019-08-30 Por tôpico Claudio Buffara
"Se x é eficiente então x(x-1) é múltiplo de 1000" : bela sacada!

On Fri, Aug 30, 2019 at 4:09 PM Luiz Gustavo Alves Brandão <
luizbg...@gmail.com> wrote:

> Se x é eficiente então x(x-1) é múltiplo de 1000. Como x e x-1 são
> coprimos, um deles é 8A e o outro é 125B, com A e B inteiros e B ímpar.
> Sendo assim, só é preciso testar B = 1, 3, 5 e 7, que nos fornece os
> números eficientes 376 e 625.
> Qualquer erro só avisarem...
>
> Em sex, 30 de ago de 2019 às 14:52, Claudio Buffara <
> claudio.buff...@gmail.com> escreveu:
>
>> Achar estes números com uma planilha deve ser mais rápido do que fazer a
>> análise usando congruências.
>>
>> On Fri, Aug 30, 2019 at 2:01 PM Carlos Monteiro <
>> cacacarlosalberto1...@gmail.com> wrote:
>>
>>> Um número x de 3 algarismos é dito eficiente se os 3 últimos algarismos
>>> de x^2 são os mesmos algarismos de x e na mesma ordem. Encontre todos os
>>> números eficientes.
>>>
>>> --
>>> Esta mensagem foi verificada pelo sistema de antivírus e
>>> acredita-se estar livre de perigo.
>>
>>
>> --
>> Esta mensagem foi verificada pelo sistema de antivírus e
>> acredita-se estar livre de perigo.
>
>
> --
> Esta mensagem foi verificada pelo sistema de antivírus e
> acredita-se estar livre de perigo.

-- 
Esta mensagem foi verificada pelo sistema de antiv�rus e
 acredita-se estar livre de perigo.



  1   2   3   4   5   6   7   8   9   10   >